Sei sulla pagina 1di 227

Capı́tulo 1

Heurı́stica

La estrategia o la táctica para resolver problemas es llamada heurı́stica.


En este capı́tulo nos ocuparemos de la heurı́stica para resolver problemas
matemáticos. Aquellos que han pensado sobre la heurı́stica han descrito un
número de ideas básicas que son tı́picamente usuales. Los cinco clásicos de
la resolución de problemas de George Polya son obras maestras dedicadas
enteramente al estudio práctico de la heurı́stica en matemáticas. Entre las
ideas desarrolladas en estos libros, nos enfocaremos a las siguientes:

1. Búsqueda de un patrón.
2. Dibujo de una figura.
3. Formulación de un problema equivalente.
4. Modificación del problema.
5. Elección de una notación efectiva.
6. Explotación de la simetrı́a.
7. División en casos.
8. Trabajo hacia atrás.
9. Argumentación por contradicción.

1
2 CAPÍTULO 1. HEURÍSTICA

10. Búsqueda de paridad.

11. Consideración de casos extremos.

12. Generalización.

Nuestro interés en esta lista de ideas para resolver problemas no es describir-


las sino implementarlas. Al examinar ejemplos de cómo otros han utilizado
estas sencillas pero poderosas ideas, podemos esperar mejorar nuestras ha-
bilidades para resolver problemas.

Antes de comenzar, un consejo acerca de los problemas al final de las sec-


ciones: no se preocupe por aplicar la heurı́stica tratada en esa sección. Aunque
los problemas están escogidos para practicar el uso de la heurı́stica en cuestión,
un enfoque estrecho puede ser psicológicamente debilitante. Un mismo prob-
lema admite usualmente varias soluciones, a veces empleando diferentes heurı́sti-
cas. Ası́, es mejor atacar cada problema con la mente abierta que con una
noción preconcebida de cómo una heurı́stica en particular debiera ser apli-
cada. Al trabajar en un problema, resolverlo es lo importante. Es la ex-
periencia acumulada de todas las ideas trabajando simultáneamente lo que
terminará en una mejor comprensión de las posibilidades de resolución de un
problema.

1.1. Búsqueda de un Patrón

Virtualmente todos los que resuelven un problema comienzan su análisis


dándose una idea del problema, convenciéndose de la plausibilidad del resul-
tado. Esto se hace mejor examinando los casos especiales más inmediatos;
cuando esta exploración es llevada a cabo de manera sistemática, pueden
surgir patrones que sugieren ideas para proceder con el problema.

1.1.1 Demuestre que un conjunto con n elementos (diferentes) tiene exac-


tamente 2n subconjuntos (diferentes).
1.1. BÚSQUEDA DE UN PATRÓN 3

Cuando el problema está dado en este modo imperativo, un principiante


puede asustarse y no saber como proceder. Suponga, sin embargo, que el
problema fuera propuesto como una indagación, tal como:

(i) Cuántos subconjuntos pueden formarse de un conjunto de n elementos?

(ii) Demuestre o dé un contraejemplo: Un conjunto con n elementos tiene 2n


subconjuntos.

En cualquiera de estas formas ya está implı́cita la sugerencia de que de-


berá comenzar revisando unos cuantos casos particulares. Ası́ es como cada
problema debiera ser atacado: permanecer escéptico respecto al resultado
hasta no estar plenamente convencido.

Solución 1. Empezaremos por analizar lo que sucede cuando el conjunto


contiene 0, 1, 2, 3 elementos; los resultados se muestran en la siguiente tabla:

n elementos subconjuntos núm.de subconjuntos


0 ninguno ∅ 1
1 x1 ∅, {x1 } 2
2 x1 , x2 ∅, {x1 }, {x2 }, {x1 , x2 } 4
3 x1 , x2 , x3 {x3 }, {x1 , x3 }, {x2 , x3 }, {x1 , x2 , x3 }, ∅, {x1 }, {x2 }, {x1 , x2 } 8

Nuestro propósito al construir esta tabla no es sólo verificar el resultado, sino


también buscar patrones que pudieran sugerir cómo proceder en el caso gen-
eral. Ası́, nuestro objetivo es ser tan sistemático como sea posible. En este
caso, nótese que cuando n = 3, hemos enlistado primero los subconjuntos de
{x1 , x2 } y luego, en la segunda lı́nea, cada uno de los subconjuntos aumen-
tados por el elemento x3 . Esta es la idea clave que nos permite proseguir con
valores más grandes de n. Por ejemplo, cuando n = 4, los subconjuntos de
S = {x1 , x2 , x3 , x4 } son los ocho subconjuntos de {x1 , x2 , x3 } (mostrados en
la tabla) junto con los ocho formados al agregar x4 a cada uno de éstos. Estos
dieciseis subconjuntos construyen toda la colección de posibilidades; ası́, un
conjunto de 4 elementos tiene 24 (= 16) subconjuntos.

Una demostración basada en esta idea es una aplicación fácil de inducción


matemática (ver sección 2.1)

Solución 2. Otro camino para presentar la idea de la última solución es ar-


gumentar como sigue. Para n, sea An el número de subconjuntos (diferentes)
de un conjunto con n elementos. Sea S un conjunto con n + 1 elementos,
4 CAPÍTULO 1. HEURÍSTICA

y desı́gnese uno de sus elementos como x. Hay una correspondencia uno a


uno entre aquellos subconjuntos de S que no contienen a x (propiamente,
un subconjunto T del tipo anterior se corresponde con T ∪ {x} ). Los tipos
anteriores son todos subconjuntos de S − {x}, un conjunto con n elementos,
y ası́, debe darse el caso de que An+1 = 2An .

Esta relación de recurrencia, verdadera para n = 0, 1, 2, 3, ..., combinando


con el hecho A0 = 1, implica que An = 2n . (An = 2An−1 = 22 An−2 = ... =
2n A0 = 2n )

Solución 3. Otra enumeración sistemática de los subconjuntos puede ser


llevada a cabo construyendo un “árbol ” .

Cada rama del árbol corresponde a un subconjunto diferente de S (la barra


sobre el nombre del elemento significa que no está incluido en el subconjunto
correspondiente a esa rama). El árbol está construido en tres etapas, corre-
spondientes a los tres elementos de S. Cada elemento de S lleva a dos posi-
bilidades: está contenido en el subconjunto de S o no lo está, y estas opciones
están representadas por dos ramas. Cuando cada elemento es considerado,
el número de ramas se duplica. Ası́, para un conjunto de tres elementos el
número de ramas es 2×2×2 = 8. Para un conjunto de n elementos el número
de elementos es
n
2| × 2 ×
{z... × 2} = 2 ;
n veces

ası́, un conjunto con n elementos tiene 2n subconjuntos.

Solución 4. Supóngase que enumeramos los subconjuntos de acuerdo a su


tamaño. Por ejemplo, cuando S = {a, b, c, d}, los subconjuntos son:

núm. de elementos núm. de subconjuntos


0 ∅ 1
1 {a}, {b}, {c}, {d} 4
2 {a, b}, {a, c}, {a, d}, {b, c}, {b, d}, {c, d} 6
3 {a, b, c}, {a, b, d}, {a, c, d}, {b, c, d} 4
4 {a, b, c, d} 1

Este comienzo podrı́a propiciar el siguiente argumento. Sea S P


un conjunto
con n elementos. Entonces el número de subconjuntos de S = n0 (número
1.1. BÚSQUEDA DE UN PATRÓN 5

de subconjuntos de S con k elementos).


n  
X n
= = 2n
0
k

El paso final en esta cadena de igualdades se sigue del teorema del binomio,
n  
n
X n k n−k
(x + y) = x y ,
0
k

haciendo x = 1 y y = 1

Solución 5. Otro comienzo sistemático está ilustrado en la siguiente tabla,


donde se enlistan los subconjuntos de S = {x1 , x2 , x3 }. Para entender el pa-
trón aquı́, nótese la correspondencia entre los objetos de la columna extrema
izquierda y la ocurrencia de 1´s en la segunda columna de triadas

Subconjunto Triada Núm. binario Núm. decimal


∅ (0,0,0) 0 0
{x3 } (0,0,1) 1 1
{x2 } (0,1,0) 10 2
{x2 , x3 } (0,1,1) 11 3
{x1 } (1,0,0) 100 4
{x1 , x3 } (1,0,1) 101 5
{x1 , x2 } (1,1,0) 110 6
{x1 , x2 , x3 } (1,1,1) 111 7

Especı́ficamente, si A es un subconjunto de S = {x1 , x2 , ...., xn }, defı́nase ai ,


para i = 1, 2, ..., n, como n
ai = 01 sisi xxii ∈∈/ A
A

Es claro que ahora podemos identificar un subconjunto A de S , con (a1 , a2 , ..., an ),


una n-ada de 0’s y 1’s. Inversamente a cada n-upla le corresponde un único
subconjunto de S. Luego el número de subconjuntos de S es igual al número
de n-uplas de 0′s y 1′s. Este último conjunto está obviamente en corre-
spondencia uno-a-uno con el conjunto de los números binarios no negativos
6 CAPÍTULO 1. HEURÍSTICA

menores que 2n . Luego a cada entero no negativo menor que 2n le corresponde


exactamente un subconjunto de S, e inversamente. Por lo tanto, debe suceder
que S tenga 2n subconjuntos.

Normalmente, daremos una solución a cada ejemplo, una solución que sirva
para ilustrar la heurı́stica bajo consideración. En este primer ejemplo, sin
embargo, sencillamente querı́amos reiterar la observación anterior de que un
mismo problema pueda ser trabajado en una variedad de formas. La lección
que debe aprenderse es que uno debe permanecer flexible en las primeras
etapas de exploración de problemas. Si un acercamiento no parece llevar a
ninguna parte, no hay que desesperarse, sino hay que buscar una idea nueva.
No se quede fijo en una sola idea hasta que no haya tenido la oportunidad
de pensar ampliamente en una variedad de acercamientos alternativos.

1.1.2 Denótese por Sn,0 , Sn,1, Sn,2 a la suma de cada tercer elemento en el
n-ésimo renglón del triángulo de pascal, comenzando por la izquierda con el
primer elemento, el segundo elemento, y el tercer elemento respectivamente.
Haga una conjetura sobre el valor de S100,1

Triángulo de Pascal n Sn,0 Sn,1 Sn,2


1 0 1+ 0 0
11 1 1 1 0−
121 2 1 2+ 1

1331 3 2 3 3
14641 4 5 5 6+
1 5 10 10 5 1 5 11 10− 11
1 6 15 20 15 61 6 22+ 21 21
1 7 21 35 35 21 7 1 7 43 43 42−

Solución. Empezaremos por examinar las casillas de orden inferior con la


esperanza de encontrar patrones que pudieran generalizarse. En la tabla 1.2,
los términos que están subrayados con una sola raya y doblemente subraya-
dos son los sumandos de Sn,1, Sn,2 , respectivamente. Las tres columnas de la
derecha muestran que, en cada caso, dos de las sumas son iguales, mientras
que la tercera es mayor (indicada por un supraı́ndice +) o menor (indicada
por un supraı́ndice -). También se ve que el término desigual en esta suce-
sión cambia en un ciclo de seis. Ası́, del patrón establecido en los primeros
1.1. BÚSQUEDA DE UN PATRÓN 7

renglones, esperamos que la anomalı́a para n = 8 ocurra en la columna de


enmedio y sea uno más que las otras dos.

Sabemos que Sn,0 + Sn,1 + Sn,2 = 2n (ver 1.1.1). Ya que 100 = 6 × 16 + 4,


esperemos que el término desigual ocurra en la tercera columna (S100,2 ) y que
sea uno más que las otras dos. Ası́ S100,0 = S100,1 = S100,2 −1 y S100,1 +S100,1 +
100
S100,1 +1 = 2100 . Estas ecuaciones nos llevan a conjeturar que S100,1 = (2 3−1)

Una demostración formal de esta conjetura es una aplicación directa de in-


ducción matemática (ver capı́tulo 2).

1.1.3 Establezca las condiciones necesarias y suficientes sobre x1 y x2 para


que xn sea un número entero para una infinidad de valores de n. Donde
x1 , x2 , x3 , ... es una sucesión de números reales distintos de cero que satisfacen
xn−2 xn−1
xn = n = 3, 4, 5, ...
2xn−2 − xn−1

Solución. Para darse una idea de la sucesión, calcularemos los primeros


términos, expresándolos en términos de x1 y x2 . Tenemos (omitiendo el álge-
bra)

x1 x2
x3 = ,
2x1 − x2
x1 x2
x4 = ,
3x1 − 2x2
x1 x2
x5 = .
4x1 − 3x2

Somos afortunados en este caso en particular de que los cálculos sean re-
alizables y de que emerja un patrón. Un sencillo argumento por inducción
establece que
x1 x2
xn = ,
(n − 1) x1 − (n − 2) x2
lo cual, al aislar el coeficiente para n toma la forma
x1 x2
xn = .
(x1 − x2 ) n + (2x2 − x1 )
8 CAPÍTULO 1. HEURÍSTICA

De este modo, vemos que si x1 6= x2 , el denominador excederá eventualemente


al numerador, en magnitud, de forma que xn no será un entero. Sin embargo,
si x1 = x2 , todos lo términos de la sucesión son iguales. Ası́, xn es un entero
para un número infinito de valores de n si y sólo si x1 = x2 .

1.1.4 Encuentre números positivos n y a1 , a2 ..., an tales que a1 + a2 + ... +


an = 1000 y el producto a1 a2 ...an sea tan grande como se pueda.

Solución. Cuando un problema involucra un párametro que hace el análi-


sis complicado, frecuentemente es de ayuda en la etapa de descubrimiento
el reemplazo temporal con algo más manejable. En este problema, podemos
comenzar por examinar una sucesión de casos particulares obtenidos al reem-
plazar 1000 por 2, 3, 4, 5, 6, 7, 8, 9, ... . En esta forma llegamos a descubrir que
en un producto máximo

(i) ningún ai será mayor que 4,

(ii) ningún ai será igual a 1,

(iii) todas las ai ′ s pueden ser tomadas como 2 ó 3 (porque 4 = 2 × 2 y


4 = 2 + 2),

(iv) a los más dos ai ′ s serán iguales a 2 (porque 2 ×2 × 2 < 3 × 3 y


2 + 2 + 2 = 3 + 3).

Cada una de éstas es fácil de establecer. Ası́ cuando el parámetro es 1000


como en el problema en cuestión, el producto máximo debe ser 3332 × 22 .

1.1.5 Muestre que x ∗ y = y ∗ x para toda x, y en S. Donde S un conjunto


y ∗ una operación binaria sobre S que satisface las dos leyes

x∗x = x
(x ∗ y) ∗ z = (y ∗ z) ∗ x

Solución. La solución, que aparece tan transparentemente abajo, es en re-


alidad el resultado de una cantidad considerable de trabajo; el procedimento
puede ser descrito sólo como la búsqueda de un patrón (el patrón guı́a está en
1.1. BÚSQUEDA DE UN PATRÓN 9

la naturaleza cı́clica de los factores en la segunda condición). Tenemos, para


toda x, y en S,

x ∗ y = (x ∗ y) ∗ (x ∗ y) = [y ∗ (x ∗ y)] ∗ x = [(x ∗ y) ∗ x] ∗ y
= [(y ∗ x) ∗ x] ∗ y = [(x ∗ x) ∗ y] ∗ y = [(y ∗ y)] ∗ (x ∗ x)
= y ∗ x.

Problemas

Desarrolle una idea para los siguientes problemas mediante la búsqueda de


patrones. Haga conjeturas apropiadas, y piense cómo podrı́an llevarse a cabo
las demostraciones.

1.1.6 Comenzando con 2 y 7, la sucesión 2, 7, 1, 4, 7, 4, 2, 8, ... se construye


al multiplicar parejas sucesivos de sus términos y añadiendo el resultado como
el siguiente término o los dos siguientes términos de la sucesión, dependiendo
de si el producto es un número de uno o dos dı́gitos. Demuestre que el dı́gito
6 aparece un número infinito de veces en la sucesión.

1.1.7 Denótese por S1 a la sucesión de los enteros positivos 1, 2, 3, 4, 5, 6, ...,


y defı́nase Sn+1 en términos de Sn añadiendo 1 a aquellos enteros en Sn que
son divisibles por n. Ası́, por ejemplo, S2 es 2, 3, 4, 5, 6, 7, ..., S3 es 3, 3, 5, 5, 7, 7, ...
Determine aquellos enteros n con la propiedad de que los primeros n − 1 en-
teros en Sn son n.

1.1.8 Demuestre que puede hacerse una lista de todos los subconjuntos de
un conjunto finito de tal forma que

(i) El conjunto vacı́o sea primero en la lista.

(ii) Cada subconjunto aparezca sólo una vez, y

(iii) Cada subconjunto en la lista se obtenga ya sea al añadir un elemen-


to al subconjunto precedente o al borrar un elemento del subconjunto
precedente.
10 CAPÍTULO 1. HEURÍSTICA

1.1.9 Determine el número de coeficientes binomiales impares en la expan-


sión de (x + y)100 .

1.1.10 Un teorema bien conocido asegura que un primo p > 2 puede ser
escrito como la suma de dos cuadrados perfectos (p = m2 + n2 , con m y n
enteros) si y sólo si p es uno más que un múltiplo de 4. Haga una conjetura
sobre cuáles primos p > 2 pueden ser escritos en cada una de las siguientes
formas, utilizando enteros x y y (no necesariamente positivos): (a) x2 +16y 2,
(b) 4x2 + 4xy + 5y 2 . (ver 1.5.10)

1.1.11 Si (an ) es una sucesión tal que para n ≥ 1, (2 − an )an+1 = 1,


qué sucede con an cuando n tiende hacia infinito?

1.1.12 Sea S un conjunto, y sea ∗ una operación binaria sobre S que sat-
isface las leyes

x ∗ (x ∗ y) = y
(y ∗ x) ∗ x = y

Demuestre que x ∗ y = y ∗ x para todas x, y en S.

Ejemplos adicionales

La mayor parte de los problemas de inducción están basados en el descubrim-


iento de un patrón. Ası́, los problemas en las secciones 2.1, 2.2, 2.3, 2.4 ofre-
cen práctica adicional para esta heurı́stica. Véase también 1.7.2, 1.7.7, 1.7.8,
2.5.6.

1.2. Dibujo de una Figura

Es de mucha ayuda describir un problema gráficamente siempre que sea posi-


ble, por medio de una figura, un diagrama, o una gráfica. Una representación
gráfica usualmente hace más fácil asimiliar los datos relevantes y notar las
relaciones y las dependencias.
1.2. DIBUJO DE UNA FIGURA 11

1.2.1 Una cuerda de longitud constante se desliza a lo largo de un semicı́rcu-


lo. El punto medio de la cuerda y las proyecciones de sus extremos son los
vértices de un triángulo. Demuestre que el triángulo es isósceles y que nunca
cambia de forma.
X M
Y

A C N D B
Figura 1.1

Solución. Denótese por AB la base del semicı́rculo, y sean XY la cuerda,


M el punto medio de XY , C y D las proyecciones de X y Y sobre AB (fig.
1.2) Denótese la proyección de M sobre AB como N. Entonces N es el punto
medio de CD y se sigue que el triángulo ∆CMD es isósceles.

Para mostrar que la forma del triángulo es independiente de la posición de


la cuerda, basta mostrar que el ∠MCD permanece sin cambio, o equivalen-
temente, que el ∠XCM es constante, para todas las posiciones de XY . Para
ver que tal es el caso, extiéndase XC hasta cortar el cı́rculo completo en Z
(fig. 1.3). Entonces CM es paralela a ZY (C y M son los puntos medios de
XZ y XY respectivamente), y consecuentemente ∠XCM = ∠XZY es igual
a la mitad del arco XY , y este arco depende sólo de la longitud de la cuerda
XY . Esto completa la demostración.
X
M
b

Y b

A b

C
b

D
b b

Z
b

Figura 1.2

Uno podrı́a preguntarse: Cómo fue que se le ocurrió a alguien extender XC


12 CAPÍTULO 1. HEURÍSTICA

de esta manera? Este es precisamente el paso que hace el argumento tan


bonito, y es ciertamente un paso difı́cil de motivar. Casi todo lo que puede
decirse es que el uso de lı́neas y arcos auxiliares (frecuentemente encontrados
por reflexión, extensión, o rotación) es una práctica común en geometrı́a. El
sólo hecho de tomar esto en cuenta contribuirá a las posibles aproximaciones
a un problema dado.
X
M
b

Y b

A b

C
b b

O D
b b

Z
b

Figura 1.3

Otro acercamiento interesante a este problema es dar coordenadas a los pun-


tos y proceder analı́ticamente. Para mostrar que el triángulo es independiente
de la posición de la cuerda, basta con mostrar que la razón entre la altura y
la base, MN/CD, es constante.

Sea O el punto medio de AB, y sea θ = ∠Y OB. Es claro que toda la config-
uración está determinada por θ (fig. 1.4)

Sea α = ∠XOY. Usando esta notación,

CD = cosθ − cos(θ + α)
senθ + sen(θ + α)
MN = .
2

y la razón altura-base es
senθ + sen(θ + α)
F (θ) = , 0≤θ ≤π−α
2(cos θ − cos (θ + α))
No es inmediatamente claro que esta cantidad es independiente de θ; este es
el contenido de 1.8.1 y 6.6.7
1.2. DIBUJO DE UNA FIGURA 13

1.2.2 Una partı́cula que se mueve sobre una lı́nea recta comienza del re-
poso y alcanza una velocidad v0 después de recorrer una distancia s0 . Si el
movimiento es tal que la aceleración nunca aumenta, encuentre el tiempo
máximo para el recorrido.
3 V
V0 b b
P
2 b

1 S0

t
b

O
1 2 3 4 5 6
−1 Figura 1.4

Solución. Concentre la atención en la gráfica de la velocidad v = v(t) (fig.


1.5). Tenemos que v(0) R t = 0, y la gráfica de v nunca es cóncava hacia arriba
(distancia recorrida= 0 v(t)dt). A partir de esta representación, es claro que
maximizaremos el tiempo de recorrido cuando la curva v(t) de 0 a P sea una
lı́nea recta (fig. 1.6). En el tiempo máximo t0 , 21 t0 v0 = s0 , o equivalentemente,
t0 = 2s0 /v0 .
3 V
V0 b b
P
2

1
S0
t
b b

O
1 2 3 4 5 6
−1 Figura

1.2.3 Si a y b son enteros positivos sin factores comunes, muestre que

 a   2a  hh ii (a − 1)(b − 1)
(b−1)a
b
+ b
+ ... + b
=
2

Solución. Cuando b = 1, vemos que la suma de la izquierda es 0 y ası́ se


cumple el resultado.
14 CAPÍTULO 1. HEURÍSTICA

No es claro cómo una figura podrı́a ser de ayuda para establecer esta iden-
tidad puramente aritmética. Además, la afirmación involucra dos variables
independientes, a y b, y ab , 2a
b
, 3a
b
, .... son los valores de la función f (x) = ax
b
cuando x = 1, 2, 3, ...., respectivamente.
   
Es posible interpretar ab , 2a b
, .... geométricamente?

Para concretar, considérese el caso a = 5 y b = 7. Los  5kpuntos


 Pk = (k, 5k/7), k =
1, 2, ..., 6, están cada uno en la recta y = 5k/7, y 7 es igual al número de
puntos de la retı́cula en la lı́nea
P vertical
 5k  a través de Pk que está por encima
del eje x y abajo de Pk . Ası́, k=1 6 7
es igual al número de puntos de la
retı́cula que están dentro del ∆ABC (ver fig. 1.7). Por simetrı́a, este número
es la mitad del número de púntos de la retı́cula que están en el interior del
rectángulo ABCD. Hay 4 × 6 = 24 puntos de la retı́cula en ABCD, lo que
significa que el triángulo ABC contiene 12 puntos de la retı́cula en su interior.

5 b b b b b b b

4 b b b b b b b

3 b b b b b b b

2 b b b b b b b

1 b b b b b b b

1 2 3 4 5 6 7
−1 Figura

El mismo argumento puede aplicarse en el caso general. La condición de que


a y b no tienen factores comunes nos asegura que ninguno de los puntos de
la retı́cula interior de ABCD caerá en la recta y = ax/b. Ası́,

b−1  
X ka 1
= k
k=1
b 2
1.2. DIBUJO DE UNA FIGURA 15

(donde k es el número de puntos de la retı́cula en el interior de ABCD)


(a − 1)(b − 1)
=
2

1.2.4 (El problema de los saludos de mano) El Sr. y la Sra. Adams


fueron hace poco a una fiesta en la que habı́a otras tres parejas. Ocurrieron
varios saludos de mano. Nadie saludó a su propio cónyugue, ni a la misma
persona dos veces, y por supuesto, nadie se saludó a si mismo.

Después de que los saludos hubieron terminado, el Sr. Adams le preguntó a ca-
da persona, incluida su esposa, a cuántas personas habı́a saludado de mano?.
Para sorpresa suya, cada quien dió una respuesta diferente. A cuántas per-
sonas saludó de mano la Sra. Adams?

Solución. Aunque un diagrama no es propiamente la solución , es de gran


ayuda visualizar los datos gráficamente de la siguiente manera. Represéntese
a los ocho individuos con los ocho puntos que muestra la figura 1.7

Ahora bien, las respuestas a la encuesta del Sr. Adams deben haber sido
los números 0, 1, 2, 3, 4, 5, 6. Ası́, uno de los individuos, A digamos, saludó a
otros seis, digamos B, C, D, E, F, G. Indı́quese esto en la gráfica dibujando
segmentos desde A a estos puntos, como en la figura 1.8.

En este diagrama vemos que H debe haber sido la persona que no saludó a
nadie. Además, A y H deben ser esposos, pues A ha saludado a seis personas,
sin contar a su cónyugue.

Por suposición, alguno entre B, C, D, E, F, G ha saludado a cinco personas.


Re-etiquetando de ser necesario, podemos suponer que se trata de la persona
B saludó son A, C, D, E, F. Esto se muestra en la figura 1.9. A partir de este
dibujo vemos fácilmente que G es la única persona que pudo haber contestado
üno”, y B y G deben estar casados.

De nuevo, como antes, re-etiquetando los puntos C, D, E, F de ser necesario,


podemos suponer que C saludó a cuatro personas y que éstas eran A, B, D, E.
El diagrama correspondiente se muestra en la figura 1.10. Usando el mismo
razonamiento, F y C son esposos, y consecuentemente, D y E son esposos.
16 CAPÍTULO 1. HEURÍSTICA

D y E han ambos saludado a otros tres. Dado que el Sr. Adams no recibió dos
respuestas ”tres”, D y E deben corresponder al Sr. y la Sra. Adams ; es decir,
el Sr. Adams saludó a otros tres.
A B b
A b B b b

H C H
b b b b
C

b
D b

G
b
C b

D
b b b b

F E F E
Figura Figura 1.8
1.7

A b b B A
b b B

H H
b b
C b b
C

b b

G
b
D G b

D
b
F Eb b

F
b

E
Figura 1.9 Figura 1.10

Problemas
1.2. DIBUJO DE UNA FIGURA 17

1.2.5 Dos postes, de alturas a y b, están a una distancia d (con respecto


al nivel del suelo). Un alambre se extiende desde la parte superior de cada
uno de ellos a un punto P en el suelo entre ellos. En dónde deberı́a estar
localizado P para minimizar la longitud total del cable? (Sugerencia: Suponga
que los postes se encuentran en los puntos C y D, que sus partes superiores
son A y B, respectivamente. Deseamos minimizar AP + P B. Aumente este
diagrama reflejándolo en la lı́nea base CD. Suponga que B se refleja a B ′
(P B = P B ′ ). Ahora el problema es: En dónde deberı́a estar localizado P para
minimizar AP + P B ′ ?)

1.2.6 Sea ABC un triángulo de ángulos agudos, y sea D en el interior


del segmento AB. Localice los puntos E sobre AC y F sobre CB tales que
el triángulo inscrito DEF tenga perı́metro mı́nimo.(Sugerencia: Refleje D
sobre la lı́nea AC a un punto D ′ ; refleje D sobre CB a un punto D ′′ y
considere el segmento D ′ D ′′ ).

1.2.7 En un cuarto rectangular que mide 30 pies de largo y 12 pies de


altura, y 12 pies de ancho. Una mosca, con un ala rota, se encuentra en un
punto a la mitad de una barda cuadrada y un pie abajo del techo. Un pedazo
de comida se encuentra localizado un pie arriba del piso y a la mitad de la
barda opuesta. La mosca tiene únicamente la energı́a para caminar 40 pies.
Muestre que hay un camino por el cual la mosca pueda caminar y conseguir
la comida.

1.2.8 Los triángulos equilateros ABP y ACQ están construidos externa-


mente sobre los lados AB y AC del triángulo ABC. Pruebe que CP = BQ.
(Sugerencia: Para encontrar una solución bonita, rote el plano del triángu-
lo 600 por el punto A, en una dirección que lleve B en la dirección de C.
Qué sucede con el segmento CP ?)

1.2.9 Sean a y b números enteros positivos tales que a < b. Si dos puntos
son seleccionados al azar de un segmento de longitud b, cuál es la probabilidad
de que la distancia entre ellos sea al menos a? (Sugerencia: Sean x y y los
números escogidos al azar del intervalo [0, b], y considere a estas variables
aleatorias independientes sobre dos ejes separados. Qué área corresponde a
|x − y| ≥ a?).
18 CAPÍTULO 1. HEURÍSTICA

1.2.10 Dé una interpretación geométrica al siguiente problema. Sea f difer-


enciable con f continua en [a, b]. Muestre que si existe un número c en (a, b]

tal que f (c) = 0, entonces podemos encontrar un número d en (a, b) tal que
f (d) − f (a)
f ′ (d) = .
b−a

1.2.11 Sea a, b números reales, a < b. Indique geométricamente la ubi-


cación precisa de cada uno de los siguientes números: (a+b)
2
(= 21 a + 12 b); 32 a +
1
3
b; 13 a + 23 b; [m/(m + n)]a + [n/(m + n)]b, donde m > 0 y n > 0. Este último
número corresponde al centro de gravedad de un sistema de dos masas, uno
de masa m, localizado en a, y el otro, de masa n, localizado en b.

1.2.12 Use la gráfica de y = senx para mostrar lo siguiente. Dado un


triángulo ABC,
SenB+SenC
(a) ≤ Sen B+C
2 2
m n m n
(b) m+n
SenB + m+n
SenC ≤ Sen( m+n
B + m+n
C), m > 0, n > 0

1.2.13
PnUse Pun diagrama (un arreglo rectangular
Pn Pn(ai aj )) para mostrar que
n
(a) ai aj = i=0 ai aj ,
Pi=0
n Pj=0
n Pj=0
n Pn
(b) aa = ai aj ,
Pn i=j 2i j
j=0 Pn Pn i=0P j=iP
n i Pn 2
(c) ( i=0 ai ) = i=0 j=0 ai aj = 2 i=0 j=0 ai aj − i=0 ai .

Ejemplos adicionales

1.3.11, 1.9.2, 1.9.4, 1.11.3, 2.1.3, 2.5.5, 2.6.11

1.3. Formulación de un Problema Equivalente

El mensaje de la sección precedente es que el primer paso para resolver un


problema es juntar información, explorar, entender, relacionar, conjeturar,
1.3. FORMULACIÓN DE UN PROBLEMA EQUIVALENTE 19

analizar. Pero, qué pasa cuando no es posible hacer esto de manera significa-
tiva, sea porque los cálculos se vuelven muy complicados o porque el proble-
ma no admite casos especiales que nos hagan vislumbrar la solución? En esta
sección consideraremos algunos problemas de este tipo. La recomendación
de esta sección es tratar de reformular el problema a una forma equivalente
pero más simple. El llamado es a la imaginación y creatividad propia. Al-
gunas técnicas estándar de reformulación involucran cálculos algebraı́cos o
manipulaciones trigonométricas, substitución o cambio de variable, uso de
correspondencias biunı́vocas, y reinterpretación del lenguaje de otra materia
(álgebra, geometrı́a, análisis, combinatoria, etc.)

1.3.1 Encuentre una fórmula general para la n-ésima derivada de f (x) =


1/(1 − x2 ).

Solución. Un paso simplificador común al trabajar con funciones racionales


es escribir la función como una suma de fracciones parciales. En este caso,

 
1 1 1
f (x) = 2
+
1−x 1+x

y en esta forma es fácil mostrar que

 
(n) n! 1 (−1)n
f (x) = 2
+
(1 − x)n+1 (1 + x)n+1

1.3.2 Encuentre todas las soluciones de x4 + x3 + x2 + x + 1 = 0

Solución. Esta ecuación puede resolverse dividiendo entre x2 , luego sub-


stituyendo y = x + 1/x, y aplicando la fórmula para resolver ecuaciones de
segundo grado. Tenemos

 x2 + x12 + x + x1 + 1 = 0,
x2 + 2 + x12 + x + x1 + (1 − 2) = 0,
2 
x + x1 + x + x1 − 1 = 0,
y 2 + y − 1 = 0,
20 CAPÍTULO 1. HEURÍSTICA

Las raı́ces de esta ecuación son


√ √
−1 + 5 −1 − 5
y1 = , y2 = .
2 2

Sólo queda determinar x resolviendo las ecuaciones

1 1
x+ = y1 y x+ = y2 ,
x x

Que son equivalentes a

x2 − y1 x + 1 = 0 y x2 − y2 x + 1 = 0.

Las cuatro raı́ces encontradas al resolver éstas son

√ p √
−1 + 5 10 + 2 5
x1 = +i ,
4 4
√ p √
−1 + 5 10 + 2 5
x2 = −i ,
4 4
√ p √
−1 − 5 10 − 2 5
x3 = +i ,
4 4
√ p √
−1 − 5 10 − 2 5
x4 = −i .
4 4

Otra aproximación a este problema es multiplicar cada lado de la ecuación


original por x − 1. Ya que

(x − 1)(x4 + x3 + x2 + x + 1) = x5 − 1, un problema equivalente es encontrar


todas las x que satisfagan x5 = 1. Estas son las cinco raı́ces de la unidad.

Como una consecuencia de haber trabajado este problema en dos formas


diferentes, vemos que
1.3. FORMULACIÓN DE UN PROBLEMA EQUIVALENTE 21
√ p √
2 2 −1 + 5 10 + 2 5
Cos π + iSen π = +i .
5 5 4 4

Al igualar las partes real e imaginarias se obtiene

√ p √
−1 + 5 10 + 2 5
Cos720 = , Sen72 = i 0
4 4

(Fórmulas similares pueden ser obtenidas para x2 , x3 , y x4 )

1.3.3 Sea P un punto dentro de un triángulo dado ABC, D, E, F son los


pies de las perpendiculares desde P a las lı́neas BC, CA, AB, respectiva-
mente. Encuentre todos los P para los cuales
BC CA AB
+ +
PD PE PF
es mı́nimo.
b
A
b
F
c b
E q
b

r
P b

C
p b

a
b

B b
D
Figura 1.11

Solución. Denótense las longitudes de BC, AC, AB, por a, b, c, respectiva-


mente (ver figura 1.11). Queremos minimizar ap + qb + rc .

Notar que

Area ∆ABC = Area ∆BP C + Area ∆CAP + Area ∆ABP


1 1 1
= ap + bq + c
2 2 2
ap + bq + cr
=
2
22 CAPÍTULO 1. HEURÍSTICA

Ası́, ap + bq + cr es una constante, independientemente de la colocación de


P . Por lo tanto, en vez de minimizar ap + qb + rc , minimizaremos (ap + bq +
cr)( ap + qb + rc ). Se obtiene

(ap + bq + cr)( ap + qb + rc ) = a2 + b2 + c2 + ab( pq + pq ) + bc( qr + qr ) + ac( pr + pr ) ≥

a2 + b2 + c2 + 2ab + 2bc + 2ac = (a + b + c)2

La desigualdad en el segundo paso se sigue del hecho de que para cualesquiera


dos números positivos x y y tenemos xy + xy ≥ 2, con igualdad si y sólo si
x = y. Como resultado de este hecho, (ap + bq + cr)( ap + qb + rc ) alcanzará su
valor mı́nimo (a+b+c)2 cuando, y sólo cuando, p = q = r. Equivalentemente,
a
p
+ qb + rc alcanza un valor mı́nimo cuando P se encuentra en el incentro del
triángulo.

1.3.4 Probar que si m y n son enteros positivos y 1 ≤ k ≤ n, entonces

    
Pk n m n+m
i=0 =
i k−i k

Solución. La afirmación del problema constituye una de las ideas funda-


mentales que involucran coeficientes binomiales. En el lado izquierdo está una
suma de productos de coeficientes binomiales. Obviamente, una substitución
directa de factoriales en los coeficientes binomiales no nos da ninguna intro-
spección.

Muy seguido, las series finitas (especialmente aquellas que involucran coefi-
cientes binomiales) pueden ser sumadas combinatoriamente. Para entender
lo que se quiere decir aquı́, transforme el problema de series en un problema
de conteo de la siguiente manera. Sea S = A∪B, donde A es un conjunto con
n elementos y B es un conjunto, ajeno a A, con m elementos. Contaremos,
de dos formas diferentes, el número de  subconjuntos
 (distintos) de S con k
m+n
elementos. Por un lado este número es . Por otro lado, el número
k
  de Scon exactamente i elementos de A (y k − i elementos
de subconjuntos
n m
de B) es . Se sigue que
i k−i
1.3. FORMULACIÓN DE UN PROBLEMA EQUIVALENTE 23

 
m+n
= No. de k − subconjuntos de S.
k
Xk
= (No. de k − subconjuntos de S con i elementos de A)
i=0
k   
X n m
=
i=0
i k−i

Los problemas de conteo frecuentemente pueden ser simplificados ”Identi-


ficando” (por medio de una correspondencia uno-a-uno) los elementos del
conjunto con los de otros cuyos elementos puedan ser contados más fácil-
mente. Los siguientes tres ejemplos ilustran esta idea.

1.3.5 Sobre un cı́rculo se seleccionan n puntos y se dibujan las cuerdas que


los unen por pares. Suponiendo que no hay tres de estas cuerdas que sean
concurrentes (excepto en los extremos), cuántos puntos de intersección hay?
b

b
b

b b
b

b
b b

b
b

Figura 1.12

Solución. Los casos para n = 4, 5, 6 se muestran en la figura 1.12. Nótese


que cada punto de intersección (interior) determina, y está determinado por,
cuatro de los n puntos dados alrededor del cı́rculo (estos cuatro puntos pro-
ducirán en forma única dos cuerdas que se intersectarán  en el interior del
n
cı́rculo). Ası́, el número de puntos de intersección es .
4

1.3.6 Dado un entero positivo n, encuentre el número de cuádruplas de


enteros (a, b, c, d) tales que 0 ≤ a ≤ b ≤ c ≤ d ≤ n.
24 CAPÍTULO 1. HEURÍSTICA

Solución. La idea clave que hace el problema transparente es notar que hay
una correspondencia uno-a-uno entre las cuádruplas de nuestro conjunto y el
de subconjuntos de cuatro elementos tomados de {0, 1, ..., n + 3}. Especı́fica-
mente, identifı́quese a (a, b, c, d), 0 ≤ a ≤ b ≤ c ≤ d ≤ n, con el subconjunto
{a, b+1, c+2, d+3}. Es fácil ver que esta correpondencia es uno-a-uno. Cada
elemento de nuestro conjunto corresponde exactamente a un subconjunto  de
n+4
cuatro de {0, 1, ..., n + 3}, y viceversa. Ası́, el número deseado es .
4

1.3.7 El número cinco puede ser expresado como una suma de 3 números
naturales, tomando en cuenta el orden, en 6 maneras, especı́ficamente, como

5= 1+1+3 =1+3+1 =3+1+1 =1+2+2= 2+1+2= 2+2+1

Sean m y n números naturales tales que m ≤ n. De cuántas maneras puede


ser escrito n como una suma de m números naturales, tomando en cuenta
el orden?

Solución. Escribir n como una suma de n unos:

n = |1 + 1 +
{z... + 1}
n

 de maneras de escoger m − 1 signos


El número que buscamos es elnúmero
n−1
de suma de los n − 1; esto es .
m−1

Problemas

1.3.8 Mostrar que x7 + 2x5 + 10x2 − 1 no tiene ninguna raı́z mayor que 1.
(Sugerencia: ya que generalmente es más fácil probar que una ecuación no
tiene raı́ces positivas, nos vemos empujados a considerar el problema equiv-
alentemente obtenido al hacer la sustitución algebráica x = y + 1.)
1.3. FORMULACIÓN DE UN PROBLEMA EQUIVALENTE 25

1.3.9 El número tres puede ser expresado como la suma de uno o más
enteros positivos, tomando en cuenta el orden, de cuatro maneras, especı́fi-
camente, como 3, 1+2, 2+1, y 1+1+1. Mostrar que cualquier entero positivo
n puede ser ası́ expresado en 2n−1 formas.

1.3.10 De cuántas maneras puede ser expresado el 10 como una suma de


5 enteros no negativos, cuando se toma en cuenta el orden? (Sugerencia:
Encontrar un problema en el que la frase “5 enteros no negativos” sea reem-
plazada por 5 enteros “positivos”).

1.3.11 Para cuáles valores de a el sistema de ecuaciones

x2 = y 2
(x − a)2 + y 2 = 1

tiene exactamente cero, una, dos, tres, cuatro soluciones, respectivamente?


(Sugerencia: Traduzca el problema a un problema geométrico equivalente).

1.3.12 Dado n objetos arreglados en un renglón, un subconjunto de estos


objetos es llamado no amigable si nunca dos de sus elementos están con-
tiguos. Mostrar
 que el número
 de subconjuntos no amigables con k elementos
n−k+1
cada uno es . (Sugerencia: Adoptar una idea similar a la uti-
k
lizada en 1.3.6)

1.3.13 Sea a(n) el número de representaciones del entero positivo n como


una suma de 1’s y 2’s tomando el orden en cuenta. Sea b(n) el número
representaciones de n como una suma de enteros mayores que 1 , nuevamente
tomando el orden en cuenta y contando el sumando n. La tabla de abajo
muestra que a(4) = 5 y b(6) = 5 :
26 CAPÍTULO 1. HEURÍSTICA

a-sumas b- sumas
1+1+2 4+2
1+2+1 3+3
2+1+1 2+4
2+2 2+2+2
1+1+1+1 6

(a) Mostrar que a(n) = b(n + 2) para toda n, describiendo una


correspondencia uno-a-uno entre las a-sumas y las b- sumas.

(b) Mostrar que a(1) = 1, a(2) = 2, y para n > 2, a(n) = a(n − 1) + a(n − 2).

1.3.14 Hallando el área de un triángulo de dos formas diferentes, mostrar


que si p1 , p2 , p3 son las alturas de un triángulo y r es el radio del cı́rculo
inscrito, entonces 1/p1 + 1/p2 + 1/p3 = 1/r.

1.3.15 Usar
 un 
argumento
 de conteo
 para  probar
 que
 para enteros r, n,
r r+1 r+2 n n+1
0 < r ≤ n, + + + .... + = .
r r r r r+1

Ejemplos adicionales

Hay tantos ejemplos de esta heurı́stica que es difı́cil señalar aquellos que son
los más tı́picos. Dignos de atención son las demostraciones indirectas en las
secciones 1.10,1.11.

1.4. Modificación del Problema

En el transcurso del trabajo sobre un problema A podemos vernos llevados


a considerar otro problema B. Comúnmente, este cambio de problemas es
anunciado con frases tales como ’basta con mostrar que.....’ o ’sin pérdida
de generalidad....’. En la última sección vimos ejemplos en los cuales A y B
1.4. MODIFICACIÓN DEL PROBLEMA 27

eran problemas equivalentes, esto es, la solución de cualquiera de ellos implica


la solución del problema modificado (o auxiliar), el problema B, implica la
solución de A, pero no necesariamente viceversa.

1.4.1 Dados los números positivos a, b, c, d, demuestre

a3 + b3 + c3 b3 + c3 + d3 c3 + d3 + a3 d3 + a3 + b3
+ + + ≥ a2 + b2 + c2 + d2 .
a+b+c b+c+d c+d+a d+a+b

Solución. Debido a la simetrı́a del problema, basta con demostrar que para
cualesquiera números positivos x, y y z

x3 + y 3 + z 3 x2 + y 2 + z 2
≥ .
x+y+z 3

Si este fuera el caso, el lado izquierdo de la desigualdad original es al menos

a2 + b2 + c2 b2 + c2 + d2 c2 + d2 + a2 d2 + a2 + b2
+ + + ≥ a2 + b2 + c2 + d2
3 3 3 3

Ahora, para probar esta última desigualdad, no hay pérdida de generalidad


al suponer que x + y + z = 1. Pero si no, simplemente dividamos cada lado de
la desigualdad por (x+y+z)2 , y hagamos X = x/(x+y+z), Y = y/(x+y+z)
y Z = z/(x + y + z).

De esta manera , el problema original se reduce al siguiente problema modificado:

Dados números positivos X, Y, Z tales que X + Y + Z = 1, probar que

X2 + Y 2 + Z2
X3 + Y 3 + Z3 ≥ .
3
28 CAPÍTULO 1. HEURÍSTICA

1.4.2 Sea C cualquier punto en el segmento AB entre A y B, y dibújense


semicı́rculos sobre el mismo lado de AB con AB, AC y CB como diámetro
(figura 1.13). Sea D un punto en el semicı́rculo de diámetro AB tal que
CD es perpendicular a AB, y sean E y F puntos sobre los semicı́rculos de
diámetros AC y CB, respectivamente, de forma que EF sea un segmento de
su tangente común. Probar que ECF D es un rectángulo.

Solución. Nótese que basta con mostrar qu A, E, y D son colineales (el


mismo argumento mostrarı́a que B, F y D son colineales). Si este fuera el
caso, AEC = 900 (E está en el cı́rculo AEC), ADB = 900 , y el resultado es
verdadero. Se hace evidente, sin embargo, que con poca introspección, hay
muchas formas de encaminarse mal con esta aproximación; es difı́cil evitar
dar por cierta la conclusión.
D b

Fb

b
E

A b

C
b b

B
Figura 1.13

Una forma de ganar introspección en las relaciones entre los parámetros de


un problema es notar el efecto cuando a uno de ellos se le hace variar (mod-
ificación del problema). En este problema, se hace variar a D a lo largo
de la circunferencia. Sean G y H (figura 1.14) las intersecciones de los seg-
mentos AD y BD con los cı́rculos de diámetro AC y CB (y centros O y
O ′) respectivamente. Entonces AGC = ADB = CHB = 900, de tal forma
que GDHC es un rectángulo. Además, OGC = OCG (∆OGC es isósceles),
y CGH = GCD porque GH y CD son diagonales de un rectángulo. Ası́,
OGH = OCD. Ahora bien, mientras D se mueve para hacer CD perpendic-
ular AB, OGH se moverá también 900 , de forma que GH sea tangente al
cı́rculo O, y G conicida con E. Un argumento similar muestra que GH
es tangente al cı́rculo O ′, entonces H = F . Esto completa la demostración.
(Nótese que la frease ün argumento similar”, otra técnica de simplificación,
tiene el mismo efecto cuando es colocada después de un argumento que el
1.4. MODIFICACIÓN DEL PROBLEMA 29

que tiene ”basta mostrar queçuando está colocada antes del argumento.)
Db

Gb

H
b

A b b

O C
b

O′
b b

B
Figura 1.14

Nótese que se ha resuelto el problema resolviendo un problema más general.


Esta es una técnica común de resolución de problemas; se verán más ejemplos
en la sección 1.12.

1.4.3 Demuestre que no existen enteros positivos x, y, z tales que

x2 + y 2 + z 2 = 2xyz

Solución. Suponga que x, y, z son enteros positivos tales que x2 + y 2 + z 2 =


2xyz. Ya que x2 + y 2 + z 2 es par (= 2xyz), dos entre x, y, z son pares y el otro
es par , o los tres son pares. Suponga que x, y, z son pares. Entonces existen
enteros positivos x1 , y1 , z1 tales que x = 2x1 , y = 2y1 , z = 2z1 . Del hecho que
(2x1 )2 + (2y1 )2 + (2z1 )2 = 2(2x1 )2 (2y1 )2 (2z1 )2 se sigue que x1 , y1 , z1 satisfacen
que x1 2 + y1 2 + z1 2 = 22 x1 y1 z1 . De nuevo, de esta ecuación, si x1 , y1, z1 son
pares un argumento similar muestra que existen enteros positivos x2 , y2 , z2
tales que x2 2 + y2 2 + z2 2 = 23 x2 y2 z2 .

Continúese de esta forma. Eventualmente se llegará a una ecuación de la


forma a2 + b2 + c2 = 2n abc donde no todos a, b, c son pares (y de aquı́ que
dos entre a, b, c sean pares y uno impar).

Ası́, hemos sido llevados a considerar el siguiente problema modificado: De-


muestre que no existen enteros positivos x, y, z y n, con x, y pares, tales que

x2 + y 2 + z 2 = 2n xyz
30 CAPÍTULO 1. HEURÍSTICA

(Este es el problema 1.9.3)

R∞ 2
1.4.4 Evalúe 0
e−x dx.

Solución. Las técnicas usuales de integración estudiadas en el primer año


de cálculo no funcionarán para esta integral. Para evaluar la integral trans-
formaremos la integral simple en una integral doble.
R∞ 2
Sea I = 0 e−x dx. Entoces

hR i hR i
∞ −x2 ∞ −y 2
I2 = 0
e dx 0
e dy
R ∞ hR ∞ −x2 i −y2
= 0 e dx e dy
R ∞ R0∞ −x2 −y2
= R0 R0 e e dxdy
∞ ∞ 2 2
. = 0 0 e−(x +y ) dxdy

Ahora cámbiese a una integral equivalente utilizando coordenadas polares.


Entonces tenemos

R π R∞ 2
I 2 = 02 0 e−r rdrdθ
Rπ 2
= 02 − 12 e−r |θ0 dθ

= 21 02 dθ
π
=
4

π
Se sigue que I = 2

Un problema modificado (auxiliar) puede surgir de muchas maneras. Puede


aparecer mediante un cambio de notación (como en 1.4.4; ver sección 1.5) o
por simetrı́a (como en 1.4.1; ver sección 1.6). frecuentemente es el resultado
de ’trabajar hacia atrás’ (ver sección 1.8) o de argumentar por contradicción
(como 1.4.3; ver sección 1.9). No es extraño considerar un problema más
general en el contexto (como 1.4.2; ver sección 1.12). De esta manera ve-
mos que la modificación de un problema es una heurı́stica muy general. Por
esta razón, haremos a un lado más ejemplos y problemas, dejándolos más
apropiadamente para las secciones más especializadas que siguen.
1.5. ELECCIÓN DE UNA NOTACIÓN APROPIADA 31

1.5. Elección de una Notación Apropiada

Uno de los primeros pasos al trabajar sobre un problema de matemáticas


es traducir el problema a términos simbólicos. Al empezar, todos los con-
ceptos claves deberian ser identificados y etiquetados; las redundancias en la
notación pueden ser eliminadas al ir descubriendo las relaciones.

1.5.1 Una mañana comenzó a nevar de manera fuerte y constante. Una


removedora de nieve comenzó a las 8:00am. A las 9:00am se habı́a alejado 2
millas. A las 10:00am se habı́a alejado 3 millas. Suponiendo que la máquina
remueve un volumen constante de nieve por hora, determine la hora en que
comenzó a nevar.

Solución. Es difı́cil creer que hay suficiente información en el problema para


resolver la pregunta. Sin embargo, si hay una solución, debemos proceder sis-
temáticamente identificando primero aquellas cantidaddes que son descono-
cidas. Introducimos la siguiente notación: sea t el tiempo que ha transcurrido
desde que comenzó a nevar, y sea T el tiempo en la que la removedora sale
(medido desde t = 0) . Sea x(t) la distancia que ha avanzado la removedora
al tiempo t (sólo estamos interesados en x(t) para t ≥ T ). Finalmente, sea
h(t) la profundidad de la nieve en el tiempo t.

Estamos ahora listos para traducir el problema a términos simbólicos. El


hecho de que la nieve caiga a razón constante significa que la profundidad
está incrementándose constantemente; esto es,
dh
= c, c constante.
dt

Integrando cada lado se obtiene


h(t) = ct + d, c, d constantes.

Ya que h(0) = 0, obtenemos d = 0. Ası́ h(t) = ct.

El hecho de que la removedora quite la nieve a una razón constante significa


que la velocidad de la removedora es inversamente proporcional a la profundi-
dad en cualquier tiempo t (por ejemplo, el doble de profundidad corresponde
a la mitad de la velocidad).
32 CAPÍTULO 1. HEURÍSTICA

Simbólicamente, para t ≥ T ,

dx k
dt
= h(t) , k constante
k K
= ct
= t, K = kc constante.

Integrando ambos lados se obtiene

x(t) = Klogt + C, C constante.

Tenemos tres condiciones: x = 0 cuando t = T , x = 2, cuando t = T + 1,


y x = 3, cuando t = T + 2. Con estas dos condiciones podemos evaluar las
constantes K y C, y con la tercera, podemos resolver para T . Resulta que
(los detalles no son de intéres aquı́)


5−1
T = ≈ 0,618 hrs. ≈ 37 minutos, 5 segundos.
2
Ası́, comenzó a llover a las 7:22:55 AM.

1.5.2 (a) Si n es un entero positivo tal que 2n+1 es un cuadrado perfecto,


muestre que n + 1 es la suma de dos cuadrados perfectos sucesivos.

(b) Si 3n + 1 es un cuadrado perfecto, muestre que n + 1 es la suma de tres


cuadrados perfectos

Solución. Introduciendo la notación adecuada esto se reduce a un problema


sencillo de álgebra. Para la parte (a), suponga que 2n + 1 = s2 , s es un
entero. Ya que s2 es un número impar, también lo es s. Sea t un entero tal
que s = 2t + 1. Entonces 2n + 1 = (2n + 1)2 , y resolviendo para n tenemos
que
(2t + 1)2 − 1 4t2 + 4t
n= = = 2t2 + 2t.
2 3
Consecuentemente,

n + 1 = 2t2 + 2t + 1 = t2 + (t + 1)2
1.5. ELECCIÓN DE UNA NOTACIÓN APROPIADA 33

(b) Suponga que 3n+1 = s2 , s es un entero. Evidentemente , s no es múltiplo


de 3, ası́ que s = 3t ± 1 para algún entero t. Entonces 3n + 1 = (3t ± 1)2 , y
ası́

(3t ± 1)2 − 1 9t2 ± 6t


n= = = 3t2 ± 2t.
2 3

De esta manera,

n + 1 = 3t2 ± 2t + 1 = 2t2 + (t ± 1)2 = t2 + t2 + (t ± 1)2

1.5.3 En el triángulo ABC, AB = AC, D es el punto medio de BC, E es


el pie de la perpendicular dibujada desde D a AC, y F es el punto medio de
DE (figura 1.15). Demuestre que AF es perpendicular a BE.

Solución. Podemos transformar el problema a términos algebráicos aso-


ciando coordenadas a los puntos relevantes y mostrando que las pendientes
mBE y mAF son recı́procos negativos.

Una forma de proceder es tomar el triángulo como aparece en la figura 1.15:


tomar D como el origen (0, 0), A = (0, a), B = (b,0). Esta es una forma nat-
ural de señalar la figura porque aprovecha la simetrı́a bilateral del triángu-
lo isóseles (ver los ejemplos en la sección 1.6). Sin embargo; en este caso
en particular, esta notación lleva a algunas complicaciones menores cuando
buscamos las coordenadas de E y F.
34 CAPÍTULO 1. HEURÍSTICA

b
A

b
E
F b

C
b

B
b

D
b

Figura 1.15
B
3
b

2
b

D
1
F
b

b b b

A 1 2 3 E 4C
Figura 1.16
−1
Una mejor asignación de coordenadas es tomar A = (0, 0), B = (4a, ab),
C = (4c, 0), como en la figura 1.16. Entonces a2 + b2 = c2 , D = (2a + 2c + 2b),
E = (2a + 2c, 0), y F = (2a + 2c + b). (casi no hay cálculos aquı́; todos los
puntos relevantes tienen coordenadas asignadas). Se sigue que

  
b 4b b2
mBE mAF = = 2 = −1,
2 (a + c) 4a − (2a + c) a − c2
y la prueba queda concluida.

1.5.4 Sea −1 < a0 < 1 y defı́nase recursivamente

(1 + an−1 )1/2
an = , n > 0.
2
Sea An = 4n (1 − an ) . Qué pasa con An cuando n tiende a infinito?
1.5. ELECCIÓN DE UNA NOTACIÓN APROPIADA 35

Solución. Intentar expresar directamente an en términos de a0 lleva a com-


plicadas expresiones sin esperanza que contiene sucesiones anidadas de radi-
cales, y no hay forma de condensarlas en una forma cerrada.

La introspección clave requerida es observar que hay un único ángulo θ,


0 < θ < π, tal que a0 = cos θ. Para este θ,

  21  
1 + cos θ θ
a1 = = cos .
2 2

Similarmente,
  12    
1 + cos (θ/2) θ θ
a2 = = cos , ..., an = cos .
2 4 2n

Ahora podemos calcular

 
n θ
An = 4 1 − cos n
2
θ
 
n
4 1 − cos 2n 1 + cos 2θn
= 
1 + cos 2θn

4n sin2 2θn
= 
1 + cos 2θn
!  !2
θ2 sin 2θn
=  .
1 + cos 2θn θ
2n

Conforme n crece, θ2 /(1 + cos(θ/2n )) tiende a θ2 /2, y (sen(θ/2n ))/(θ/2n ) se


aproxima a 1 (recuerde que senx/x → 1 conforme x → 0), y ası́ , An converge
a θ2 /2 conforme n tiende a infinito.

Problemas
36 CAPÍTULO 1. HEURÍSTICA

1.5.5 Escriba una ecuación para representar las siguientes afirmaciones:

(a) En el restaurante Mindy, por cada cuatro personas que ordenaron pastel
de queso, hubo cinco que pidieron pay de manzana.

(b) Hay seis veces la cantidad de profesores en estudiantes en este colegio.

1.5.6 Unos cables son extendidos desde la punta de cada uno de los polos a
la base del otro. Cuál es la altura medida desde el piso al punto en que estos
alambres se cruzan?

1.5.7 Una pieza de papel de 8 pulgadas de ancho es doblada como en la


figura 1.17 de forma que una esquina es puesta en el lado opuesto. Exprese
la longitud del doblez L en términos del ángulo θ solamente.

1.5.8 Sean P1 , P2 , ....., P12 los vértices sucesivos de un dodecaedro regular


(12 lados). Concurren las diagonales P1 P9 , P2 P11 , P4 P12 .?

8
Figura 1.17
1.5. ELECCIÓN DE UNA NOTACIÓN APROPIADA 37

1.5.9 Utilice álgebra para respaldar sus respuestas a cada uno de los sigu-
ientes.

Un carro viaja desde A hasta B a razón de 40 millas por hora y luego regresa
de B a A a razón de 60 millas por hora? 50 millas por hora es apróximada-
mente la velocidad promedio par el viaje redondo?

Le dan a usted una taza de café y una de crema que contienen la misma
cantidad de lı́quido. Una cucharada de la mezcla es extraı́da de la taza y se
devuelve a la de crema. Hay más o menos crema ahora en la taza de café que
café en la taza de crema? ( Este problema tiene una elegante solución no
algebráica basda en la observación de que el café en la taza de crema ha
desplazado una cantidad igual de crema que debe estar ahora en la taza de
café.)

Imagine que la tierra es una esfera suave y que se le enreda una cuerda
alrededor del Ecuador. Suponga ahora que la cuerda es alargada 6 pies y que
la nueva longitud es empujada al parejo para formar un nuevo cı́rculo justo
encima del Ecuador. Es la distancia entre la cuerda y la superficie de la tierra
más o menos de 1 pulgada?

1.5.10 Un teorema bien conocido afirma que un primo p > 2 puede ser es-
crito como la suma de dos cuadrados perfectos (p = m2 + n2 , con m y n enteros)
si y sólo si p es mayor por 1 que un múltiplo de 4. Dando este resultado por
cierto, demuestre que:

Todo primo mayor por 1 que un múltiplo de 8 puede ser escrito en la forma
x2 + 16y 2, x y y enteros.

Todo primo mayor por 5 que un múltiplo de 8 puede ser escrito en la forma
(2x + y)2 + 4y 2, x y y enteros.

Ejemplos adicionales

1.1.10, 2.5.10, 3,2,15, 3.3.11, 3.3.28, 3.4.2, .3.4.4, 4.1.5, 6.4.2, 7.2.4, 8.1.15,
8.2.3, 8.2.17. También, ver las Secciones 2.5 (Relaciones de Recurrencia), 3.2
38 CAPÍTULO 1. HEURÍSTICA

(Aritmética Modular), 3.4 (Notación Posicional), 8.3 (Geometrı́a Vectorial),


8.4 (Geometrı́a en los Números Complejos).

1.6. Explotación de la Simetrı́a

La presencia de simetrı́a de un problema provee usualmente de medios para


reducir la cantidad de trabajo para llegar a una solución. Por ejemplo, con-
siderese el producto (a + b + c) (a2 + b2 + c2 − ab − ac − bc) . Ya que cada
factor es simétrico en a, b, c (la expresión no cambia cuando cualquier par de
sus variables son intercambiadas), lo mismo sucederá con el producto. Como
resultado de ello, si a3 aparece en el producto, también aparecerán b3 y c3 .
Similarmente, si a2 b aparece en el producto, también lo harán a2 c, b2 a, b2 c, c2 b,
y cada uno aparecerá con el mismo coeficiente, etc. De esta manera, una re-
visada rapida muestra que el producto tendrá la forma

 
A a3 + b3 + c3 + B a2 b + a2 c + b2 c + c2 a + c2 b + C (abc) .

Es fácil checar que A = 1, B = 0, y C = −3.

1.6.1 Se construyen triángulos equilateros ABK, BCL, CDM, DAN den-


tro del cuadrado ABCD. Demuestre que los puntos medios de los cuatro
segmentos KL, LM, MN, NK y los puntos medios de los ocho segmentos
AK, BK, BL, CL, CM, DM, DN, AN son los 12 vértices de un dodecaedro
regular.
1.6. EXPLOTACIÓN DE LA SIMETRÍA 39

Db b
C

b
K

b b b

a b b

b b

L b b
O b
N
b b

b b

b b

A b b

B
Figura 1.18

Solución. Los doce vértices están indicados en la Figura 1.18 con puntos
gruesos; dos de estos vértices están señalados como a y b según se muestra.

Utilizando la simetrı́a de la figura, basta mostrar que ∠bOK = 15◦ , ∠aOb =


30◦ , y |aO| = |bO| .

Notese que AN es parte del bisector perpendicular de BK, y entonces |KN| =


|NB| . Utilizando simetrı́a que sigue que ∠CBN = 15◦ . Considérese ahora
el triángulo equilátero DBN; nótese que Ob une los puntos medios de DB
y DN, ası́ qu Ob es la paralela a BN y de la mitad de longitud. De esta
manera |Ob| = s/2 y ∠bOK = 15◦ . Desde aquı́ es fácil verificar que
b

∠aOb = ∠DOK − ∠bOK = 45◦ − 15◦ = 30◦ , y |Oa| = |KN| /2 = s/2.

La presencia de simetrı́a en un problema también provee de visión clara


que nos permite ver y descubir relaciones que pueden ser más difı́ciles de
encontrar por otros medios. Por ejemplo, considerando sólo la simetrı́a se
ocurre que el valor máximo de xy, bajo la restricción x + y = 1, x > 0,
y > 0, deberı́a ocurrir cuando x = y = 1/2 (x y y están simétricamente
40 CAPÍTULO 1. HEURÍSTICA

relacionadas). Este es un ejemplo del principio de razón insuf iciente, que


puede resumirse de la siguiente manera: ”Donde no hay suficiente razón para
distinguir, no puede haber distinción.” Ası́, no hay razón para esperar que el
valor máximo ocurra cuando x tiene el valor distinto de 1/2, esto es, cercano
a 0 o a 1. Para verificar esto, sea x = 1/2 + e. Entonces y = 1/2 − e, y,
xy = (1/2 + e) (1/2 − e) = 1/4 − e2 . De esta forma es claro que el máximo
ocurre cuando e = 0; esto es, x = y = 1/2.

El siguiente problema ofrece varios Ejemplos adicionales de este principio.

1.6.2 .

(a) De todos los rectángulos que pueden inscribirse en un cı́rculo dado, cuál
tiene mayor área?

b b

x
b b

Figura 1.19

(b) Maximizar senA + senB + senC, donde A, B, C son las medidas de los
tres ángulos de un triángulo.

(c) De todos los triángulos con un perı́metro dado, cuál tiene mayor área?

(d) De todos los paralelepı́pedos de volumen 1, cuál tiene la menor área de


superficie?

(e) De todos los n − ágonos que pueden ser inscritos en un cı́rculo dado, cuál
tiene mayor área?
1.6. EXPLOTACIÓN DE LA SIMETRÍA 41

Solución. .(a) El principio de razón insuficiente nos lleva a sospechar que el


rectángulo con mayor área que puede inscribirse en un cı́rculo es un cuadrado
(Figura 1.19). Para verificar esto, sean x y y el largo y el ancho del rectángu-
lo, y supóngase sin pérdida de generalidad que las unidades han sido escogidas
de manera que el diámetro del cı́rculo es la unidad. Queremos maximizar xy
sujetos a que x2 + y 2 = 1. Esto es equivalente a maximizar x2 y 2 bajo la
restricción x2 + y 2 = 1. Pero este es el mismo problema que se consideró pre-
viamente a este ejemplo; el valor máximo ocurre cuando x2 + y 2 = 1/2, esto
es, cuando el rectángulo es un cuadrado.

(b) Nótese que la suma, senA + senB + senC, es siempre positiva (ya que
cada uno de los términos es positivo), y que puede ser hecha arbitrariamente
pequeña (en magnitud) haciendo A arbitrariamente cercano a 180◦ . No hay
razón para esperar que el máximo ocurra en otro punto que no sea A = B =
C = 60◦ (un triángulo equilátero). Una demostración de esto se sigue de la
discusión en 2.4.1.

De manera similar, sospechamos que las respuestas a (c),(d), y (e) son un


triángulo equilátero, un cubo, y un n − ágono regular.

1.6.3 Evaluar

Z π
2 dx

2
0 1 + (tan x)
42 CAPÍTULO 1. HEURÍSTICA

1
1/2

pi/41 pi/2 2

Figura 1.20

Solución. . He aquı́ un problema que no puede resolverse utilizando las


técnicas usuales de integración; es decir, el integrando no tiene una an-
tiderivada. Sin embargo, el problema puede ser manejado si notamos que
el integrando (Figura 1.20) es simétrico alrededor del punto (π/4, 1/2) .
 √ 
Para mostrar que esto es ası́ (no es obvio), sea f (x) = 1/ 1 + (tan x) 2 .
√ + f (π/2 − x) = 1 para toda x, 0 ≤ x ≤ π/2. Ası́,
Basta mostrar que f (x)
calculamos, para r = 2,

1 1
f (π/2 − x) + f (x) = +
1 + tan (π/2 − x) 1 + tanr (x)
1 1
= +
1 + cot (x) 1 + tanr (x)
r

tanr (x) 1
= +
1 + tan (x) 1 + tanr (x)
r

= 1.

Se sigue de la simetrı́a recientemente demostrada que el área bajo la curva


en [0, π/2] es la mitad de área en el rectángulo (ver Figura 1.20); esto es, la
1.6. EXPLOTACIÓN DE LA SIMETRÍA 43

integral es (π/2) /2 = π/4.

Otra manera de sacar ventaja de la simetrı́a es la elección de notación. He


aquı́ un par de ilustraciones.

1.6.4 Sea P un punto en la gráfica de y = f (x), donde f es un polinomio


de grado tres; supóngase que la tangente en P intersecta de nuevo a la curva
en Q; y sea A el área de la región limitada por la curva y el segmento P Q.
Sea B el área de la región definida si comenzamos por Q en vez de P. Cuál
es la relación entre A y B?.

P
b

−2 −1 x0 1Q
b

−1

−2

−3

R
b

−4
Figura 1.21

Solución. . Sabemos que un polinomio cúbico es simetrı́co respecto a su


punto de inflexión. Ya que las áreas de interés no son afectadas por la elección
del sistema de coordenadas, fijaremos el origen en el punto de inflexión. Ası́,
podemos suponer que la función cúbica es

f (x) = ax3 + bx, a 6= 0

(ver Figura 1.21).

Supóngase que x0 es la abcisa de P. Resulta que la abcisa de Q es −2x0 .


(No nos detendremos en los detalles de estos cálculos tan directos. Existe,
44 CAPÍTULO 1. HEURÍSTICA

ciertamente, una manera elegante de llegar a este hecho, pero utiliza ideas
que no se verán en esta sección).

Integrando directamente se muestra que el área A es igual a Kx40 , donde K


es independiente de x0 . (De nuevo no nos detendremos en los detalles del
cálculo.)

Podemos aplicar ahora nuestras conclusiones previas al punto Q. La tan-


gente en Q intersectará a la curva en R, la abcisa del cual es evidentemente
−2 (−2x0 ) = 4x0 , y el área B es igual a K (−2x0 )4 = 16Kx40 = 16A.

1.6.5 Determine todos los valores de x que satisfacen

tan x = tan (x + 10◦ ) tan (x + 20◦ ) tan (x + 30◦) .

Solución. . Introduciremos simetrı́a haciendo un simple cambio de variable.


Ası́, sea y = x + 15◦ . La ecuación es entonces

tan (y − 15◦ ) = tan (y − 5◦ ) tan (y + 5◦ ) tan (y + 15◦ ) ,


que equivale a

sen (y − 15◦ ) cos (y + 15◦ ) sen (y − 5◦ ) sen (y + 5◦ )


=
cos (y − 15◦ ) sen (y + 15◦ ) cos (y − 5◦ ) cos (y + 5◦ )
usando las identidades

1
sen (A) cos (B) = [sen (A − B) + sen (A + B)] ,
2
1
sen (A) sen (B) = [cos (A − B) − cos (A + B)] ,
2
1
cos (A) cos (B) = [cos (A − B) + cos (A + B)] ,
2
1.6. EXPLOTACIÓN DE LA SIMETRÍA 45

obtenemos
sen (−30◦ ) + sen2y cos (−10◦ ) − cos 2y
= ,
sen (30◦ ) + sen2y cos (−10◦ ) + cos 2y
o equivalentemente,
2sen2y − 1 cos 10◦ − cos 2y
= .
2sen2y + 1 cos 10◦ + cos 2y
Esto se simplifica en
sen (4y) = cos 10◦ ,
4y = 80◦ + 360◦ k, 100◦ + 360◦ k, k = ±0, ±1, ±2, ... ,
lo cual implica que
x = 5◦ + 90◦ k, 10◦ + 90◦ k, k = ±0, ±1, ±2, ... .

Problemas

1.6.6 .

(a) Explote la simetrı́a para expandir el producto


 
x2 y + y 2 z + z 2 x xy 2 + yz 2 + zx2 .

(b) Si x + y + z = 0, demuestre que


 2  5 
x + y2 + z2 x + y5 + z5 x7 + y 7 + z 7
=
2 5 7

(substituya z = −x − y y aplique el teorema del binomio.)


46 CAPÍTULO 1. HEURÍSTICA

1.6.7 Las caras de cada una de las quince monedas, acomodadas como se
muestra en la Figura 1.22, son pintadas de blanco o de negro. Demuestre
que exiten tres monedas del mismo color cuyos centros son los vértices de un
triángulo equilátero. (Hay muchas maneras de explotar la simetrı́a y de crear
argumentos del tipo ”sin pérdida de generalidad”.)

Figura 1.22

1.6.8 Haga uso del principio de razón insuficiente para minimizar x21 +
x22 + ... + x2n , bajo la condición de que 0 < xi < 1, y x1 + x2 + ... + xn = 1.
Demuestre su conjetura (Para la demostración, tome xi = n1 + ei .)

1.6.9 Un punto P está localizado en el interior de un triángulo equilátero


ABC. Desde P se trazan perpendiculares que intersectan a cada lado en los
puntos D, E, y F , respectivamente. Dónde deberı́a estar P para que P D +
P E + P F sea un máximo? Dónde deberı́a estar P para que P D + P E + P F
sea un mı́nimo? Justifique sus respuestas. (Sugerencia: Es de gran ayuda
reflejar la figura respecto a uno de los lados. Qué pasa con P D + P E + P F
conforme P se mueve paralelamente a la lı́nea de reflexión?)

1.6.10 En la Figura 1.23, ABCD es un cuadrado, ∠EDC = ∠ECD =


15◦ . Demuestre que el triángulo AEB es equilátero. (La clave para este bello
problema es crear simetrı́a central. Especı́ficamente, agregar ángulos idénticos
de 15◦ sobre los lados AB, BC, y AD (como en el lado CD) y crear un
diagrama muy parecido al construido en 1.6.1.)

1.6.11 El producto de cuatro términos consecutivos de una serie aritmética


de enteros más la cuarta potencia de la diferencia común es siempre un
1.7. DIVISIÓN EN CASOS 47

cuadrado perfecto. Verifique esta identidad incorporando simetrı́a en la no-


tación.
B b
C b

b E

b b

A D
Figura 1.23

Ejemplos adicionales

1.4.1, 8.1.4, 8.1.5, 8.1.8, 8.2.3.

1.7. División en Casos

Sucede con frecuencia que un problema pueda ser dividido en un número


pequeño de subproblemas, cada uno de los cuales puede ser manejado por
separado a la manera de caso por caso. Esto es cierto especialmente cuando el
problema contiene un cuantificador universal (’para todo x ...’). Por ejemplo,
la demostración de una proposición de la forma ’para todos los enteros ...’
puede hacerse argumentando para los pares y los impares por separado. De
forma similar, un teorema sobre triángulos puede probarse dividiéndolo en
tres casos dependiendo de si el triángulo es agudo, recto u obtuso. Ocasional-
mente, los subproblemas pueden ser arreglados jerárquicamente en submetas,
de forma que los primeros casos, una vez establecidos, pueden ser utilizados
para probar las siguientes etapas. Tal procedimiento es llamado escalada.
48 CAPÍTULO 1. HEURÍSTICA

En las primeras etapas del análisis, es útil pensar en cómo subdividir un


problema en un número pequeño de subproblemas (con suerte) más sencillos.
La heurı́stica de esta sección se da muy con frecuencia de la siguiente manera:
’Si no puede resolver el problema, encuentre un problema relacionado más
sencillo y resuélvalo.’

1.7.1 Demuestre que un ángulo inscrito en un cı́rculo es igual a la mitad


del ángulo central que subtiende el mismo arco.
A A A B
b b

b
B
O
O
b b b

O
b
B P
b

P
b

P
b

Figura 1.24

Solución. Tenemos un cı́rculo, con centro O digamos, y un ángulo inscrito


AP B; se muestran algunos ejemplos en la figura 1.24. Tenemos que demostrar
que para todos los casos ∠AP B = 12 ∠AOB. Las tres figuras precedentes
representan situaciones esenciales diferentes. Especı́ficamente, el centro del
cı́rculo, O, está en el interior de ∠AP B (diagrama 2), o fuera de ∠AP B
(diagrama 3), o en uno de los rayos de ∠AP B (diagrama 1). Probaremos el
teorema considerando cada uno de estos casos por separado.

Caso1. Supongamos que el centro O está en P A. Entonces ∠AOB = ∠OP B+


∠OBP (un ángulo exterior es igual a la suma de los ángulos interiores no
adyasentes) = 2∠DP B (∆OP B es isósceles)= 2∠AP B. El resultado es in-
mediato.

Caso2. Si O está en el interior del ∠AP B (diagrama 2), extendemos la lı́nea


P O hasta cortar el cı́rculo en D. Acabamos de demostrar que 2∠AP B =
∠AOD y 2∠DP B = ∠DOB. La suma de estas ecuaciones dará el resultado
deseado.

Caso 3. Si O es exterior ∠AP B (diagrama 3), extendemos P O hasta cortar el


cı́rculo en Ḋ. Entonces, utilizando el caso 1, 2∠DP B = ∠DOB y 2∠DP A =
1.7. DIVISIÓN EN CASOS 49

∠DOA. Restando la segunda ecuación de la primera se obtiene el resultado


deseado. Esto completa la demostración.

1.7.2 Una función real de variable racional f satisface

f (x + y) = f (x) + f (y)

para todos x y y racionales. Demuestre que f (x) = f (1) · x para todo x


racional.

Solución. Procedemos por pasos. Primero probaremos el resultado para los


enteros positivos, luego los negativos, luego los recı́procos de los enteros, y
finalmente para todos los números racionales.

Caso 1 (Enteros positivos). El resultado se cumple cuando x = 1. Para x = 2,


tenemos f (2) = f (1 + 1) = f (1) + f (1) = 2f (1) . Para

x = 3, f (3) = f (2 + 1) = f (2)+f (1) = 2f (1)+f (1) = 3f (1) . Es claro que


este proceso puede ser continuado, y que para cualquier entero positivo n,
f (n) = nf (1) . (Puede darse una demostración formal basada en el principio
de inducción matemática, ver Capı́tulo2).

Caso 2 (Enteros no positivos). Primero, f (0) = f (0 + 0) = f (0) + f (0) .


Restando f (0) de ambos lados se obtiene 0 = f (0) ; esto es f (0) = 0 · f (1) .
Ahora, 0 = f (0) = f (1 + (−1)) = f (1) + f (−1) . De aquı́ que f (−1) =
f (1) . De manera similar, para cualquier entero positivo n, f (n) + f (−n) =
f (n + (−n)) = f (0) = 0, de forma que f (−n) = −nf (1) .

Caso 3(Recı́procos).
 Para x = 21 , procedemos como sigue: f (1)= f 21 + 21
= f 21 + f 21 = 2f 12 . dividiendo entre 2 se obtiene f 21 = f (1) /2.
Para x = 31 ,
    
f (1) = f 31 + 13 + 31 = f 31 + f 31 + f 31 = 3f 31 , o equivalentemente, 
1
f (1) /3. De forma similar, para  cualquier
 entero positivo
 n, f n
= f (1) /n.
Para x = −1n
, tenemos f n
1
+ f −1
n
= f 1
n
+ −1
n
= f (0) = 0, de forma
−1
que f n = −f (1) /n.
2
 1 1

Caso 4 (todos los racionales).
 2 Sea n un entero. Entonces f n
= f n
+ n
=
1 1 1 m
f n +f n = 2f n = n f (1) . De forma similar, si n es cualquier racional,
50 CAPÍTULO 1. HEURÍSTICA

com m un entero positivo y n un entero, entonces


m        
1 1 1 1 1 m
f =f + ... + =f + ... + f = mf = f (1) .
n n n n n n n
| {z } | {z }
m−veces m−veces

Esto establece el resultado, un buen ejemplo de escalada.

1.7.3 Demuestre que el área de un triángulo retı́cula es igual a I + 12 B − 1,


donde I y B representan respectivamente el número de puntos interiores y de
frontera de la retı́cula en el triángulo. (Un triángulo retı́cula es un triángulo
en el plano con puntos de una retı́cula como vértices.)
b
C C b
C b

B
b

b b b b b

A 1 B A 2 B A 3

b
C b
C

B b

B
b b

A 4 A 5
Figura 1.25

Solución. Este es un caso especial del teorema de Pick (ver 2.3.1.). Hay
varias pruebas ingeniosas, cada una de las cuales divide al conjunto de los
triángulos retı́cula en unos pocos de tipos especiales. Una forma de hacer esto
es ’circunscribir’ sobre el triángulo un rectángulo con bordes paralelos a los
ejes coordenados. Al menos un vértice del rectángulo debe coincidir con un
vértice del triángulo. Ahora es fácil ver que cada triángulo retı́cula puede ser
clasificado en una de las clases no equivalentes esbozadas en la Figura 1.25.

En la primera clase están aquellos triángulos rectos cuyos catetos son par-
alelos a los ejes coordenados. La segunda clase incluye triángulos de ángulos
1.7. DIVISIÓN EN CASOS 51

agudos uno de cuyos lados es paralelo a un eje coordenado. Tales triángulos


son la ‘suma’ de dos triángulos de la primera clase. En la tercera clase están
los triángulos obtusos con un lado paralelo a un eje coordenado. Ellos son
la ’resta’ de dos triángulos de la primera clase. Las clases cuarta y quin-
ta abarcan a aquellos triángulos que no tienen lados paralelos a los ejes de
coordenadas.

La demostración del resultado sigue un patrón de escalada. Para empezar,


consideremos el rectángulo ABCD del caso 1. Supongamos que los segmentos
AB y AD contienen a y b puntos de la retı́cula, respectivamente, sin contar
sus extremos. Entonces, con I y B los puntos interiores y de frontera de
ABCD,
1 1
I + B − 1 = ab + (2a + 2b + 4) − 1
2 2
= ab + a + b + 1
= (a + 1) (b + 1)
= Area ABCD

Ahora supongamos que AB, BC, y AC contienen a, b, y c puntos de la retı́cu-


la, respectivamente, sin contar sus extremos, y supongamos que ABC con-
tiene i puntos interiores. Entonces el rectángulo ABCD tiene 2i + c puntos
interiores, y tenemos, con I y B los puntos interiores y frontera de ABC,

1 1
I + B − 1 = i + (a + b + c + 3) − 1
2 2
1
= (2i + a + b + c + 1)
2 
1 1
= (2i + c) + (2a + 2b + 4) − 1
2 2
1
= Area ABCD
2
= Area ABC.
Los otros casos pueden ser manejados de forma similar; dejamos los detalles
al lector.

Problemas
52 CAPÍTULO 1. HEURÍSTICA

1.7.4 ( Desiguladad del triángulo ).

(a) Demuestre que para cualesquiera números reales x y y, |x + y| ≤ |x| +


|y| .

(b) Demuestre que para cualesquiera números reales x, y, y z, |x − y| ≤


|x − z| + |y − z| .

1.7.5 Encuentre todos los valores de x que satisfacen

3 2
< .
x−1 x+1

1.7.6 Sean S = {i (3, 8) + j (4, −1) + k (5, 4) / i, j, k son enteros}, y T =


{m (1, 5) + n (0, 7) / m, n son enteros} . Demuestre que S = T. (Nota: las
parejas ordenadas de enteros se suman componente a componente: (s, t) +
(s′ , t′ ) = (s + s′ , t + t′ ) , y n (s, t) = (ns, nt) .)

1.7.7 Una función real f, definida en los números racionales positivos, sat-
isface f (x + y) = f (x) f (y) para cualesquiera x y y racionales positivos.
Demuestre que f (x) = [f (1)]x para todo racional positivo x.

1.7.8 Determine F (x) si, para cualesquiera reales x y y, F (x) F (y) +


F (xy) = x + y.

Ejemplos adicionales

1.1.7,2.5.11c,2.5.12,2.5.13,2.6.3, 3.2.14, 3.2.15, 3.2.16, 3.2.17, 3.2.18, 3.4.1,


4.1.3, 4.1.4, 4.4.14, 4.4.29, 5.2.1, 5.3.14c, 6.5.4, 7.4.3, 7.6.2, 7.6.4, 7.6.10, 8.2.4.
Algunos ejemplos particulares que se reducen a estudiar casos muy especiales
son 3.3.8, 3.3.9, 3.3.21, 3.3.22, 3.3.26.
1.8. TRABAJO HACIA ATRÁS 53

1.8. Trabajo Hacia Atrás

Trabajar hacia atras significa dar cierta conclusión y entonces obtener de-
duciones de la conclusión hasta que llegamos al algo conocido o algo que
puede demostrarse fácilmente. Después de que llegamos a lo dado o a lo
conocido, entonces invertimos los pasos del argumento y procedemos hacia
la conclusión.

Este procedimiento es común en álgebra y trigonometrı́a de secundaria. Por


ejemplo, para encontrar todos los números reales que satisfacen 2x + 3 = 7,
argumentos como sigue. Supongamos que x satisface 2x + 3 = 7. Entonces,
restamos 3 de cada lado de la ecuación y dividimos cada lado entre 2, para
obtener x = 2. Ya que cada paso en esta deducción puede ser invertido,
concluimos que 2 ciertamente satisface 2x + 3 = 7 y es el único número que
lo hace.

Con frecuencia, en manipulaciones de rutina, tales como las del ejemplo pre-
vio, no se hace una reescritura explı́cita de los pasos. Sin embargo, es im-
portante estar atento a lo que
√ puede, √ y no puede, ser invertido. Por ejemp-
lo, considerese la ecuación x + 1 − x − 1 = 2. (Aquı́, como es usual, la
raı́z √
cuadrada es
√ la raı́z cuadrada positiva.) Escriba la ecuación en la for-
ma x + 1 = x√− 1 + 2, y eleve ambos lados al√cuadrado para obtener
x + 1 = x − 1 + 4 x − 1 + 4, o equivalentemente, x − 1 = −1/2. Eleve al
cuadrado una vez más para obtener x−1 = 1/4, o x = 5/4. Sin embargo, 5/4
no satisface la ecuación original. La razón de esto es que √
no todos los pasos
son reversibles. Ası́, en este ejemplo, procedemos desde x − 1 = −1/2 a
x − 1 = 1/4. Cuando
√ invertimos esto, sin embargo, el argumento va desde
x − 1 = 1/4 a x − 1 = 1/2.

1.8.1 Sea α un número real fijo, 0 < α < π, y sea


senθ + sen (θ + α)
F (θ) = , 0 ≤ θ ≤ −α.
cos θ − cos (θ + α)
Demuestre que F es constante. (Este problema surgió en 1.2.1.)

Solución. Supongamos que F es constante. Entonces F (θ) = F (0) para


todo θ, 0 ≤ θ ≤ −α. Esto es,
54 CAPÍTULO 1. HEURÍSTICA

senθ + sen (θ + α) senα


= , (1.1)
cos θ − cos (θ + α) 1 − cos α
[senθ + sen (θ + α)] [1 − cos α] = senα [cos θ − cos (θ + α)] , (1.2)
senθ + sen (θ + α) − senθ cos α − sen(θ + α) cos α
= senα cos θ − senα cos (θ + α) , (1.3)
senθ + sen (θ + α) − [senθ cos α + senα cos θ]
− [sen(θ + α) cos α − senα cos (θ + α)] = 0, (1.4)
senθ + sen (θ + α) − sen (θ + α) − sen (θ + α − α) = 0. (1.5)
La última ecuación es una identidad. Para la demostración, debemos invertir
los pasos. El único paso cuestionable es de ?? a ??: la prueba sólo es válida si
no dividimos por cero al ir de ?? a ??. Pero (1 − cos α) 6= 0 ya que 0 < α < π,
y cos θ − cos(θ + α) > 0 ya que 0 ≤ θ < θ + α ≤ π. La demostración puede
entonces llevarse a cabo; esto es, comenzando por la identidad conocida en
??, podemos argumentar (vı́a los pasos ??,??,??,??) que para todo θ, 0 ≤
θ ≤ π − α, F (θ) = senα/ (1 − cos α) = constante.

1.8.2 Si a, b, c denotan las longitudes de los lados de un triángulo, demuestre


que
3 (ab + bc + ca) ≤ (a + b + c)2 ≤ 4 (ab + bc + ca) .
Solución. Considerese la desigualdad del extremo izquierdo:

3 (ab + bc + ca) ≤ (a + b + c)2 ,


3 (ab + bc + ca) ≤ a2 + b2 + c2 + 2 (ab + bc + ca) ,
(ab + bc + ca) ≤ a2 + b2 + c2 ,
a2 + b2 + c2 − (ab + bc + ca) ≥ 0
2a2 + 2b2 + 2c2 − 2 (ab + bc + ca) ≥ 0
  
a2 − 2ab + b2 + b2 − 2bc + c2 + c2 − 2ca + a2 ≥ 0
(a − b)2 + (b − c)2 + (c − a)2 ≥ 0

Esta desigualdad es verdadera para todos los valores de a, b, c. Considerese


ahora la desigualdad de la derecha:
1.8. TRABAJO HACIA ATRÁS 55

(a + b + c)2 ≤ 4 (ab + bc + ca)


a2 + b2 + c2 + 2 (ab + bc + ca) ≤ 4 (ab + bc + ca)
a2 + b2 + c2 ≤ 2 (ab + bc + ca)
a2 + b2 + c2 ≤ a (b + c) + b (a + c) + c (b + a)

Esta desigualdad final es verdadera, ya que la suma de cualesquiera dos de los


lados de un triángulo es mayor que el lado restante. Ası́, a2 ≤ a (b + c) , b2 ≤
b (a + c) , y c2 ≤ c (b + a) .

Los pasos en cada uno de estos argumentos pueden ser invertidos, ası́ que la
prueba está completa.

1.8.3 Se tiene: AOB es el diámetro del cı́rculo O; BM es tangente al cı́rculo


en B; CF es tangente al cı́rculo en E e intersecta BM en C; la cuerda AE,
al ser prolongada, intersecta BM en D. Demuestre que BC = CD. (Ver la
Figura 1.26)
A b

a b

b
O
b
E
c d

b e
M
b b b b

B C D
Figura 1.26

Solución. Supongamos que BC = CD. Entonces CE = CD, ya que BC =


CE (tangentes al cı́rculo desde C en B y E son iguales). Ası́, ∠CED =
∠CDE (los ángulos base de un triángulo isósceles son iguales). Esto nos
lleva a considerar los ángulos de la manera en que están etiquetados en la
Figura 1.26.
56 CAPÍTULO 1. HEURÍSTICA

Ahora bien, ∠D es el complemento de ∠a ya que el ∆ABD es un triángulo


rectángulo, y el ∠e es complemento del ∠c ya que ∠BEA es un ángulo recto
(AOB es un diámetro). Ası́, ∠a = ∠c. Pero sabemos que ∠a = ∠c, ya que
ambos subtienden el mismo arco BE en el cı́rculo O.

La prueba puede ser completada invirtiendo los pasos. De esta manera (omiti-
mos las razones),partimos de que ∠a = ∠c, y por lo tanto, ∠e = ∠d.
Ası́ CD = CE, CE = BC, y por lo tanto BC = CD.

1.8.4 En un torneo con n jugadores P1 , P2 , ..., Pn , con n > 1, cada jugador


juega un juego con cada uno de los otros jugadores y las reglas son de tal
manera que no puede haber empates. Sean Wr y Lr el número de juegos
ganados y perdidos, respectivamente, por el jugador Pr . Demuestre que
n
X n
X
Wr2 = L2r .
r=1 r=1

Pn Pn
Solución. Supongamos que r=1 Wr2 = r=1 Wr2 . Entonces,

n
X 
Wr2 − L2r = 0,
r=1
n
X
(Wr − Lr ) (Wr + Lr ) = 0.
r=1

Pero Wr + Lr = n − 1 para cada r, ası́ que

n
X
(n − 1) (Wr − Lr ) = 0,
r=1
Xn
(Wr − Lr ) = 0,
r=1
n
X n
X
Wr = Lr .
r=1 r=1
1.8. TRABAJO HACIA ATRÁS 57

Esta última ecuación es verdadera, ya que el número total de juegos ganados


por n jugadores es igual al número total de juegos perdidos. La demostración
se sigue al invertir el argumento precedente.

Problemas

1.8.5 .

(a) Dados números positivos reales x y y, demuestre que


2 √ x+y
≤ xy ≤ .
1/x + 1/y 2

(b) Dados a y b, números reales positivos tales que a + b = 1, demuestre


que
2
a
≤ ax + by, x > 0, y > 0.
x
+ yb

1.8.6 .

(a) Si a, b, c son números reales positivos, y a < b + c, muestre que

a b c
< + .
1+a 1+b 1+c

(b) Si a, b, c son las longitudes de tres segmentos que pueden formar un


triángulo, muestre que lo mismo se vale para 1/ (a + c) , 1/ (b + c) , 1/ (a + b) .

1.8.7 Dos cı́rculos son tangentes externamente en A, y una tangente común


externa los toca en B y C. El segmento BA se prolonga hasta intersectar al
segundo cı́rculo en D. Demuestre que CD es un diámetro.
58 CAPÍTULO 1. HEURÍSTICA

1.8.8 Considerese el siguiente argumento. Suponga que θ satisface


cot θ + tan 3θ = 0.
Entonces, como
tan α + tan β
tan (α + β) = ,
1 + tan α tan β
se sigue que
tan θ + tan 2θ
cot θ + = 0
1 − tan θ tan 2θ
cot θ (1 − tan θ tan 2θ) + tan θ + tan 2θ = 0
cot θ − tan 2θ + tan θ + tan 2θ = 0
cot θ + tan θ = 0
1 + tan2 θ = 0
tan2 θ = −1.

Ya que la última ecuación no puede ser cierta, la ecuación original no tiene


solución (no necesitamos invertir ningún paso porque el paso final no lleva a
nada). Sin embargo, θ = 14 π si satisface cot θ + tan 3θ = 0. Qué está mal en
la argumentación?

1.8.9 Con las herramientas Euclidianas (regla y compás), inscriba un cuadra-


do en un triángulo dado de manera que un lado del cuadrado quede en un
lado dado del triángulo. (Sugerencia: Comience con el cuadrado y construya
un triángulo alrededor de él semejante al triángulo dado. Luego haga uso del
hecho de que figuras semejantes tienen partes proporcionales.)

Ejemplos adicionales

2.1.5, 7.1.1, 7.4.6. También ver la Sección 2.2 (Inducción) y la Sección 2.5
(Recurrencia).

1.9. Argumentación por Contradicción

Arguentar por contradicción significa suponer que la conclusión es falsa y


entonces obtener deducciones hasta que lleguemos a algo que sea contradic-
1.9. ARGUMENTACIÓN POR CONTRADICCIÓN 59

torio, ya sea con lo que está dado (el método indirecto) o a lo que se sabe que

es verdadero (reductio ad absurdum) . Ası́, por ejemplo, para probar que 2
es irracional, podemos suponer que es racional y proceder hasta obtener una
contradicción. El método es con frecuencia apropiado cuando la conclusión
es fácil de negar, cuando las hipótesis ofrecen poco material susceptible de
ser manipulado, o cuando se tienen pocas ideas sobre cómo proceder.

Como un ejemplo sencillo de este método de demostración, considere la sigu-


iente argumentación que muestra que la serie armónica diverge. Supóngase,
por el contrario, que converge, a r digamos. Entonces

1 1 1 1 1 1 1
r = 1+ + + + + + + + ...
2 3 4 5 6 7 8
1 1 1 1 1 1 1 1
> + + + + + + + + ...
2 2 4 4 6 6 8 8
1 1 1
= 1 + + + + ...
2 3 4
= r,

una contradicción. Nos vemos forzados a concluir que la serie diverge.

1.9.1 Dado que a, b, c son enteros impares, demuestre que la ecuación ax2 +
bx + c = 0 no puede tener raı́ces racionales.

Solución. Supongamos que p/q es una raı́z racional, donde (sin perdida de
generalidad) p y q son enteros pares. Primero estableceremos que ninguno
entre p y q puede ser par. Supongamos que p es par. De ax2 + bx + c = 0
tenemos que ap2 + bpq + cq 2 = 0. Ya que ap2 + bpq es par, cq 2 debe ser par,
pero esto es imposible, ya que c y q son ambos impares. Obtenemos una
contradicción similar si suponemos que q es par. Por lo tanto, ambos p y q
son impares y ap2 + bpq + cq 2 = 0. Pero esta última ecuación establece que
la suma de tres números impares es igual a cero, lo cual es imposible. Por lo
tanto, la ecuación no tiene raı́ces racionales.

Es ilustrativo considerar otra demostración de este resultado. Las raı́ces de


ax2 + bx + c = 0 son racionales si y sólo si b2 − 4ac es un cuadro perfecto.
Ası́, supongamos que b2 −4ac = (2n + 1)2 para algún entero n (por hipótesis,
60 CAPÍTULO 1. HEURÍSTICA

b2 − 4ac es impar, y por lo tanto, si es un cuadrado, debe ser el cuadrado de


un entero impar). Agrupando múltiplos de 4 tenemos que

b2 − 1 = 4[n (n + 1) + ac].

Ya que ni n ni n + 1 son pares, n (n + 1) + ac es impar. Ası́, el lado derecho


de la última ecuación es divisible por 4 pero no por 8. Sin embargo, el lado
izquierdo es divisible por 8, ya que b2 − 1 = (b − 1) (b + 1) y entre b − 1 y
b + 1 es divisible por 4, mientras que el otro es divisible por 2. Por lo tanto
la ecuación antes escrita no puede ser cierta, y tenemos una contradicción.
(En esta demostración, hemos llegado a una contradicción al examinar como
se relacionan dos números respecto a múltiplos de 8, más que respecto a
múltiplos de 2 como en la primera demostración.)

En las dos secciones siguientes se dan Ejemplos adicionales de demostración


por contradicción.

Problemas

1.9.2 En una fiesta con 2000 personas, en cada grupo de 4 hay al menos una
persona que conoce a las otras tres. Hay tres personas que no se conocen entre
sı́. Demuestre que las otras 1997 conocen a todo mundo en la fiesta. (Suponga
que ’conocer’ es una relación simétrica; esto es, si A conoce B entonces B
también conoce a A. Cuál es la respuesta si ’conocer’ no necesariamente es
una relación simétrica?)

1.9.3 Demuestre que no existen enteros positivos a, b, c y n tales que a2 +


b + c2 = 2n abc. (De 1.4.3, podemos suponer que a y b son impares y c es
2

par. Cómo son los lados de la ecuación respecto a 4?)


1.9. ARGUMENTACIÓN POR CONTRADICCIÓN 61

1.9.4 Cada par de comunidades de un condado están enlazadas por exac-


tamente un medio de transporte: camión, tren o avión. Los tres medios son
utilizados en el condado; ninguna de las comunidades cuenta con los tres
medios simultáneamente, y ninguna tercia de comunidades está enlazada por
parejas por el mismo medio. Cuatro comunidades pueden estar conectadas
de acuerdo a lo estipulado de la siguiente forma: camión, AB, BC, CD, DA;
tren, AC; avión, BD.

(a) Dé un argumento para mostrar que ninguna comunidad puede tener
un medio de transporte en especial que la conecte a tres comunidades
diferentes.

(b) Dé una demostración que muestre que cinco comunidades no pueden
ser conectadas de acuerdo a lo estipulado.

1.9.5 Sea S un conjunto de números racionales que es cerrado bajo la suma


y producto (esto es, si a y b son elementos de S, también lo son a + b y ab),
y que tiene la propiedad de que para cada número racional r se cumple una
y sólo una de las siguientes afirmaciones: r ∈ S, −r ∈ S, r = 0.

(a) Demuestre que 0 no es elemento de S.

(b) Demuestre que todos los enteros positivos son elementos de S.

(c) Demuestre que S es el conjunto de todos los números racionales posi-


tivos.

Ejemplos adicionales

1.5.10,1.6.7, 3.2.1, 3.2.6, 3.2.11, 3.2.13, 3.2.15, 3.2.17, 3.2.18, 3.3.4, 3.3.11,
3.3.15, 3.3.28, 3.4.2, 4.1.3, 4.4.6, 5.4.1. También ser Sección 1.10 (Paridad) y
la Sección 1.11 (Casos Extremos).
62 CAPÍTULO 1. HEURÍSTICA

1.10. Búsqueda de Paridad

La sencilla idea de paridad, ser par o ser impar, es un concepto poderoso en la


resolución de problemas con una gran cantidad de aplicaciones. Revisaremos
algunos ejemplos en está sección.

1.10.1 Sean nueve puntos de una retı́cula en el espacio euclidiano de tres


dimensiones. Muestre que hay un punto de la retı́cula en uno de los segmentos
que unen dos de estos puntos.

Solución. Sólo hay ocho patrones de paridad para estos puntos de la retı́cu-
la: (par, par, par), (par, par, impar),...,(impar, impar, impar). Ya que hay
nueve puntos dados, dos de ellos deben tener el mismo patrón de paridad.
Su punto medio es un punto de la retı́cula, y ası́ queda terminada la de-
mostración.

1.10.2 Coloque un peón en cada silla de un tablero de ajedrez de 7 por 7.


Es posible que cada peón haga simultáneamente un movimiento permitido?

Solución. Supongamos que un tablero está iluminado de la forma usual. El


tablero tiene 49 casillas; supongamos que 24 de ellas son balncas y 25 son
negras.

Consideremos los 25 peones que están en las casillas negras. Si cada uno de
ellos hiciera un movimiento permitido, tendrı́an que moverse a 25 casillas
blancas. Sin embargo, sólo hay 24 casillas blancas disponibles, por lo que
dicha jugada no puede llevarse a cabo.

1.10.3 Coloque un peón sobre el tablero de ajedrez de 4 por n. Es posible,


en 4n movimientos consecutivos del peón, visitar cada casilla del tablero y
regresar a la casilla de partida?
1.10. BÚSQUEDA DE PARIDAD 63

Figura 1.27

Solución. Antes de considerar este problema, es interesante analizar la mis-


ma pregunta para un tablero de 7 por 7. Suponga que se intenta dicho ’paseo
cerrado’. En el primer movimiento el peón se mueve a una casilla de color
opuesto; en el segundo regresa a una casilla del mismo color; y ası́ se sigue. Ve-
mos que después de un número impar de movimientos el peón ocupará una
casilla del color opuesto al de su casilla original. Un paseo cerrado por el
tablero de 7 por 7 requiere de 49 movimientos, un número impar. Ası́, el
peón no puede regresar a su casilla original, y el paseo cerrado es imposible.

Consideremos ahora el tablero de 4 por n. La argumentación utilizada para


el de 7 por 7 no puede ser aplicada en este caso, porque 4n es un número
par. Para manejar este caso, iluminemos el tablero de la forma que se indica
en la Figura 1.27.

observemos que los movimientos del peón hechos desde una casilla blanca
en las filas superior e inferior llevan a casillas blancas en las filas segunda y
tercera. Inversamente, en un paseo como el requerido, los movimientos del
peón desde las dos filas interiores llevan a casillas blancas en las dos filas
exteriores. Esto se debe a que hay exactamente n casillas blancas en las filas
exteriores, y éstas sólo pueden ser alcanzadas desde las n casillas blancas
en las filas inteiores. Ası́ pues, el camino del peón nunca puede pasar de las
casillas blancas a las negras, y por lo tanto el paseo cerrado es imposible.

1.10.4 Sea n un entero impar mayor que 1, y sea A una matriz simétrica
de n por n tal que cada renglón y cada columna son una permutación de los
enteros 1, ..., n. Muestre que cada uno de los enteros 1, ..., n debe aparecer en
la diagonal de A.
64 CAPÍTULO 1. HEURÍSTICA

Solución. Los elementos fuera de la diagonal están por pares ya que A es


simétrica. Cada número aparece exactamente n veces, y esto, junto con el
hecho de que n es impar, no lleva al resultado.

1.10.5 Sea a1 , a2 , ..., a2n+1 un conjunto de enteros con la siguiente propiedad


(P ) : si alguno de ellos es eliminado, los restantes pueden ser separados en
dos conjuntos de n enteros con sumas iguales. Demuestre que a1 = a2 = ... =
a2n+1 .

Solución. Primero, observamos que todos los enteros a1 , a2 , ..., a2n+1 tienen
la misma paridad. Para ver esto, sea A = a1 + a2 + ... + a2n+1 . La afirmación
se sigue después de observar que para cada i, A + ai es par (de otra forma
los números restantes no podrı́an ser separados de la manera requerida).

Sea a el número más pequeño de a1 , a2 , ..., a2n+1 , y para cada i, sea bi = ai +a.
El problema es equivalente a mostrar que bi = 0 para toda i.

Ahora bien b1 , b2 , ..., b2n+1 cumplen la propiedad (P ) . Ya que ninguno de


ellos es cero, positivo más grande tal que 2k divide a cada bi . Para cada i,
sea ci = bi /2k . Entonces c1 , c2 , ..., c2n+1 cumplen (P ) ; sin embargo, no todos
tienen la misma paridad ( ya que uno de ellos es cero, y otro es impar dada
la elección de k). Por lo tanto, todos los bi son cero y la demostración queda
terminada.

Problemas

1.10.6 .

(a) Quite la casilla inferior izquierda y la superior derecha de un tablero


normal de ajedrez de 8 por 8. Se puede cubrir el tablero resultante con
31 fichas de dominó ? Suponga que cada ficha cubre exactamente dos
casillas adyacentes del tablero.

(b) Sean P1 , ..., P13 puntos del plano y suponga que estan unidos por los
segmentos P1 P2 , P2 P3 , ..., P12 P13, P13 P1 . Es posible trazar una lı́nea recta
que pase por el interior de cada uno de estos segmentos?
1.10. BÚSQUEDA DE PARIDAD 65

1.10.7 .

(a) Es posible trazar una trayectoria que pase por los arcos de la Figu-
ra 1.28(a) que atraviese cada arco una vez y sólo una ? (Sugerencia:
Cuente el número de arcos que salen de cada vértice.)
(b) Es posible trazar una trayectoria por las lı́neas de la Figura 1.28(b)
que pase por cada unión una y sólo una vez? (Sugerencia: Ilumine los
vértices de forma alternada.)

b b

b b

(a)
b

b
b b

b b

b b

b b b

b b
b

(b)
Figura 1.28

1.10.8 Sea a1 , a2 , ..., an un arreglo arbitrario de los números 1, ...., n. De-


muestre que, si n es impar, el producto

(a1 − 1) (a2 − 2) ... (an − n)


66 CAPÍTULO 1. HEURÍSTICA

es un número par.


1.10.9 Muestre que (2a − 1) 2b − 1 = 2c + 1 es imposible para enteros no
negativos a, b, y c. (Sugerencia: Escriba la ecuación de la forma equivalente
2a+b − 2a − 2b = 2c e investigue las posibilidades de a, b y c.)

1.10.10 Muestre que x2 − y 2 = a3 siempre tiene soluciones enteras para x


y y siempre que a sea un número positivo.

Ejemplos adicionales

1.5.10,1.9.1, 2.2.7, 3.2.13, 3.3.4, 3.3.20, 4.2.16(a), 4.3.4, 7.4.6. Ver sección 3.2
para una generalización de este método.

1.11. Consideración de Casos Extremos

En las primeras etapas de exploración de un problema, con frecuencia es de


gran ayuda considerar las consecuencias de variar los parámetros del proble-
ma de un valor extremo a otro. En esta sección veremos que la existencia de
posiciones extremas son frecuentemente la clave para entender resultados de
existencia (problemas del estilo ’pruebe que existe x tal que P (x)’).

1.11.1 Dado un número finito de puntos en un plano, no todos colineales,


pruebe que existe una lı́nea recta que pasa exactemente por dos de ellos.
P b

b b b

Q
b

P3 P2 P1
Figura 1.29

Solución. Si P es un punto y L una lı́nea, sea d (P, L) la distancia desde


P a L. Sea S el conjunto de las distancias positivas d (P, L) conforme P
1.11. CONSIDERACIÓN DE CASOS EXTREMOS 67

varı́a por los puntos dados, y L varı́a por las lı́neas que no pasan por P pero
que si pasan por al menos dos de los puntos dados. El conjunto S es no
vacı́o (porque los puntos dados no son todos colineales ) y finito (hay sólo
un número finito de puntos y un número finito de rectas que pasan por al
menos dos puntos ). Por lo tanto S tiene un elemento al minimal, digamos
d (P, M) . Afirmamos que M pasa exactamente por dos de los puntos dados.

Supongamos que M pasa por tres de los puntos dados, digamos P1 , P2 y P3 .


Sea Q el punto de M más cercano a P. Al menos dos de los puntos P1 , P2
y P3 están del mismo lado de Q (uno de ellos podrı́a ser Q), digamos P2 y
P3 (ver Figura 1.29). Supongamos que los puntos están etiquetados de forma
que P2 está más cerca de P que P3 . Sea N la lı́nea através de P y P3 , y
observamos que d (P2 , N) < d (P, M), una contradicción respecto a nuestra
elección de P y M. Se sigue que M sólo puede pasar por dos de los puntos
dados.

Supongamos que M pasa por tres de los puntos dados, digamos P1 , P2 y P3 .


Sea Q el punto de M más cercano a P. Al menos dos de los puntos P1 , P2 , P3
están del mismo lado que Q (uno de llos podrı́a ser Q ), digamos P1 y P3 (ver
Figura 1.27). Supongamos que los puntos están etiquetados de forma que P2
está más cerca de P que P3 . Sea N la lı́nea através de P y P3 , y observemos
que d (P2 , N) < d (P, N) , una contradicción respecto a nuestra elección de P
y M. Se sigue que M sólo púede pasar por dos de los puntos dados.

1.11.2 Sea A un conjunto de 2n puntos en el plano, de los cuales no hay


tres que sean colineales. Suponga que n de ellos son rojos y los otros n son
azules. Pruebe o refute: Existen n segmentos de lı́nea recta cerrados, de los
cuales no hay dos que tengan un punto en común, tales que los extremos de
cada segmento son puntos de A de distinto color.

Solución. Si ignoramos las intersecciones de lı́neas, existe una variedad de


formas en las que los puntos rojos pueden ser apareados con lo azules con n
segmentos de lı́nea cerrados. Asignemos a cada uno de estos apareamientos
la longitud total de los segmentos de lı́nea en la configuración. Como sólo
hay un número finito de tales apareamientos, una de estas configuraciones
tendrá una longitud total minimal. Este apareamiento no tendrá cruces entre
lı́neas. (Si los segmentos R1 B1 y R2 B2 se intersectan, con R1 , R2 puntos rojos
y B1 , B2 puntos azules, entonces podrı́amos reducir la longitud total de la
68 CAPÍTULO 1. HEURÍSTICA

configuración reemplazando estos segmentos por R1 B2 y R2 B1 .)

1.11.3 En una fiesta, ningún muchacho baila con todas las muchachas, pero
cada muchacha baila al menos con un muchacho. Pruebe que existen dos
parejas bg y b′ g ′ que bailan, considerando que si b no baila con g ′ , tampoco g
baila con b′ .

Solución. Aunque no es necesario, el problema podrı́a ser más compren-


sible si lo ponemos en términos matriciales. Hagamos que los renglones de
una matriz correspondan a los muchachos y las columnas a las muchachas.
Escribamos un 1 o un 0 en el renglón b y la columna g dependiendo de si b
y g bailan juntos o no. La condición de que no hay un muchacho que baile
con todas las muchachas implica que (i) cada renglón tiene al menos una
entrada 0. Similarmente, (ii) cada columna tiene al menos una entrada 1.
Queremos probar que existen dos renglones, b y b′ , y dos columnas, g y g ′,
cuyas entradas de intersección tienen el patrón

   
.. .. .. ..
. . . .
   
 ··· 1 ··· 0 ···   ··· 0 ··· 1 ··· 

 .. .. 
 o

 .. .. 

 . .   . . 
 ··· 0 ··· 1 ···   ··· 1 ··· 0 ··· 
   
.. .. .. ..
. . . .

Sea h un renglón arbitrario. Por (i) hay una entrada 0 en este renglón, dig-
amos en la columna k, y por (ii) hay una entrada 1 en la columna k, digamos
en el renglón m :

k
 
.. ..
. .
 
 ··· 1 ··· 0 ··· 
h 
 .. .. 

 . . 
 
m  · · · 0? · · · 1 · · · 
.. ..
. .
1.11. CONSIDERACIÓN DE CASOS EXTREMOS 69

Ahora bien, ya que los hicimos si hay una columna que tenga 1 en el renglón
h y 0 en la columna m, En general, tal columna podrı́a existir. Sin embargo,
si h hubiera sido escogido con anticipación como renglón con un número
maximal de 1’s, entonces tal columna no hubiera existido y el problema no
hubiera sido resuelto.

Con estos antecedentes, podemos reescribir la solución en lenguaje indepen-


diente de la notación matricial. Sea b un muchacho que baila con un número
maximal de muchachas. Sea g ′ una muchacha con quien b no baila, y b′ un
muchacho con el que g ′ baila. Entre los compañeros de b, debe haber al menos
una muchacha g que no baile con b′ (de otra forma b′ tendrı́a más compañeras
que b ). Entonces las parejas bg y b′ g ′ resuelven el problema.

1.11.4 Demuestre que el producto de n enteros sucesivos siempre es divisible


por n!.

Solución. Primero, observamos que es suficiente demostrar el resultado para


n enteros positivos sucesivos. El resultado es obviamente cierto si uno de los
enteros es 0, mientras que si todos los enteros son negativos, basta mostrar
que n! divide a su valor absoluto.

Ası́ pues, supongamos que existen n enteros sucesivos cuyo producto no


es divisible por n!. De todos estos n números, escojamos al más pequeño;
llamémosle N. Observemos que N > 2, ya que el producto de dos enteros
sucesivos siempre es par. Estamos suponiendo, por lo tanto, que existe un
número no negativo m tal que (m + 1) (m + 2) ... (m + n) no es divisi-
ble por N!. De todos estos números m, sea M el más chico. Observemos
que M > 0, ya que N! es divisible por N!. Ası́, estamos suponiendo que
(M + 1) (M + 2) ... (M + N) no es divisible por N!. Ahora,

(M + 1) (M + 2) ... (M + N − 1) (M + N)
= M[(M + 1) (M + 2) ... (M + N − 1)] +
N[(M + 1) (M + 2) ... (M + N − 1)]

Por la forma en que escogimos M, N! divide a M [(M + 1) (M + 2) ...


(M + N − 1)]. Por la forma en que escogimos N, (N − 1)! divide a (M + 1) (M + 2) ...
70 CAPÍTULO 1. HEURÍSTICA

(M + N − 1), y en consecuencia N! divide a N [(M + 1) (M + 2) ... (M + N − 1)].


Combinando, vemos que N! divide al lado derecho de la última ecuación, en
contradicción con nuestra suposición. Esta contradicción establece el resul-
tado.

(Una prueba ingeniosa de este resultado esidentificar


 al cociente (m + 1) (m + 2) ...
m+n
(m + n) /n! como el coeficiente binomial , y por lo tanto es un en-
n
tero si m es un entero.)

Problemas

1.11.5 Sea f (x) un polinomio de grado n con coeficientes reales y tal que
f (x) ≥ 0 para todo número real x. Muestre que f (x) + f ′ (x) + ... + f (n) (x)
para todo real x. ( f (k) (x) denota la k-ésima derivada de f (x) . )

1.11.6 Dé un ejemplo para mostrar que el resultado de 1.11.1 no necesari-


amente se cumple para un número infinito de puntos en el plano. En dónde
falla la demostración de 1.11.1 para el caso infinito?

1.11.7 Muestre que existe un número racional, c/d, con d < 100, tal que
h c i  73 
k = k para k = 1, 2, ..., 99.
d 100

1.11.8 Suponga que Pn es una afirmación, para n = 1, 2, 3, ... . Suponga


además que

(i) P1 es cierto, y

(ii) para cada entero positivo m, Pm+1 es cierto si Pm es cierto.


1.12. GENERALIZACIÓN 71

Demuestre que Pn es cierto para todo n. (Sugerencia: Sea S el conjunto de


todos los enteros positivos para los cuales Pn no es cierto. Sea m el elemento
más pequeño en S, suponiendo que S es no vacı́o.)

Ejemplos adicionales

3.1.9, 3.3.11, 3.3.28, 4.4.7, 4.4.10, y los referentes dados en 6.3.7. También, ver
sección 7.6 (El Principio del Emparedado) y 6.2 (El teorema del valor inter-
medio) para ejemplos que requieren la consideración de casos ”extremales”.

1.12. Generalización

Puede parecer paradójico, pero con frecuencia se presenta el caso en que un


problema puede ser simplificado, y hecho accesible y comprensible, cuando
se le generaliza. Este hecho de la vida es muy apreciado por los matemáticos;
de hecho, la abstracción y la generalización son caracterı́sticas básicas de la
matemática moderna. Un contexto más general provee de una perspectiva
más amplia, hace a un lado los rasgos no esenciales, y proporciona todo un
arsenal de técnicas nuevas.

Pn k2
1.12.1 Evalúe la suma k=1 2k .
P
Solución. Evaluaremos
 en cambio la suma S (x) = nk=1 k 2 xk y entonces
calcularemos S 21 . La razón para introducir la variable x es que ahora
podemos utilizar técnicas del análisis. Sabemos que

n
X 1 − xn+1
xk = , x 6= 1.
k=1
1−x

Derivando ambos lados obtenemos


72 CAPÍTULO 1. HEURÍSTICA

n
X (1 − x) (− (n + 1) xn ) + (1 + xn+1 )
kxk−1 =
k=1
(1 − x)2
1 − (n + 1) xn + nxn+1
= .
(1 − x)2

Multiplicando ambos lados de está ecuación por x, derivando una segunda


vez, y multiplicando el resultado por x obtenemos

n
X x (1 + x) − xn+1 (nx − n − 1)2 − xn+2
S (x) = k 2 xk = .
k=1
(1 − x)3

Se sigue que

  n  2
1 X k2 1 1 1
S = k
= 6 − n−2 n − n − 1 − n−1
2 k=1
2 2 2 2
 2 
n + 4n + 6
= 6− .
2n

1.12.2 Evalúe el siguiente determinante (determinante de Vandermonde):

 
1 a1 a21 · · · a1n−1

 1 a2 a22 · · · a2n−1 

det  .. .. .. .. .
 . . . . 
1 an a2n · · · ann−1

Solución. Supondremos que ai 6= aj , i 6= j, porque de otra forma el de-


terminate serı́a cero. Con el objeto de enfocarnos más claramente en la idea
principal, consideraremos el caso n = 3 :
1.12. GENERALIZACIÓN 73

 
1 a a2
det  1 b b2  .
1 c c2

En este determinante, reemplacemos c por una variable x. Entonces el de-


terminate es un polinomio P (x) de grado 2. Además, P (a) = 0 y P (b) = 0,
ya que la matriz correspondiente, con c sustituı́da por a o b respectivamente,
tiene entonces dos renglones idénticos. Por lo tanto,

P (x) = A (x − a) (x − b)
para alguna constante A. Ahora, A es el coeficiente de x2 , y, regresando al
determinante, encontramos que este coeficiente es

 
1 a
det .
1 b

Ası́ A = b − a, y el determinante original de 3 por 3 es

P (c) = (b − c) [(c − a) (c − b)].

El caso general es análogo. Sea Dn el determinante buscado (de orden n


). Sustituimos an en el último renglón de la matriz por la variable x. El
determinante resultante es un polinomio Pn (x) de grado n − 1, que se hace
cero en a1 , a2 , ..., an−1 . Luego, por el Teorema del Factor,

Pn (x) = A (x − a1 ) (x − a2 ) ... (x − an−1 ) ,


donde A es constante. Como en el caso anterior, A es el coeficiente de xn , y
expandiendo sobre el último renglón queda claro que A = Dn−1 . Esto es,

Dn = Pn (an ) = Dn−1 [(an − a1 ) (an − a2 ) ... (an − an−1 )].


Podemos repetir el argumento para Dn−1 ; etc. El resultado final será
74 CAPÍTULO 1. HEURÍSTICA

n
"k−1 #
Y Y
Dn = (ak − ai ) .
k=2 i=1

R∞ R∞
1.12.3 Dado que 0
(senx) /x dx = 21 π, evalúe 0
(sen2 x) /x2 dx.

Solución. Evaluaremos la integral más general

Z ∞ 
I (a) = sen2 ax /x2 dx, a ≥ 0.
0
utilizando una técnica llamada diferenciación de parámetros.

Derivando cada lado de la ecuación con respecto a a; obtenemos

Z ∞
′ 2senax cos ax · x
I (a) = dx
x2
Z0 ∞
sen2ax
= dx.
0 x
Ahora, con y = 2ax, obtenemos dy = 2adx, y

Z ∞
′ seny 1
I (a) = = π.
0 y 2
Integrando ambos lados queda

1
I (a) = πa + C. C constante.
2
1
Ya que I (0) = 0, tenemosR ∞que C2 = 0,2 Ası́ I (a)1 = 2 πa, a ≥ 0. Haciendo
a =R 1 se tiene I (1) = 0 (sen x) /x dx = 2 π. (Casualmente, el valor

de 0 (senx) /x dx puede ser encontrado al evaluar una curva en el plano
complejo.)

Problemas
1.12. GENERALIZACIÓN 75

1.12.4 Tomando x los valores apropiados en la expansión binomial


n  
X n n k
(1 + x) = x
k=1
k

(o en una de sus derivadas) evalúe cada uno de los siguientes

   
Pn n
2
Pn k n
(a) k=1 k , (b) k=1 3 ,
k  k 
Pn 1 n Pn n
(c) k=1 k+1 , (d) k=1 (2k + 1) .
k k

1.12.5 Evalúe  
1 a a2 a4
 1 b b2 b4 
det  .
 1 c c2 c4 
1 d d2 d4
(Sustituya d por una variable x; haga uso del hecho de que la suma de las
raı́ces de un polinomio de cuarto grado es igual al coeficiente de x3 .)

1.12.6 .

R∞ R∞
(a) Evalúe 0 (e−x senx) /x dx. (Considere G (k) = 0 (e−x senkx) /x dx
y utilice diferenciación paramétrica.)
R1 R1
(b) Evalúe 0 (x − 1) / ln x dx. (Considere H (m) = 0 (xm − 1) / ln x dx
y utilice diferenciación paramétrica.)

(c) Evalúe Z ∞
arctan (πx) − arctan x
dx.
0 x
R∞ arctan(ax)−arctan x
(Considere F (a) = 0 x
dx y utilice diferenciación paramétri-
ca.)
76 CAPÍTULO 1. HEURÍSTICA
√ √
1.12.7 Cuál número es más grande. 3 60 o 2 + 3 7? (Elevar al cubo cada
número lleva a complicaciones que no son de fácil resolución.
p Considere
√ en
cambio el problema más general: Cuál cantidad es mayor. 4 (x + y) o x+
3 3
√3 y, donde x, y ≥ 0? Haga x = a3 , y = b3 .)

Ejemplos adicionales

1.4.2, 2.2.6, 2.2.7, 4.1.4, 5.1.3, 5.1.4, 5.1.9, 5.1.11, 5.4.4, 5.4.5, 5.4.6, 5.4.7,
6.9.2, 7.4.4. También, ver la sección 2.4 (Inducción y Generalización)
Capı́tulo 2

Dos Importantes Principios: El


de Induccción y el del Palomar.

Las proposiciones matemáticas vienen en dos formas: proposiciones univer-


sales las cuales establecen que algo es verdadero para todos los valores de
X en algún conjunto especificado, y proposiciones existenciales las cuales
establecen que algo es verdadero para algún valor de X en algún valor es-
pecificado. Las primeras son expresables en la forma ”para todo X (en un
conjunto S), P (X)”., las del segundo tipo son expresables en la forma ”allı́ ex-
iste un X (en el conjunto S)tal que P (X)”, donde P (X) es una afirmación
a cerca de X. En este capı́tulo vanos a considerar dos importantes técnicas
para el trato con esos dos tipos de afirmaciones: (i) el principio de inducción
matemática, para proposiciones universales, y (ii) el principio del palomar
para proposiciones existenciales.

2.1. Inducción: Construcción Sobre P (k).

Sea a un entero y P (n) una proposición (afirmación) sobre n para cada entero
n ≥ a. El principio de inducción matemática establece que:

Si

77
78CAPÍTULO 2. DOS IMPORTANTES PRINCIPIOS: EL DE INDUCCCIÓN Y EL DEL P

(i) P (a) es verdadero, y

(ii) para cada entero k ≥ a, P (k) verdadero implica P (k + 1) verdadero


entonces P (n) es verdadero para todo entero n ≥ a.

Nótese que el prinicipio lo habilitamos, en dos simples pasos, probar un


número infinito de proposiciones (a saber, P (n) es verdadero para todo entero
n ≥ a).

El método es especialmente conveniente cuando un modelo ha sido estable-


cido (ver sección 1.1 ”búsqueda de un modelo”) para los primeros casos
especiales (P (a), P (a + 1), P (a + 2),.....).

En esta sección vamos a considerar argumentos de inducción como, en el


paso (ii), procediendo directamente de la verdad de P (k) hacia la verdad de
P (k + 1), esto es la verdad de P (k + 1) es ”construida sobre” la consideración
inicial de la verdad de P (k).

Esto es en ligero contraste con los argumentos (considerados en la siguiente


sección) en los que se comienza con una consideración de P (k + 1).

2.1.1 Use inducción matemática para probar el teorema del binomio:


P 
(a + b)n = ni=0 ni ai bn−i , n un entero positivo.

Solución. Es fácil checar que la fórmula vale para n = 1.

Suponiendo que la fórmula vale para el entero k (estamos trabajando sobre


la verdad de P (k)), multiplicando ambos lados por (a + b) obtenemos

   
k Pn k i k−i
(a + b) (a + b) = i=0 ab (a + b)
  i  
P k i+1 k−i Pk k i k+1−i
= ki=0 a b + i=0 ab
i i

en la primera suma, hacemos el cambio de variables j = i + 1, y obtenemos.


2.1. INDUCCIÓN: CONSTRUCCIÓN SOBRE P (K). 79

   
Pk+1 k j k+1−j
Pk k i k+1−i
= j=1 ab + i=0 ab
 j − 1   i    
Pk k j k+1−j k+1
Pk k i k+1−i k+1
= j=1 ab +a + i=1 ab +b
 j − 1      i
Pk k k
= ak+1 + i=1 + ai bk+1−i + bk+1
 i −1 i
k+1
Pk k + 1 i k+1−i
=a + i=1 ab + bk+1
  i
Pk+1 k + 1 i k+1−i
= i=0 ab
i
     
k k k+1
donde hemos hecho uso de la identidad básica + =
i−1 i i
(ver sección 2.5.2). Esta es la forma para P (k + 1), entonces por inducción,
la prueba se ha completado.

2.1.2 Suponga que 0 < a1< a2 <  .... < an , y suponga ei = ±1. Probar que
Pn n+1
i=1 ei ai toma al menos valores distintos según varie ei sobre 2n
2
posibles combinaciones de signos.

Solución.
  Cuando n = 1, hay exactamente 2 valores distintos (a1 y −a1 ),
2
y = 1, el resultado es válido.
2

Supóngase que este resultado


 es verdadero cuando n = k; esto es, que

Pn k+1
i=1 ei ai tiene al menos valores distintos, y suponga que ak+1 > ak .
2   
k+2 k+1
Nosotros necesitamos generar − = k + 1 sumas adicionales.
2 2
P
Esto lo podemos conseguir en la siguiente forma: Suponga que S = ki=1 ai y
note que S + ak+1 , S + (ak+1 − 2a1 ) , ..., S + (ak+1 − 2ak−1 ) , S + (ak+1 − 2ak ) ,
son distintos y mayores que cada una de las sumas obtenidas anteriormente.
(Para ver esto note que S + (ak+1 − 2ak ) > S + (ak+1 − 2ak+1 ) = S − ak+1 ).
Alli hay k + 1 números en esa lista, entonces el resultado se sigue por induc-
ción.
80CAPÍTULO 2. DOS IMPORTANTES PRINCIPIOS: EL DE INDUCCCIÓN Y EL DEL P

La inducción matemática es un método que se puede utilizar en un proble-


ma de la forma ”Probar que P (n) es verdadero para todo n ≥ a. Pero se
debe notar que la inducción también se aplica a muchos problemas donde la
cuantificación es sobre conjuntos mas generales. Por ejemplo, una proposi-
ción acerca de todos los polinomios puede ser probada por inducción sobre
el grado del polinomio. Un teorema acerca de todas las matrices puede ser
manejado por inducción sobre el tamaño de la matriz. Varios resultados ref-
erentes a proposiciones en lógica simbólica son realizados por inducción sobre
el número de conectivos lógicos en la proposición. La lista de conjuntos in-
ductivos poco comunes puede continuar indefinidamente. Nosotros estamos
satisfechos de ver exactamente dos ejemplos aqui., otros ejemplos son dis-
tribuidos a traves del libro. (ver las cuatro secciones siguientes y las listas en
los ejmplos adicionales).

2.1.3 Si V , E y F son, respectivamente, los números de vertices, aristas y


caras de un mapa plano conexo, entonces

V − E + F = 2.

Solución. El entendimiento intuitivo de los términos es este resultado es


probablemente exacto, pero para asegurarse, aqui están las definiciones.

Una red es una figura (en un plano o en el espacio) consistente de un número


finito y mayor que cero de arcos, donde dos arcos no se intersectan excep-
to posiblemente en sus puntos finales. Los puntos finales de esos arcos son
llamados vértices de la red.

Un camino en una red es una sucesión de diferentes arcos en la red que


puede ser recorrido continuamente sin repasar un arco. Una red es conexa si
cualesquiera dos diferentes vértices de la red son vertices de algún camino en
la red. Un mapa es una red, junto con una superficie la cual contiene a la
red. Si esta superficie es plana el mapa es llamado un mapa plano, los arcos
de un mapa plano son llamados aristas. Las caras de un mapa plano son las
regiones que estan definidas por las fronteras (aristas) de el mapa (el oceano
es tomado en cuenta como una cara).
2.1. INDUCCIÓN: CONSTRUCCIÓN SOBRE P (K). 81

b b b b

b b

b b b

b b
b b b

1 2 3
Figura

La figura 2.1. muestra tres ejemplos de redes conexas. Las primeras dos son
mapeos planares. En el primero, V = 4, E = 4, F = 2., en el segundo,
V = 5, E = 6, F = 3. La tercera red es un mapa no plano. Sin embargo,
si nosotros lo aplastamos sobre un plano y llamamos vertices a los nuevos
puntos de intersección, entonces tenemos V = 10, E = 20, F = 12.

Ahora volviendo a la consideración del teorema. La idea clave en la prueba


de este resultado es que es realizable porque los mapas planos conexos se
pueden construir de un vertice singular por una sucesión de las siguientes
construcciones (cada uno de los cuales es un mapeo conexo):

i)] sumando un vertice en una arista existente (. . quedando ası́ . . .).

ii)] sumando una arista a un vertice la cual parte de el y regresa a el (.


quedando ası́ ).

iii)] sumando una arista entre dos vertices existentes (: : quedando ası́ ).

iv)] sumando una arista y un vertice a otro vertice existente (. quedando


ası́ . .).

Vamos aplicar inducción sobre el número de pasos requeridos para la con-


strucción del mapeo planar conexo. Si la red consiste de un solo punto, en-
tonces V = 1, F = 1, E = 0, teniendose V − E + F = 2.

Supóngase que el resultado es valido cuando se requieren K pasos en la con-


strucción. El cambio neto para cada uno de los pasos es dado en la siguiente
tabla.
82CAPÍTULO 2. DOS IMPORTANTES PRINCIPIOS: EL DE INDUCCCIÓN Y EL DEL P

Operación △V △E △F △ (V − E + F )
(i) +1 +1 0 0
(ii) 0 +1 +1 0
(iii) 0 +1 +1 0
(iv) +1 +1 0 0

Observese que la cantidad V −F +E permanece invariable cuando es tomado


el (k + 1) ésimo paso. Ası́ la prueba se completa por inducción.

2.1.4 Dado un entero positivo n y un número real x, probar que:

     
1 2 n−1
[[x]] + x+ + x+ +···+ x + = [[nx]] .
n n n

Solución. Aunque hay un parametro n entero en este problema, no podemos


usar inducción sobre n para un x fijo. También por regla no podemos usar
inducción sobre x, ya que x toma rangos sobre los números reales (pues dado
un número x real no podemos determinar el número real y mayor inmediato a
x). Por tanto no es claro como se puede aplicar la inducción a este problema.

La
h idea ies probar la fórmula para un n fijo y para todo x en el subintervalo
k (k+1)
n n
para k = 0, ±1, ±2, . . . .
 
Primero supóngase que x pertenece al subintervalo [0 1/n) entonces x + nl =
Pn−1  1

0 para todo l = 0, 1, . . . , n − 1, también que i=0 x + n
= 0. Además
[nx] = 0. Luego la fórmula es válida en el “primer” subintervalo.
 
Ahora supóngase que la fórmula es cierta en el subintervalo k−1n n
k
, donde k
es un entero positivo y x se supone un número real en este intervalo. Entonces

     
1 2 n−1
[[x]] + x+ + x+ +···+ x + = [[nx]] .
n n n
2.1. INDUCCIÓN: CONSTRUCCIÓN SOBRE P (K). 83
 
Ahora sumando 1/n a x (lo que nos da un nuevo número en nk , k+1 n
) en cada
uno de los términos, excepto en el término final, del lado izquierdo se repite

cada uno de los términos de la ecuación previa, y el término final x + n−1 n
,
nos queda [x + 1] que excede a [x] por 1. Luego reemplazando en la ecuación
previa a x por x + n1 el lado izquierdo crece en 1.

Al mismo tiempo, cuando x en [nx] es reemplazado por x + n1 , el valor se


incrementa en 1. Ya que ambos lados de la ecuación se incrementan en 1
cuando x es reemplazado por x + n1 , elresultado continua siendo verdadero
para todos los números en el intervalo nk , k+1
n
.

Por inducción la fórmula es válida para todos los valores positivos de x. Un


argumento similar muestra que es válida para todos los valores negativos de
x (reemplazando x por x − n1 ).

El siguiente ejemplo es una buena ilustración de la construcción de P (k + 1)


a partir de P (k) .

2.1.5 Si a > 0 y b > 0, entonces (n − 1) an + bn ≥ nan−1 b, n es un entero


positivo, con la igualdad solo si a = b.

Solución. El resultado es verdadero para n = 1, suponiendo que la fórmula


es verdadera para el entero k. Para construir P (k + 1) , debemos, obtener el
lado izquierdo apropiado

(i) multiplicar por a:

(k − 1) ak+1 + bk a ≥ kak b

(ii) sumando ak+1 :

kak+1 + bk a ≥ kak b + ak+1

(iii) restando bk a :
84CAPÍTULO 2. DOS IMPORTANTES PRINCIPIOS: EL DE INDUCCCIÓN Y EL DEL P

kak+1 ≥ kak b + ak+1 − bk a

(iv) sumando bk+1 :

kak+1 + bk+1 ≥ kak b + ak+1 − bk a + bk+1

Estamos suponiendo que esta desigualdad es una igualdad solo si a = b. Solo


queda mostrar que kak b + ak+1 − bk a + bk+1 ≥ (k + 1) ak b con la igualdad
solo si a = b. Para ello trabajaremos hacia atrás:

kak b + ak+1 − bk a + bk+1 ≥ (k + 1) ak b


−ak b + ak+1 − bk a + bk+1 ≥ 0,
ak (a − b) + bk (b − a) ≥ 0,

ak − bk (a − b) ≥ 0


el último paso es verdadero ( ak − bk y (a − b) tienen el mismo signo) con
la igualdad se da solo si a = b, entonces la prueba se sigue por inducción.
(Observe que: este resultado es un caso especial de el significado geométrico
de una desigualdad aritmética, ver sección 7.2.).

Problemas

2.1.6 .
√ √ √ √
Use inducción para probar que 1 + 1/ 2 + 1/ 3 + · · · + 1/ n < 2 n.

Use inducción para probar que 2!4! . . . (2n!) ≥ ((n + 1)!)n .


2.1. INDUCCIÓN: CONSTRUCCIÓN SOBRE P (K). 85

2.1.7 El plano euclidiano es dividido en regiones por el trazo de un número


finito de lineas rectas. Mostrar que es posible colorear cada una de esas re-
giones de azul o rojo en una manera tal que no haya dos regiones adyacentes
que tengan el mismo color.

2.1.8 Probar que la ecuación x2 + y 2 = z n tiene una solución en los enteros


positivos (x, y, z) para todo n = 1, 2, 3, . . . . (Para una buena prueba, divida
en dos casos: n par y n impar. Para una prueba no inductiva, ver 3.5.1.).

2.1.9 Un grupo de n personas juega un torneo robin-round. Cada juego


finaliza en una victoria o una dorrota. Mostrar que es posible etiquetar a los
jugadores con p1 , p2 , p3 , . . . , pn en una manera tal que p1 derrote a p2 , p2
derrote a p3 ,. . .,pn−1 derrote a pn .

2.1.10 Si cada persona, en un grupo de n personas, es amigo de al menos


la mitad de personas en el grupo, entonces es posible sentar las n personas
en un cı́rculo de manera que cada uno este sentado solo junto a amigos.

2.1.11 Los siguientes pasos conducen a otra prueba del teorema del bi-
nomio. Conocemos que (a + x)n se puede escribir como un polinomio de gra-
do n, de manera que ahi hay constantes A0 , A1 , . . . An tales que
(a + x)n = A0 + A1 x + A2 x2 + · · · + An xn .

Use inducción para describir la ecuación que resulta tomando progresiva-


mente la k − ésima derivada de cada lado de esta ecuación (k = 1, 2, . . . , n) .

Evaluar Ak para k = 0, 1, . . . , n poniendo x = 0 en la k − ésima ecuación


obtenida en la parte (a).

2.1.12 Supóngase que f : R → R es una función para la cual f (2x − f (x)) =


x para todo x, suponga que r es un número real fijo.

Probar que si f (x) = x + r, entonces f (x − nr) = (x − nr) + r para todo


entero positivo n.
86CAPÍTULO 2. DOS IMPORTANTES PRINCIPIOS: EL DE INDUCCCIÓN Y EL DEL P

Probar que si f es una función uno a uno (o sea f (x) = f (y) implica x = y
) entonces la propiedad en (a) es válida para todo entero n.

Ejemplos adicionales

1.1.2, 1.1.8, 3.2.8, 6.5.13, 7.1.4.

2.2. Inducción: Pensando sobre P (k + 1)

En esta sección vamos a considerar argumentos de inducción en los cuales


empezamos con un avance directo sobre p (k + 1) y en los cuales maniobramos
hacia atras para explotar la hipotetica verdad de P (k) . Teoricamente, los
argumentos en esta sección se puedieron todos refundar como en la sección
previa y viceversa. Sin embargo, desde un punto de vista práctico, muchas
veces es mas convincente pensar un camino, que es mejor que el otro.

n5 n4 n3 n
2.2.1 Probar que 5
+ 2
+ 3
− 30
es un entero para n = 0, 1, 2, . . . .

Solución. El resultado es obvio cuando n = 0, suponiendo que la fórmula


da un entero cuando n = k. Necesitamos probar que

(k + 1)5 (k + 1)4 (k + 1)3 (k + 1)


+ + −
5 2 3 30
es un entero. Expandiendo la fórmula

k 5 + 5k 4 + 10k 3 + 10k 2 + 5k + 1 k 4 + 4k 3 + 6k 2 + 4k + 1 k 3 + 3k 2 + 3k + 1 k + 1
+ + −
5 2 3 30
y recombinando (para hacer uso de P (k)):

 
k5 k4 k3 k    
+ + − + k 4 + 2k 3 + 2k 2 + k + 2k 3 + 3k 2 + 2k + k 2 + k .
5 2 3 30
2.2. INDUCCIÓN: PENSANDO SOBRE P (K + 1) 87

La primera parte es un entero por la hipótesis inductiva y la segunda parte es


también un entero porque es la suma de enteros. Luego la prueba se sigue por
inducción. (Observe que dificil hubiera sido llegar a la verdad de P (k + 1)
partiendo de la de P (k) .)

2.2.2 Sean a, b, p1 , p2 , . . . , pn números reales con a 6= b, Definimos f (x) =


(p1 − x) (p2 − x) (p3 − x) · · · (pn − x) . Mostrar que

 
p1 a a a ··· a a

 b p2 a a ··· a a 


 b b p3 a ··· a a 
 bf (a) − af (b)

det  b b b p4 ··· a a 
= .
 .. .. .. .. .. .. ..  b−a
 . . . . . . . 
 
 b b b b · · · pn−1 a 
b b b b ··· b pn

Solución. Este es similar a muchos problemas de determinantes que se


pueden resolver con inducción matemática. Cuando n = 1, tenemos det (p1 ) =
p1 y f (x) = (p1 − x) entonces

bf (a) − af (b) b (p1 − a) − a (p1 − b)


= = p1
b−a b−a

y el resultado se cumple.

Supóngase que la igualdad es cierta para k − 1, k > 1, considere el caso


para k números reales p1 , . . . pk . (Comenzamos por la situación para P (k) y
el método recae hacia atras sobre la verdad de P (k − 1) para completar el
paso inductivo.) Deseamos evaluar:
88CAPÍTULO 2. DOS IMPORTANTES PRINCIPIOS: EL DE INDUCCCIÓN Y EL DEL P

 
p1 a a a ··· a a

 b p2 a a ··· a a 


 b b p3 a ··· a a 


det  b b b p4 ··· a a 

 .. .. .. .. .. .. .. 
 . . . . . . . 
 
 b b b b · · · pk−1 a 
b b b b ··· b pk

Restando la segunda columna a la primera (esto no hace cambiar el deter-


minante):

 
p1 − a a a a ··· a a

 b − p2 p2 a a ··· a a 


 0 b p3 a ··· a a 


det  0 b b p4 ··· a a 

 .. .. .. .. .. .. .. 
 . . . . . . . 
 
 0 b b b · · · pk−1 a 
0 b b b ··· b pk

y expandiendo la primera columna obtenemos

   
p2 a ··· a a a a ··· a a

 b p3 · · · a a 


 b p3 · · · a a 


(p1 − a) det  .. .. .. .. ..   .... . . .. .. 
. . . . . −(b − p2 ) det  . . . . . 
   
 b b ··· pk−1 a   b b ··· pk−1 a 
b b ··· b pk b b ··· b pk

Estos dos nuevos determinantes (de matrices (k − 1) por (k − 1)) son de la


forma para la cual podemos aplicar la suposición inductiva P (k − 1) . Para
hacer esto, necesitamos introducir alguna notación. Para el primer determi-
nate, establecemos F (x) = (p2 − x) (p3 − x) . . . (pk − x) y para el segundo,
establecemos G (x) = (a − x) (p3 − x) . . . (pk − x) . Entonces por la suposi-
ción inductiva,
2.2. INDUCCIÓN: PENSANDO SOBRE P (K + 1) 89

   
bF (a) − aF (b) bG (a) − aG (b)
(p1 − a) − (b − p2 ) .
b−a b−a
Pero G (a) = 0 y (p1 − a) F (a) = f (a) , y por lo tanto tenemos

bf (a) − a (p1 − a) (p2 − b) . . . (pk − b) − a (a − b) (p2 − b) . . . (pk − b) ,

b−a

bf (a) − a (p2 − b) . . . (pk − b) [(p1 − a) + (a − b)]

b−a

bf (a) − af (b)
b−a
El resultado se sigue por inducción.

Problemas

2.2.3 De una prueba para el paso inductivo en 1.1.3.

2.2.4 Para todo x en el intervalo 0 ≤ x ≤ π, probar que |sen nx| ≤ n sen


x, n un entero no negativo.

2.2.5 Suponga que Q denota el conjunto de los números racionales. Hallara


todas las funciones f : Q → Q las cuales satisfacen las dos condiciones
siguientes: (i) f (1) = 2 y (ii) f (xy) = f (x) f (y) − f (x + y) + 1 para todo
x, y en Q.
90CAPÍTULO 2. DOS IMPORTANTES PRINCIPIOS: EL DE INDUCCCIÓN Y EL DEL P

2.2.6 Si a, b, c ≥ 1, probar que 4 (abc + 1) ≥ (1 + a) (1 + b) (1 + c) . (Sug-


erencia: Probar, en forma mas general, que 2n−1 (a1 a2 . . . an + 1) ≥ (1 + a1 ) (1 + a2 ) . . . (1 +

2.2.7 Dado un conjunto de 51 enteros entre 1 y 100 (inclusive), mostrar


que al menos un elemento de el conjunto debe dividir a otro elemento de el
conjunto. (Sugerencia: probar en forma más general, que la misma propiedad
se cumple cuando n + 1 enteros son elegimos de los enteros entre 1 y 2n
(inclusive).) Para una prueba no inductiva ver 2.6.1.

2.2.8 Critique la prueba dada abajo para el siguiente teorema: Una matriz
n × n de enteros no negativos tiene la propiedad de que para cualquier ele-
mento que sea cero, la suma de la fila mas la columna que contienen a dicho
elemento es al menos n. Mostrar que la suma de todos los elementos de la
matriz es al menos n2 /2.

Prueba ( ?): El resultado se cumple para n = 1. Suponiendo que el resultado


se cumple para n = k −1, considere una matriz k por k. Si no tiene elementos
iguales a cero, el resultado se cumple obviamente. Si aij = 0, la suma de la fila
i y la columna j es al menos k, por hipótesis, y la suma de los elementos en la
submatriz (k − 1) × (k − 1) obtenida al suprimir la fila i y la columna j es al
menos (k − 1)2 /2 (por hipótesis inductiva). Se sigue la suma de los elementos
en la matriz k × k es al menos (k − 1)2 /2 + k = (k 2 − 2k + 1) /2 + k =
(k 2 + 1) /2 ≥ k 2 /2. El resultado se sigue por inducción.

Ejemplos adicionales.

1.1.11, 1.12.2, 3.1.11, 4.2.21, 4.3.5, 4.3.24, 6.5.12, 6.6.1, 7.1.6, 7.1.13, 7.2.5,
7.3.5.

2.3. Inducción Forma Fuerte

Suponga que a es un entero y P (n) una proposición acerca de n para todo


entero n ≥ a. La forma fuerte de inducción matemática estable que:

si
2.3. INDUCCIÓN FORMA FUERTE 91

(i) P (a) es verdadero,

(ii) para cada entero k ≥ a, P (a) , P (a + 1) , . . . , P (k) verdaderos implica


P (k + 1) sea verdadero,

Entonces P (n) es verdadero para todo n ≥ a.

La diferencia de la forma de inducción anterior esta en que ahora esta-


mos dando una suposición fuerte en el paso (ii), a saber, podemos suponer
P (a) , P (a + 1) , . . . , P (k) , en lugar solamente de P (k), para probar P (k + 1) .
Teóricamente, las dos formas de inducción son equivalentes, pero en la prácti-
ca hay problemas los cuales son mas fáciles de resolver con esta inducción
fuerte.

2.3.1 (Teorema de Pick) Probar que el área de un polı́gono retı́cula sim-


ple (un polı́gono con puntos reticulares como vértices cuyos lados no se
cruzan) es dado por I + 12 B − 1, donde I y B denotan respectivamente
el número de puntos interiores y puntos reticulares frontera del polı́gono.

Solución. Nosotros usaremos inducción sobre el número de lados del polı́gono.


El caso de un triángulo es dado en 1.7.3. Considere, entonces, un polı́gono
reticula simple P con k lados, k > 3. polı́gono reticula simple P con k lados,
k > 3. Primero establecemos que un tal polı́gono tiene una diagonal interior.
Esto es claro si el polı́gono es convexo (equivalente, si todos

Ası́ que supóngase el ángulo interior de algún vértice, llamado V, con mas de
1800 . Entonces un rayo que nace en V y barre el interior del polı́gono debe
tocar otro vértice (de no ser ası́ el polı́gono encierra un área infinita), y esto
determina una diagonal interior D con V como un punto final.

Supóngase que nuestro polı́gono P tiene I puntos interiores y B puntos fron-


tera. La diagonal interior D divide a P en dos polı́gonos retı́culas simples
P1 y P2 con I1 y I2 puntos interiores respectivamente, también con B1 y
B2 puntos frontera respectivamente. Supóngase que hay x puntos reticulares
sobre D, excluyendo sus puntos extremos. Entonces B = B1 + B2 − 2 − 2x,
I = I1 + I2 + x.

Ahora, si A, A1 , A2 denotan las áreas de P, P1 y P2 respectivamente. Entonces,


92CAPÍTULO 2. DOS IMPORTANTES PRINCIPIOS: EL DE INDUCCCIÓN Y EL DEL P

A = A1 + A2
   
1 1
= I1 + B1 − 1 + I2 + B2 − 1
2 2
1
= (I1 + I2 ) + (B1 + B2 ) − 2
2
1
= (I1 + I2 + x) + (B1 + B2 − 2x) − 2
2
1
= I + (B + 2) − 2
2
1
= I + B − 1.
2
El resultado se sigue por inducción.

Observe en este ejemplo que es el primer paso del argumento de inducción el


mas difı́cil (realizado en 1.7.3); el paso inductivo (paso (ii) ) es concepcional-
mente mas simple.

Problemas

2.3.2 .

(a) Probar que cualquier entero positivo y mayor que uno puede ser escrito
como un producto de números primos.

(b) El postulado de Bertrand’s, alguna vez postulado pero ahora un cono-


cido teorema, establece que para cualquier número x > 1. alli existe un
número primo entre x y 2x. Use este hecho para mostrar que cualquier
entero positivo puede ser escrito como una suma de distintos primos.
(Para esta prueba suponga que uno es un primo).

2.3.3 .

(a) Mostrar que cualquier entero positivo puede ser escrito como una suma
de distintos números de Fibonacci.
2.4. INDUCCIÓN Y GENERALIZACIÓN 93

(b) Suponga que k ≫ m significando esto k ≥ m+ 2. Mostrar que cualquier


entero positivo n tiene una representación de la forma n = Fk1 + Fk2 +
. . . + Fkr , donde Fki son números de Fibonacci y k1 ≫ k2 ≫ . . . ≫
kr ≫ 0.

(c) Mostrar que la representación en la parte (b) es única.

Ejemplos adicionales

3.1.1, 3.1.2, 3.1.18, 3.5.5, 6.2.3.

2.4. Inducción y Generalización

Hemos visto (en la sección 1.12) que un problema es algunas veces más fácil de
manejar cuando se reconsidera en forma mas general. Esto es verdadero tam-
bién en problemas de inducción. Por ejemplo, puede suceder que las proposi-
ciones originales P (1) , P (2) , P (3) , . . . , no contengan suficiente información
que permitan a uno llevar a cabo el paso inductivo (paso (ii) ). En este caso
es natural reformular las proposiciones en una forma más consistente, más
generalmente Q (1) , Q (2) , . . . (donde Q (n) implica P (n) para cada n), y
mirando nuevamente hacia una prueba inductiva.

2.4.1 Si A1 + · · · + An = π, 0 < Ai ≤ π, i = 1, . . . , n., entonces

π
senA1 + · · · + senAn ≤ nsen .
n
94CAPÍTULO 2. DOS IMPORTANTES PRINCIPIOS: EL DE INDUCCCIÓN Y EL DEL P

b
Pi
b

Ai b
Pi+1
b

Figura 2.2

Solución. Suponga que P (k) es la proposición de el teorema para un k


dado y suponiendo que P (k) es verdadera. Para el paso inductivo, suponga
que A1 + A2 + · · · Ak + Ak+1 = π, 0 < Ai ≤ π, i = 1, . . . , k + 1. En esta forma,
no es claro como hacer uso de P (k) ., sin embargo podemos, por ejemplo,
agrupar Ak y Ak+1 juntos, ası́ que A1 + .....+ Ak−1 + (Ak + Ak+1 ) = π y
aplicando la suposición inductiva obtenemos

π
sen A1 + · · · + sen Ak−1 + sen (Ak + Ak+1 ) ≤ k sen .
k
Pero todavı́a no vemos claro que esto implique P (k + 1) :
π
sen A1 + · · · + sen Ak + sen Ak+1 ≤ (k + 1) sen .
k+1

El requisito de que las A′i s sumen π parece muy restrictivo. Considere in-
mediatamente la siguiente proposición Q (n) :

Si 0 < Ai ≤ π, i = 1, . . . , entonces

 
A1 + · · · + An
sen A1 + · · · + sen An ≤ n sen .
n

(Observe que Q (n) implica P (n) . ) Obviamente Q (1) es verdadera. Supóngase


que Q (k) es verdadera, también que 0 < Ai ≤ π, , i = 1, . . . , k + 1. Entonces
2.4. INDUCCIÓN Y GENERALIZACIÓN 95

sen A1 + · · · + sen Ak + sen Ak+1


 
A1 + · · · + Ak
≤ k sen + sen Ak+1
k
   
k A1 + · · · + Ak 1
= (k + 1) sen + sen Ak+1
k+1 k k+1
  
k 1
≤ (k + 1) sen (A1 + · · · + Ak ) + sen Ak+1
k+1 k+1
 
A1 + · · · + Ak+1
= (k + 1) sen .
k+1

(la desigualdad se mantiene en sentido favorable, en estos pasos gracias al


resultado de 1.2.12b).)

El resultado se sigue ahora por inducción.

Ahora ya podemos probar la conjetura creada en 1.6.2 e): El polı́gono de


mayor área que puede ser inscrito en un cı́rculo es el polı́gono regular, para
verificar esto, suponga que P1 , P2 , . . . , Pn , n ≥ 3, son los vertices sucesivos de
un polı́gono inscrito (inscrito en un cı́rculo de radio r). Suponga que O denota
el centro del cı́rculo; suponga que Ti denota el área del triángulo P1 OPi+1 ,
i = 1, . . . , n (establecemos que Pn+1 = P1 ); Sea Ai = ∠Pi OPi+1 (figura 2.2).
Entonces

   
1 1 1
Ti = 2 r cos Ai r sen Ai
2 2 2
1 1
= r 2 cos Ai sen Ai
2 2
1 2
= r sen Ai .
2

El polı́gono de área máxima debe satisfacer 0 < Ai < π para cada i. nuestro
resultado precedente muestra que
96CAPÍTULO 2. DOS IMPORTANTES PRINCIPIOS: EL DE INDUCCCIÓN Y EL DEL P

n
X
Area del polı́gono = Ti
i=1
n n
X 1 1 X
= r sen Ai = r 2
2
sen Ai
i=1
2 2 i=1
" n #
n 2 1X
≤ r sen Ai
2 n i=1
  
1 2 2π
= n r sen .
2 n

El lado derecho representa el área de un n − ágono regular, esto completa la


prueba.

1
2.4.2 Sea f (x) = (x2 − 1) 2 , x > 1; probar que f (n) (x) > 0 para n impar y
f (n) (x) < 0 para n par.

Solución. Podemos suponer que se puede expresar f (k+1) (x) en términos


de f (k) (x) . Pero una mirada a las primeras derivadas hace que este plan
paresca imposible:

x 1
f ′ (x) = (x2 −1)1/2
, f ′′ (x) = − (x2 −1) 3/2 ,

3x 2
f ′′′ (x) = (x2 −1) 5/2 , f (iv) (x) = − (x12x +1
2 −1)7/2
,
60x3 +31x 4 2
(v)
f (x) = (x2 −1)9/2 , f (vi) (x) = − 522x(x2+266x +31
−1)11/2
.

1/2
Considerese inmediatamente la siguiente reformulación: si f (x) = (x2 − 1) ,
x > 1, entonces

gn (x)
f n (x) = ,
(x2 − 1)(2n−1)/2
donde gn (x) es un polinomio de grado n − 2, además
2.4. INDUCCIÓN Y GENERALIZACIÓN 97



 una función impar cuyos coeficientes todos

son no negativos si n es impar,
gn (x) es

 una función par cuyos coeficientes todos

son no positivos si n es par.

Esta proposición puede ser establecida por inducción (omitimos los confusos
detalles), y esto implica el resultado original.

2.4.3 Suponga que Fi denota el i-ésimo término en la suseción de fibonacci.


2
Probar que Fn+1 + Fn2 = F2n+1 .

Solución. La fórmula vale para n = 1, también suponiendo que la fórmula


vale para el entero k > 1, entonces

2 2
Fk+2 + Fk+1 = (Fk+1 + Fk )2 + Fk+1
2
2
= Fk+1 + 2Fk+1 Fk + Fk2 + Fk+1
2
2 2 2
= (Fk+1 + Fk ) + (2Fk+1Fk + Fk+1 )
2
= F2k+1 + (2Fk+1Fk + Fk+1 ),

El último paso uso la hipótesis inductiva.


2
Habremos terminado si podemos mostrar que 2Fk+1 Fk + Fk+1 = F2k+2 , podi-
2
endo entonces continuar los argumentos previos, F2k+1 + (2Fk+1Fk + Fk+1 )=
F2k+1 + F2k+2 = F2k+3 , y esto completa el paso inductivo. Por lo tanto, que-
2
da por probar que 2Fk+1 Fk + Fk+1 = F2k+2 . Procederemos por inducción. La
fórmula es cierta si n = 1, suponiendo que es cierta para n = k, entonces
tenemos:

2
2Fk+2 Fk+1 + Fk+2
2
= 2(Fk+1 + Fk )Fk+1 + Fk+2
2 2
= 2Fk+1 + 2Fk+1 Fk + Fk+2
2 2
= (2Fk+1 Fk + Fk+1 ) + (Fk+1 + Fk+2 )
2 2
= F2k+2 + (Fk+1 + Fk+2 ).

2
Pero ahora estamos de vuelta con el problema anterior : Cómo hacer Fk+2 +
2 2 2
Fk+1 = F2k+3 ?. Si es ası́, entonces F2k+2 + (Fk+1 + Fk+2 ) = F2k+2 + F2k+3 =
98CAPÍTULO 2. DOS IMPORTANTES PRINCIPIOS: EL DE INDUCCCIÓN Y EL DEL P

F2k+4 y la inducción es completa. De esta manera, los problemas estan in-


terrelacionados: la verdad de lo primero depende finalmente de la verdad de
lo segundo, y a la inversa, la verdad de lo segundo depende finalmente de la
verdad de lo primero.

Podemos resolver la dificultad con una prueba de las dos en la siguiente


forma. Considere las dos proposiciones

2
P (n) : Fn+1 + Fn2 = F2n+1 ,
2
Q(n) : 2Fn+1 Fn + Fn+1 = F2n+2 .

P (1) y Q(1) son cada una verdadera, los argumentos previos muestran que
P (k) y Q(k) implican P (k + 1), ahora P (k + 1) y Q(k) implica Q(k + 1).
De ello se sigue que P (k) y Q(k) implican P (k + 1) y Q(k + 1), la prueba se
completa ası́.

2.4.4 Sea f (x) = a1 senx + a2 sen2x + .... + an sennx, donde a1 , ....an son
números reales y donde n es un entero positivo. Suponga que |f (x)| < |senx|
para todo x real, probar que |a1 + 2a2 + .... + nan | ≤ 1.

Solución. Supóngase que intentamos usar inducción sobre el número de


términos en f (x). Cuando n = 1, f (x)
 = a1 senx
 y de la hipótesis
 |f (x)| ≤
|senx|, se sigue que |a1 | = a1 sen π2 = f π2 ≤ sen π2 = 1

Supóngase que el resultado se cumple para k, y considere la función


f (x) = a1 senx + a2 sen2x + .... + ak senkx + ak+1 sen(k + 1)x,

Para alguna elección de números reales a1 , a2 , ..., ak+1 , supóngase que |f (x)| ≤
|senx| para todo x real. Como sen(k + 1)x = senkxcosx + senxcoskx, pode-
mos escribir

f (x) = (a1 + ak+1 coskx)senx + a2 sen2x + ....


+ak−1 sen(k − 1)x + (ak + ak+1 cosx)senkx.

Tenemos ahora reescrito f (x) como una suma de k térmionos, más o menos
del mismo tipo con lo cual ya podemos aplicar la hipótesis de inducción. La
2.4. INDUCCIÓN Y GENERALIZACIÓN 99

dificultad es que los coeficientes de los términos seno en esta expresión son
no constantes; la razón es que algunos de ellos contiene funciones de x. Esto
sugiere considerar el siguiente problema más general.

Sean a1 (x), .....an (x) funciones diferenciables de x, y sea f (x) = a1 (x)senx +


+a2 (x)sen2x + .... + an (x)sennx. Suponga que |f (x)| ≤ |senx| para todo x
real, probar que |a1 (0) + 2a2 (0) + ..... + nan (0)| ≤ 1.

Si podemos probar esta proposición, tendremos resuelto también el problema


original, porque, tomando ai (x) ≡ ai , ai una constante, i = 1, 2, ....., n, para
todo x, cubrimos el problema original.

Nuevamente procederemos por inducción. Estamos suponiendo |a1 (x)senx| ≤


|senx|. Mientras x se aproxima a 0, senx 6= 0, para tales x |a1 (x)| ≤ 1. Ahora
a1 (x) es continua en x = 0, entonces |a1 (0)| ≤ 1. Esto implica que el resultado
se cumple para el caso n = 1.

Ahora supóngase que el resultado se cumple para n = k, y considere la


función

f (x) = a1 (x)senx + a2 (x)sen2x + .... + ak+1 (x)sen(k + 1)x,

donde |f (x)| < |senx| con las ai (x) diferenciables. Como antes, esta puede
ser reescrita en la forma equivalente
f (x) = [a1 (x) − ak+1 (x)coskx] senx + a2 (x)sen2x + .....
+ak−1 (x)(k − 1)x + [ak (x) + ak+1 (x)cosx] senkx

Podemos ahora aplicar la hipótesis inductiva, y cocluir que

|[a1 (0) + ak+1 (0)] + 2a2 (0) + ..... + (k − 1)ak−1 (0) + k [ak (0) + ak+1 (0)]| ≤ 1

pero esto es lo mismo que

|a1 (0) + 2a2 (0) + ..... + kak (0) + (k + 1)ak+1(0)| ≤ 1


100CAPÍTULO 2. DOS IMPORTANTES PRINCIPIOS: EL DE INDUCCCIÓN Y EL DEL

que es la forma deseada (una prueba no inductiva es dada en 6.3.2)

Problemas

2.4.5 Suponga que S denota un retı́culo cuadrado n × n, n ≥ 3. Mostrar


que es posible dibujar un camino poligonal consistente de 2n − 2 segmentos
los cuales pasan a traves de todos los n puntos reticulares de S.

1
2.4.6 Sea f0 (x) = , y defina fn+1 (x) = xfn′ (x). Probar que fn+1 (x) >
(1 − x)
0 para 0 < x < 1.
2.5. RECURRENCIA 101

2.5. Recurrencia

En la segunda solución de 1.1.1, suponga que An denota el número de subcon-


juntos de un conjunto con n elementos. Hemos mostrado que An+1 = 2An ,
A0 = 1. Este es un ejemplo de relación recurrente. Aunque no tengamos
una fórmula explı́cita para An (como el método de inducción requiere), la
relación de recurrencia define una ”espiral ” o algoritmo el cual nos muestra
como calcular An+1 . En esta sección veremos acerca de problemas que pueden
ser reducidos a problemas equivalentes con parámetros pequeños, la idea es
aplicar la reducción con argumentos recursivos hasta que los parámetros al-
canzan valores para los cuales el problema puede ser resuelto.

2.5.1 (Problema de la Torre de Hanoi). Supóngase n anillos, con diferentes


diámetros (en su circunferencia exterior), son pasados sobre una estaca verti-
cal, el más grande en el fondo, en forma de piramide (figura 2.3). Otras dos
estacas verticales son colocadas suficientemente lejos. Deseamos transferir
todos los anillos, uno en una vez, a la segunda estaca para formar una pi-
ramide idéntica. Durante las transferencias, no estamos permitidos a colocar
un anillo sobre otro menor (esto hace necesario usar el tercer anillo). Cual es
el menor número de movimientos necesarios para completar la transferencia?

Solución. Suponga que Mn denota el mı́nimo número de movimientos para


una pila de n anillos. Claramente M1 = 1, ahora suponga n > 1. En orden
hay que conseguir pasar el anillo más grande al fondo de la segunda estaca,
para ello hay que mover los restantes n−1 anillos a la tercera estaca. Esto nos
toma un mı́nimo de Mn−1 movimientos (según nuestra notación elegida). Un
movimiento es necesario para transferir el mayor anillo a la segunda estaca,
finalmente Mn−1 movimientos son necesarios para transferir los restantes n−1
anillos a la segunda estaca. De esta manera

Mn = 2Mn−1 + 1 M1 = 1.

Un proceso de inducción fácil, basado en esta recurrencia, muestra que Mn =


2n − 1,

(Mn+1 = 2Mn + 1 = 2 [2n − 1] + 1 = 2n+1 − 1).


102CAPÍTULO 2. DOS IMPORTANTES PRINCIPIOS: EL DE INDUCCCIÓN Y EL DEL

Figura 2.3.

Suponga que a1 , a2 , ....., an es una permutación de 1, 2, ...., n. Podemos inter-


pretar esta permutación geométrica en la siguiente forma. Tome un tablero
de ajedrez n × n, ahora para cada i, coloque una torre en la i-ésima colum-
na (partiendo de la izquierda) y la ai -ésima fila (partiendo de abajo). Por
ejemplo, la permutación 3, 2, 5, 4, 1 esta representada en la figura 2.4. En
esta manera vemos que una permutación de 1, 2, ...., n corresponde a una
colección de n torres ”inofensivas” en un tablero de ajedrez n × n. Esta cor-
respondencia permite a uno pensar las permutaciones geométricas y utilizar
el lenguaje e imagenes de torres inofensivas en un tablero de ajedrez.
5 R

4 R

3 R

2 R

1 R
1 2 3 4 5
Figura 2.4

2.5.2 Suponga que Qn denota el número de maneras de colocar n torres


inofensivas en un tablero de n × n tal que que el arreglo es simétrico respecto
a la diagonal que va de la esquina inferior izquierda a la esquina supeior
2.5. RECURRENCIA 103

derecha. Mostrar que

Qn = Qn−1 + (n − 1)Qn−2 .

Solución. Una torre en la primera columna puede o no ocupar el cuadro en


la esquina inferior izquierda del tablero. Si ello ocurre, hay Qn−1 maneras de
colocar las restantes n − 1 torres. Si ello no ocurre, una torre puede ocupar
cualquiera de los n − 1 cuadros en la primera columna. Una vez que es
colocada, se determina de manera única la colocación de una torre puesta
simétricamente (simétrica con respecto a la diagonal dada) en la primera fila.
Las restantes n − 2 torres pueden ser colocadas en Qn−2 maneras. Poniendo
esas ideas juntas da el resultado.

2.5.3 Una moneda es lanzada n veces. Cuál es la probabilidad de que dos


caras aparezcan una tras otra en algún momento en la secuencia de tiros?

Solución. Suponga que Pn denota la probabilidad de que no aparezcan dos


3
caras consecutivas en n lanzamientos. Claramente P1 = 1, P2 = . Si n > 2,
4
allı́ hay dos casos.

Si el primer lanzamiento es cruz, entonces dos caras consecutivas no apare-


ceran en los restantes n − 1 lanzamientos con probabilidad Pn−1 (de acuerdo
a nuestra notación). Si el primer lanzamiento es cara, el segundo lanzamiento
debe ser cruz para evitar dos caras consecutivas, entonces dos caras consecu-
tivas no aparecerán en los restantes n − 2 con probabilidad Pn−2 . Ası́ de esa
manera,

1 1
Pn = Pn−1 + Pn−2 , n > 2.
2 4

Esta recurrencia puede ser transformada a una forma más familiar multipli-
cando cada lado por 2n :

2n Pn = 2n−1 Pn−1 + 2n−2Pn−2


104CAPÍTULO 2. DOS IMPORTANTES PRINCIPIOS: EL DE INDUCCCIÓN Y EL DEL

y poniendo Sn = 2n Pn para cada n:

Sn = Sn−1 + Sn−2 .

Esta es la recurrencia para la suseción de Fibonacci (observe que Sn = Fn+2 ).


Ası́ pues, la probabilidad que buscamos es Qn = 1 − Pn = 1 − Fn+2 /2n .

Problemas

2.5.4 Suponga que Pn denota el número de regiones formadas cuando n


lı́neas son dibujadas en el plano Euclidiano de tal manera que no hay tres que
son concurrentes ni dos que son paralelas. Mostrar que Pn+1 = Pn + (n + 1).

2.5.5 .

Suponga que En denota el determinante de una matriz n × n teniendo −1’s


abajo de la diagonal principal (que va de la izquierda arriba a la derecha
abajo) y 1’s en y sobre la diagonal principal. Mostrar que E1 = 1 y que
E2 = 2En−1 para n > 1.

Suponga que Dn denota el determinante de otra matriz n × n cuyo (i, j)-


ésimo elemento (el elemento en la i-ésima fila y la j-ésima columna) es el
valor absoluto de la diferencia de i y j. Mostrar que Dn = (−1)n−1 (n−1)2n−2 .

Suponga que Fn denota el determinante de otra matriz n × n con a en la


diagonal principal, b en la superdiagonal (la diagonal inmediata abajo de la
diagonal principal teniendo n − 1 elementos). Mostrar que Fn = aFn−1 −
bcFn−2 , n > 2. Que pasa cuando a = b = 1 y c = −1?

Evalúe el determinante An n×n, cuyo (i, j)-ésimo elemento es a|i−j| hallando


una relación recursiva entre An y An−1 .

2.5.6 .
2.5. RECURRENCIA 105

Suponga que a1 , a2 , ...., an son números reales positivos y An = (a1 + .... +


1
(n−1)/n
an )/n. Mostrar que An ≥ An−1 ann con la igualdad solo si An−1 = an .(Sugerencia:
aplique la desigualdad de 2.1.5)

La desigualdad entre la media geométrica y la media aritmética. Usando la


parte (a), mostrar que

a1 + ...... + an
≥ (a1 ....an )1/n
n
con la igualdad solo si a1 = a2 = .....an .

2.5.7 Dos jugadores de ping-pong, A y B, acuerdan varios juegos. Los ju-


gadores son igualmente capaces, supóngase, que quien quiera que juegos. Los
jugadores son igualmente capaces, supóngase, que quien quiera puede ser el
jugador A en un juego, o el jugador B en otro). Supongase que A sirve
primero en el primer juego, pero después el perdedor sirve primero. Suponga
que Pn denota la probabilidad de que A gane el n-ésimo juego. Mostrar que

Pn+1 = Pn (1 − P ) + (1 − Pn )Ṗ .

2.5.8 Un estudiante juega a lanzar una moneda equitativa y alcanza un


punto para cada cara que aparece y dos puntos para cada cruz. Probar que
la probabilidad de que el estudiante alcance exactamente n puntos en algún
1 n 
momento en una suseción de n lanzamientos es 2 + − 21 . (Sugerencia:
3
suponga que Pn denota la probabilidad de alcanzar exactamente n puntos en
algún momento. Exprese Pn en términos de Pn−1 y Pn−2 . Use esta relación
recurrente para dar una prueba inductiva).

2.5.9 (Problema de Josephus). Arregle los números 1, 2, ..., n consecutiva-


mente (en el sentido de las agujas del reloj) sobre la circunferencia de un
cı́rculo. Ahora, remueva el número 2 y proceda en el sentido de las agujas
del reloj removiendo cualquier otro número, entre aquellos que quedan, hasta
que solo un número queda. (ası́, para n = 5, los números son removidos en el
106CAPÍTULO 2. DOS IMPORTANTES PRINCIPIOS: EL DE INDUCCCIÓN Y EL DEL

orden 2, 4, 1, 5 quedando solo el 3). Suponga que f (n) denota el número final
que queda. Mostrar que

f (2n) = 2f (n) − 1
f (2n + 1) = 2f (n) + 1

este problema continua en (3.4.5).

2.5.10 .

(a) Suponga que Rn denota el número de maneras de colocar n torres in-


ofensivas en el tablero de ajedrez n × n, también que el arreglo es
simétrico con respecto a una rotación (en el sentido de la agujas del
reloj) de 900 de el tablero alrededor de el centro. Mostrar que

R4n = (4n − 2)R4n−4 ,


R4n+1 = R4n ,
R4n+2 = 0 = R4n+3 .

(b) Suponga que Sn denota el número de maneras de colocar n torres in-


ofensivas en un tablero de ajedrez n × n, también que el arreglo es
simétrico con respecto a el centro del tablero. Mostrar que

S2n = 2nS2n−2
S2n+1 = S2n .

(c) Suponga que Tn denota el número de caminos de colocar n torres in-


ofensivas en un tablero de ajedrez de n × n, también que el arreglo es
simétrico con respecto a las dos diagonales. Mostrar que

Tn = 2,
T2n+1 = T2n ,
T2n = 2T2n−2 + (2n − 2)T2n−4 .

2.5.11 Un (n + 2)-ágono regulares es inscrito en un cı́rculo. Suponga que


Tn denota el número de formas posibles para juntar sus vértices en pares tal
2.5. RECURRENCIA 107

que los segmentos resultantes no se intersectan uno con otro. Si establecemos


T0 = 1, mostrar que

Tn = T0 Tn−1 + T1 Tn−2 + T2 Tn−3 + ...... + Tn−1 T0

(para una continuación de este problema, ver 5.4.10)

2.5.12 Sean a1 , a2 , ....., an una permutación de el conjunto Sn = {1, 2, ...., n}.


Un elemento i en Sn es llamado un punto fijo de esta permutación si ai = i.

a. Un desarreglo de Sn es una permutación de Sn sin puntos fijos. Sea gn


el número de desarreglos de Sn . Mostar que

g1 = 0, g2 = 1,
gn = (n − 1)(gn−1 + gn−2 ), Para n > 2.

(Sugerencia: Un desarreglo, o intercambia el primer elemento con otro


o no lo es)

b. Sea fn el número de permutaciones de Sn con exactamente un punto


fijo. Mostar que |fn − gn | = 1.

2.5.13 Supóngase que n hombres entregan sus sombreros al llegar a un


banquete, como los dejan, los sombreros les son de vueltos tomandolos al
azar. Cuál es la probabilidad de que un hombre no consiga en esta devolución
su propio sombrero? (Sugerencia: Suponga que Pn denota esta probabilidad.
Entonces Pn = gn /n!, donde gn es como en 2.5.13. Sea Cn = Pn − Pn−1 . Use
la relación de recurrencia hallando en 2.5.13 (a) para mostrar que C2 = 1/2,
Cn = −Cn−1 /n. Use esto para mostrar que Pn = 1/2! − 1/3! + .... + (−1)n /n!.
Entonces para n grande Pn ≈ 1/e)

2.5.14 .

R π/2
(a) Sea In = 0
senn xdx. Hallar una relación de recurrencia para In .
108CAPÍTULO 2. DOS IMPORTANTES PRINCIPIOS: EL DE INDUCCCIÓN Y EL DEL

(b) Mostrar que


1 × 3 × 5 × ..... × (2n − 1) π
I2n = .
2 × 4 × 6 × ....... × 2n 2
(c) Mostrar que
2 × 4 × 6 × ..... × (2n − 2)
I2n+1 = .
1 × 3 × 5 × ....... × (2n − 1)

Ejemplos adicionales

1.1.1(Solución 2), 4.3.9, 5.3.5, 5.3.14, 5.3.15, 5.4.8, 5.4.9, 5.4.24, 5.4.25, 5.4.26.
Estrechamente relacionadas la inducción y la recursión son argumentos basa-
dos sobre ”argumentos repetidos”. Ejemplos de que eso ha significado aquı́ son
4.4.4, 4.4.17, la prueba de el teorema del valor medio en 6.1, 6.1.5, 6.1.6,
6.3.6, 6.8.10, y la heurı́stica para el significado geométrico de una desigual-
dad arimética dada en la sección 7.2

2.6. Principio del Palomar

Cuando una colección suficientemente grande de objetos dividida en un número


suficientemente pequeño de clases, una de las clases contendrá un ciertamente
mı́nimo número de objetos. Este hecho se preciso más en la siguiente proposi-
ción evidente:

Principio del palomar. Si kn + 1 objetos (k ≥ 1) son distribuidos entre n


cajas una de las cajas va a contener al menos k + 1 objetos

Este principio, incluso cuando k = 1, es una muy poderosa herramienta para


probar teoremas de existencia.

2.6.1 Dado un conjunto de n + 1 enteros positivos, ninguno de los cuales


excede a 2n, mostrar que al menos un miembro de el conjunto debe dividir a
otro miembro de el conjunto.

Solución. Este es el mismo que el de 2.2.7, donde estaba planteado por


inducción sobre n. Sin embargo, el problema es realmente un problema de
2.6. PRINCIPIO DEL PALOMAR 109

existencia para un n dado, y puede ser llevado muy atinadamente hacia el


principio del palomar, como veremos en seguida.

Suponga que los números elegidos se denota por x1 , x2 , ....., xn+1 , ahora para
cada i escribimos xi = 2ni yi , donde ni es entero no negativo y yi es impar.
Sea T = {yi : 1, 2, ....., n + 1}. Entonces T es una colección de n + 1 enteros
impares, cada uno de ellos menor que 2n. De aquı́ que allı́ hay solo n números
impares menores que 2n, el principio del palomar implica que dos núemros
en T son iguales, digamos yi = yj , i < j. Entonces

xi = 2ni yi y xj = 2nj yi .

Si ni ≤ nj , entonces xi divide a xj ; si ni > nj , entonces xj divide a xi . Esto


completa la prueba.

2.6.2 Considere cinco puntos cualesquiera P1 , P2 , P3, P4 , P5 en el interior de


un cuadrado S de lado con longitud 1. Denote por dij la distancia entre los
puntos
√ Pi y Pj . Probar que al menos una de las distancias dij es menor que
2/2.

Figura 2.5

Solución. Divida S en cuatro cuadrados congruentes como se muestra en


la figura 2.5. Por el principio de palomar, dos puntos pertenecen a uno de
esos cuadrados (un punto en la frontera de dos cuadrados pequeños puede ser
considerado
√ de ambos cuadrados). La distancia entre esos puntos es menor
que 2/2.
110CAPÍTULO 2. DOS IMPORTANTES PRINCIPIOS: EL DE INDUCCCIÓN Y EL DEL

Figura 2.6

2.6.3 Supóngase que cada cuadrado de un tablero de ajedrez 4 × 7, como


se muestra en la figura, es coloreado en blanco o negro. Probar que en una
tal coloración, el tablero debe contener un rectángulo (formado por las lı́neas
horizontales y verticales de el tablero), tal como el resaltado en la figura 2.6,
cuyos cuadrados de las distintas esquinas son todos del mismo color.

Figura 2.7

Solución. Un tal rectángulo existe incluso en un tablero 3×7. El color de las


configuraciones de cada columna debe de ser de uno de los tipos mostrados
en la figura 2.7. Suponga que una de las columnas es del tipo 1. Habremos
terminado si una de las 6 restantes es del tipo1, 2, 3 o 4. Suponga pues que
cada una de las seis columnas restantes es del tipo 5, 6, 7 u 8. Entonces por
el principio del palomar , dos de esas seis columnas deben ser del mismo tipo
y habremos terminado.

El mismo razonamiento se hace si una de las columnas es del tipo 8.

Ahora suponga que ninguna de las columnas es del tipo 1 o del tipo 8.
Entonces tenemos siete columnas pero solo seis tipos. Por el principio del
palomar, dos columnas deben tener el mismo tipo y la prueba es terminada.
2.6. PRINCIPIO DEL PALOMAR 111

2.6.4 Probar que existen tres enteros a, b, c no todos cero y cada uno con
valor absoluto menor que un millon, tales que

√ √
a + b 2 + c 3 < 10−11 .

√ √
Solución. Sea S el conjunto de 1018 números reales √ r + s √2 + t 3 con r, s, t
cada uno en {0, 1, 2, ...., 106 − 1}, ahora sea d = (1 + 2 + 3)106 . Entonces
cada x en S está en el intervalo 0 ≤ x < d. Dividiendo este intervalo en
1018 − 1 subintervalos iguales, cada uno de longitud e = d/(1018 − 1). Por el
18
principio del palomar, dos de los 10 √ números
√ de S deben estar en el mismo
subintervalo. Su diferencia, a + b 2 + c 3, dá los deseados a, b, c. ya que
e < 107 /1018 = 10−11 .

2.6.5 Dado un conjunto de diez números naturales entre 1 y 99 inclusive


(notación decimal), probar que allı́ existen dos subconjuntos no vacios y dis-
juntos, del conjunto con sumas iguales de sus elementos.

Solución. Con la elección del conjunto de diez números, podemos formar


210 − 1 = 1023 (diferentes) subconjuntos no vacios. Cada uno de esos subco-
juntos tiene una suma menor que 1000, ya que incluso 90+91+. . .+99 < 1000.
Por tanto por el principio del palomar, dos subconjuntos A y B deben tener
la misma suma. Ası́ por extracción de los elementos que pertenescan a ambos
conjuntos obtenemos dos conjuntos disjuntos x = A−A∩B y y = B −A∩B,
con la misma suma. (Ni x ni y son vacios, porque esto significarı́a que: A ⊂ B
o B ⊂ A, lo cual es imposible, ya que la suma de sus elementos da el mismo
número.)

Problemas

2.6.6 Sea A conjunto de 20 enteros distintos elegidos de la progresión ar-


itmética 1, 4, 7, . . . , 100. Probar que existen dos enteros distintos en A cuya
suma es 104.

2.6.7 .
112CAPÍTULO 2. DOS IMPORTANTES PRINCIPIOS: EL DE INDUCCCIÓN Y EL DEL

a. Sea S una región cuadrada (en el plano) de lado con longitud 2 pulgadas.
Mostrar que entre cualesquiera nueve puntos en S, hay tres los cuales son los
vértices de un triángulo de área ≤ 1/2 de una pulgada cuadrada.

b. Diecinueve dardos son lanzados hacia un blanco el cual tiene la forma de


un hexágono regular con lado de longitud
√ un pie. Mostrar que dos dardos
están a una distancia de a lo más 3/3 pies uno de otro.

2.6.8 Mostrar que si hay n personas en una fiesta, entonces dos de ellas
conocen el mismo número de personas (entre aquellas presentes).

2.6.9 Quince sillas son equitativamente colocadas alrededor de una mesa


circular en la cual estan asignadas fichas para quince invitados. Los invitados
ignoran la información de estas fichas hasta después que se han sentado,
y resulta que ninguno está sentado frente a su propia ficha. Probar que la
mesa puede ser rotada de manera que al menos dos de los invitados estén
simultaneamente sentados correctamente.

2.6.10 Sea X un número real. Probar que entre los números

x, 2x, . . . , (n − 1) x
hay uno que difiere de un entero por a lo mas 1/n.

2.6.11 .

(a.) Probar que en un grupo de seis personas hay tres que, o bien son mutu-
amente conocidas o bien son mutuamente desconocidas. (Sugerencia: Repre-
sente las personas con los vértices de un hexágono regular. Una dos vértices
con un segmento de lı́nea rojo si el par representado por esos vértices es de
conocidos de lo contrario una esos vértices con un segmento de lı́nea azul.
Considere uno de los vértices, llamemosle A. Al menos tres segmentos de
lı́nea parten de A teniendo el mismo color. Aqui hay dos casos a considerar.)
2.6. PRINCIPIO DEL PALOMAR 113

(b.) Diecisiete personas mantienen correspondencia entre si (cada uno con


todos los demás) por correo. En sus cartas solo tres temas son discutidos.
Cada par de correspondencias trata solo uno de los temas. Probar que hay al
menos tres personas que se escriben unos a otros sobre el mismo tema.

2.6.12 Probar que no existe un conjunto de siete enteros positivos, no may-


ores de 24, en el cual sus subconjuntos tienen todos sumas diferentes.

Ejemplos adicionales

1.10.1, 3.2.1, 3.2.5, 3.2.19, 3.2.20, 3.3.24, 4.4.10. [12pt]book [spanish]babel


[pctex32]graphicx epsfig amsmath pst-all amscd amsfonts pstricks,pstricks-
add,pst-math,pst-xkey
114CAPÍTULO 2. DOS IMPORTANTES PRINCIPIOS: EL DE INDUCCCIÓN Y EL DEL
Capı́tulo 3

Heurı́stica

La estrategia o la táctica para resolver problemas es llamada heurı́stica.


En este capı́tulo nos ocuparemos de la heurı́stica para resolver problemas
matemáticos. Aquellos que han pensado sobre la heurı́stica han descrito un
número de ideas básicas que son tı́picamente usuales. Los cinco clásicos de
la resolución de problemas de George Polya son obras maestras dedicadas
enteramente al estudio práctico de la heurı́stica en matemáticas. Entre las
ideas desarrolladas en estos libros, nos enfocaremos a las siguientes:

1. Búsqueda de un patrón.
2. Dibujo de una figura.
3. Formulación de un problema equivalente.
4. Modificación del problema.
5. Elección de una notación efectiva.
6. Explotación de la simetrı́a.
7. División en casos.
8. Trabajo hacia atrás.
9. Argumentación por contradicción.

115
116 CAPÍTULO 3. HEURÍSTICA

10. Búsqueda de paridad.

11. Consideración de casos extremos.

12. Generalización.

Nuestro interés en esta lista de ideas para resolver problemas no es describir-


las sino implementarlas. Al examinar ejemplos de cómo otros han utilizado
estas sencillas pero poderosas ideas, podemos esperar mejorar nuestras ha-
bilidades para resolver problemas.

Antes de comenzar, un consejo acerca de los problemas al final de las sec-


ciones: no se preocupe por aplicar la heurı́stica tratada en esa sección. Aunque
los problemas están escogidos para practicar el uso de la heurı́stica en cuestión,
un enfoque estrecho puede ser psicológicamente debilitante. Un mismo prob-
lema admite usualmente varias soluciones, a veces empleando diferentes heurı́sti-
cas. Ası́, es mejor atacar cada problema con la mente abierta que con una
noción preconcebida de cómo una heurı́stica en particular debiera ser apli-
cada. Al trabajar en un problema, resolverlo es lo importante. Es la ex-
periencia acumulada de todas las ideas trabajando simultáneamente lo que
terminará en una mejor comprensión de las posibilidades de resolución de un
problema.

3.1. Búsqueda de un Patrón

Virtualmente todos los que resuelven un problema comienzan su análisis


dándose una idea del problema, convenciéndose de la plausibilidad del resul-
tado. Esto se hace mejor examinando los casos especiales más inmediatos;
cuando esta exploración es llevada a cabo de manera sistemática, pueden
surgir patrones que sugieren ideas para proceder con el problema.

3.1.1 Demuestre que un conjunto con n elementos (diferentes) tiene exac-


tamente 2n subconjuntos (diferentes).
3.1. BÚSQUEDA DE UN PATRÓN 117

Cuando el problema está dado en este modo imperativo, un principiante


puede asustarse y no saber como proceder. Suponga, sin embargo, que el
problema fuera propuesto como una indagación, tal como:

(i) Cuántos subconjuntos pueden formarse de un conjunto de n elementos?

(ii) Demuestre o dé un contraejemplo: Un conjunto con n elementos tiene 2n


subconjuntos.

En cualquiera de estas formas ya está implı́cita la sugerencia de que de-


berá comenzar revisando unos cuantos casos particulares. Ası́ es como cada
problema debiera ser atacado: permanecer escéptico respecto al resultado
hasta no estar plenamente convencido.

Solución 1. Empezaremos por analizar lo que sucede cuando el conjunto


contiene 0, 1, 2, 3 elementos; los resultados se muestran en la siguiente tabla:

n elementos subconjuntos núm.de subconjuntos


0 ninguno ∅ 1
1 x1 ∅, {x1 } 2
2 x1 , x2 ∅, {x1 }, {x2 }, {x1 , x2 } 4
3 x1 , x2 , x3 {x3 }, {x1 , x3 }, {x2 , x3 }, {x1 , x2 , x3 }, ∅, {x1 }, {x2 }, {x1 , x2 } 8

Nuestro propósito al construir esta tabla no es sólo verificar el resultado, sino


también buscar patrones que pudieran sugerir cómo proceder en el caso gen-
eral. Ası́, nuestro objetivo es ser tan sistemático como sea posible. En este
caso, nótese que cuando n = 3, hemos enlistado primero los subconjuntos de
{x1 , x2 } y luego, en la segunda lı́nea, cada uno de los subconjuntos aumen-
tados por el elemento x3 . Esta es la idea clave que nos permite proseguir con
valores más grandes de n. Por ejemplo, cuando n = 4, los subconjuntos de
S = {x1 , x2 , x3 , x4 } son los ocho subconjuntos de {x1 , x2 , x3 } (mostrados en
la tabla) junto con los ocho formados al agregar x4 a cada uno de éstos. Estos
dieciseis subconjuntos construyen toda la colección de posibilidades; ası́, un
conjunto de 4 elementos tiene 24 (= 16) subconjuntos.

Una demostración basada en esta idea es una aplicación fácil de inducción


matemática (ver sección 2.1)

Solución 2. Otro camino para presentar la idea de la última solución es ar-


gumentar como sigue. Para n, sea An el número de subconjuntos (diferentes)
de un conjunto con n elementos. Sea S un conjunto con n + 1 elementos,
118 CAPÍTULO 3. HEURÍSTICA

y desı́gnese uno de sus elementos como x. Hay una correspondencia uno a


uno entre aquellos subconjuntos de S que no contienen a x (propiamente,
un subconjunto T del tipo anterior se corresponde con T ∪ {x} ). Los tipos
anteriores son todos subconjuntos de S − {x}, un conjunto con n elementos,
y ası́, debe darse el caso de que An+1 = 2An .

Esta relación de recurrencia, verdadera para n = 0, 1, 2, 3, ..., combinando


con el hecho A0 = 1, implica que An = 2n . (An = 2An−1 = 22 An−2 = ... =
2n A0 = 2n )

Solución 3. Otra enumeración sistemática de los subconjuntos puede ser


llevada a cabo construyendo un “árbol ” .

Cada rama del árbol corresponde a un subconjunto diferente de S (la barra


sobre el nombre del elemento significa que no está incluido en el subconjunto
correspondiente a esa rama). El árbol está construido en tres etapas, corre-
spondientes a los tres elementos de S. Cada elemento de S lleva a dos posi-
bilidades: está contenido en el subconjunto de S o no lo está, y estas opciones
están representadas por dos ramas. Cuando cada elemento es considerado,
el número de ramas se duplica. Ası́, para un conjunto de tres elementos el
número de ramas es 2×2×2 = 8. Para un conjunto de n elementos el número
de elementos es
n
2| × 2 ×
{z... × 2} = 2 ;
n veces

ası́, un conjunto con n elementos tiene 2n subconjuntos.

Solución 4. Supóngase que enumeramos los subconjuntos de acuerdo a su


tamaño. Por ejemplo, cuando S = {a, b, c, d}, los subconjuntos son:

núm. de elementos núm. de subconjuntos


0 ∅ 1
1 {a}, {b}, {c}, {d} 4
2 {a, b}, {a, c}, {a, d}, {b, c}, {b, d}, {c, d} 6
3 {a, b, c}, {a, b, d}, {a, c, d}, {b, c, d} 4
4 {a, b, c, d} 1

Este comienzo podrı́a propiciar el siguiente argumento. Sea S P


un conjunto
con n elementos. Entonces el número de subconjuntos de S = n0 (número
3.1. BÚSQUEDA DE UN PATRÓN 119

de subconjuntos de S con k elementos).


n  
X n
= = 2n
0
k

El paso final en esta cadena de igualdades se sigue del teorema del binomio,
n  
n
X n k n−k
(x + y) = x y ,
0
k

haciendo x = 1 y y = 1

Solución 5. Otro comienzo sistemático está ilustrado en la siguiente tabla,


donde se enlistan los subconjuntos de S = {x1 , x2 , x3 }. Para entender el pa-
trón aquı́, nótese la correspondencia entre los objetos de la columna extrema
izquierda y la ocurrencia de 1´s en la segunda columna de triadas

Subconjunto Triada Núm. binario Núm. decimal


∅ (0,0,0) 0 0
{x3 } (0,0,1) 1 1
{x2 } (0,1,0) 10 2
{x2 , x3 } (0,1,1) 11 3
{x1 } (1,0,0) 100 4
{x1 , x3 } (1,0,1) 101 5
{x1 , x2 } (1,1,0) 110 6
{x1 , x2 , x3 } (1,1,1) 111 7

Especı́ficamente, si A es un subconjunto de S = {x1 , x2 , ...., xn }, defı́nase ai ,


para i = 1, 2, ..., n, como n
ai = 10 sisi xxii ∈∈/ A
A

Es claro que ahora podemos identificar un subconjunto A de S , con (a1 , a2 , ..., an ),


una n-ada de 0’s y 1’s. Inversamente a cada n-upla le corresponde un único
subconjunto de S. Luego el número de subconjuntos de S es igual al número
de n-uplas de 0′s y 1′s. Este último conjunto está obviamente en corre-
spondencia uno-a-uno con el conjunto de los números binarios no negativos
120 CAPÍTULO 3. HEURÍSTICA

menores que 2n . Luego a cada entero no negativo menor que 2n le corresponde


exactamente un subconjunto de S, e inversamente. Por lo tanto, debe suceder
que S tenga 2n subconjuntos.

Normalmente, daremos una solución a cada ejemplo, una solución que sirva
para ilustrar la heurı́stica bajo consideración. En este primer ejemplo, sin
embargo, sencillamente querı́amos reiterar la observación anterior de que un
mismo problema pueda ser trabajado en una variedad de formas. La lección
que debe aprenderse es que uno debe permanecer flexible en las primeras
etapas de exploración de problemas. Si un acercamiento no parece llevar a
ninguna parte, no hay que desesperarse, sino hay que buscar una idea nueva.
No se quede fijo en una sola idea hasta que no haya tenido la oportunidad
de pensar ampliamente en una variedad de acercamientos alternativos.

3.1.2 Denótese por Sn,0 , Sn,1, Sn,2 a la suma de cada tercer elemento en el
n-ésimo renglón del triángulo de pascal, comenzando por la izquierda con el
primer elemento, el segundo elemento, y el tercer elemento respectivamente.
Haga una conjetura sobre el valor de S100,1

Triángulo de Pascal n Sn,0 Sn,1 Sn,2


1 0 1+ 0 0
11 1 1 1 0−
121 2 1 2+ 1

1331 3 2 3 3
14641 4 5 5 6+
1 5 10 10 5 1 5 11 10− 11
1 6 15 20 15 61 6 22+ 21 21
1 7 21 35 35 21 7 1 7 43 43 42−

Solución. Empezaremos por examinar las casillas de orden inferior con la


esperanza de encontrar patrones que pudieran generalizarse. En la tabla 1.2,
los términos que están subrayados con una sola raya y doblemente subraya-
dos son los sumandos de Sn,1, Sn,2 , respectivamente. Las tres columnas de la
derecha muestran que, en cada caso, dos de las sumas son iguales, mientras
que la tercera es mayor (indicada por un supraı́ndice +) o menor (indicada
por un supraı́ndice -). También se ve que el término desigual en esta suce-
sión cambia en un ciclo de seis. Ası́, del patrón establecido en los primeros
3.1. BÚSQUEDA DE UN PATRÓN 121

renglones, esperamos que la anomalı́a para n = 8 ocurra en la columna de


enmedio y sea uno más que las otras dos.

Sabemos que Sn,0 + Sn,1 + Sn,2 = 2n (ver 1.1.1). Ya que 100 = 6 × 16 + 4,


esperemos que el término desigual ocurra en la tercera columna (S100,2 ) y que
sea uno más que las otras dos. Ası́ S100,0 = S100,1 = S100,2 −1 y S100,1 +S100,1 +
100
S100,1 +1 = 2100 . Estas ecuaciones nos llevan a conjeturar que S100,1 = (2 3−1)

Una demostración formal de esta conjetura es una aplicación directa de in-


ducción matemática (ver capı́tulo 2).

3.1.3 Establezca las condiciones necesarias y suficientes sobre x1 y x2 para


que xn sea un número entero para una infinidad de valores de n. Donde
x1 , x2 , x3 , ... es una sucesión de números reales distintos de cero que satisfacen
xn−2 xn−1
xn = n = 3, 4, 5, ...
2xn−2 − xn−1

Solución. Para darse una idea de la sucesión, calcularemos los primeros


términos, expresándolos en términos de x1 y x2 . Tenemos (omitiendo el álge-
bra)

x1 x2
x3 = ,
2x1 − x2
x1 x2
x4 = ,
3x1 − 2x2
x1 x2
x5 = .
4x1 − 3x2

Somos afortunados en este caso en particular de que los cálculos sean re-
alizables y de que emerja un patrón. Un sencillo argumento por inducción
establece que
x1 x2
xn = ,
(n − 1) x1 − (n − 2) x2
lo cual, al aislar el coeficiente para n toma la forma
x1 x2
xn = .
(x1 − x2 ) n + (2x2 − x1 )
122 CAPÍTULO 3. HEURÍSTICA

De este modo, vemos que si x1 6= x2 , el denominador excederá eventualemente


al numerador, en magnitud, de forma que xn no será un entero. Sin embargo,
si x1 = x2 , todos lo términos de la sucesión son iguales. Ası́, xn es un entero
para un número infinito de valores de n si y sólo si x1 = x2 .

3.1.4 Encuentre números positivos n y a1 , a2 ..., an tales que a1 + a2 + ... +


an = 1000 y el producto a1 a2 ...an sea tan grande como se pueda.

Solución. Cuando un problema involucra un párametro que hace el análi-


sis complicado, frecuentemente es de ayuda en la etapa de descubrimiento
el reemplazo temporal con algo más manejable. En este problema, podemos
comenzar por examinar una sucesión de casos particulares obtenidos al reem-
plazar 1000 por 2, 3, 4, 5, 6, 7, 8, 9, ... . En esta forma llegamos a descubrir que
en un producto máximo

(i) ningún ai será mayor que 4,

(ii) ningún ai será igual a 1,

(iii) todas las ai ′ s pueden ser tomadas como 2 ó 3 (porque 4 = 2 × 2 y


4 = 2 + 2),

(iv) a los más dos ai ′ s serán iguales a 2 (porque 2 ×2 × 2 < 3 × 3 y


2 + 2 + 2 = 3 + 3).

Cada una de éstas es fácil de establecer. Ası́ cuando el parámetro es 1000


como en el problema en cuestión, el producto máximo debe ser 3332 × 22 .

3.1.5 Muestre que x ∗ y = y ∗ x para toda x, y en S. Donde S un conjunto


y ∗ una operación binaria sobre S que satisface las dos leyes

x∗x = x
(x ∗ y) ∗ z = (y ∗ z) ∗ x

Solución. La solución, que aparece tan transparentemente abajo, es en re-


alidad el resultado de una cantidad considerable de trabajo; el procedimento
puede ser descrito sólo como la búsqueda de un patrón (el patrón guı́a está en
3.1. BÚSQUEDA DE UN PATRÓN 123

la naturaleza cı́clica de los factores en la segunda condición). Tenemos, para


toda x, y en S,

x ∗ y = (x ∗ y) ∗ (x ∗ y) = [y ∗ (x ∗ y)] ∗ x = [(x ∗ y) ∗ x] ∗ y
= [(y ∗ x) ∗ x] ∗ y = [(x ∗ x) ∗ y] ∗ y = [(y ∗ y)] ∗ (x ∗ x)
= y ∗ x.

Problemas

Desarrolle una idea para los siguientes problemas mediante la búsqueda de


patrones. Haga conjeturas apropiadas, y piense cómo podrı́an llevarse a cabo
las demostraciones.

3.1.6 Comenzando con 2 y 7, la sucesión 2, 7, 1, 4, 7, 4, 2, 8, ... se construye


al multiplicar parejas sucesivos de sus términos y añadiendo el resultado como
el siguiente término o los dos siguientes términos de la sucesión, dependiendo
de si el producto es un número de uno o dos dı́gitos. Demuestre que el dı́gito
6 aparece un número infinito de veces en la sucesión.

3.1.7 Denótese por S1 a la sucesión de los enteros positivos 1, 2, 3, 4, 5, 6, ...,


y defı́nase Sn+1 en términos de Sn añadiendo 1 a aquellos enteros en Sn que
son divisibles por n. Ası́, por ejemplo, S2 es 2, 3, 4, 5, 6, 7, ..., S3 es 3, 3, 5, 5, 7, 7, ...
Determine aquellos enteros n con la propiedad de que los primeros n − 1 en-
teros en Sn son n.

3.1.8 Demuestre que puede hacerse una lista de todos los subconjuntos de
un conjunto finito de tal forma que

(i) El conjunto vacı́o sea primero en la lista.

(ii) Cada subconjunto aparezca sólo una vez, y

(iii) Cada subconjunto en la lista se obtenga ya sea al añadir un elemen-


to al subconjunto precedente o al borrar un elemento del subconjunto
precedente.
124 CAPÍTULO 3. HEURÍSTICA

3.1.9 Determine el número de coeficientes binomiales impares en la expan-


sión de (x + y)100 .

3.1.10 Un teorema bien conocido asegura que un primo p > 2 puede ser
escrito como la suma de dos cuadrados perfectos (p = m2 + n2 , con m y n
enteros) si y sólo si p es uno más que un múltiplo de 4. Haga una conjetura
sobre cuáles primos p > 2 pueden ser escritos en cada una de las siguientes
formas, utilizando enteros x y y (no necesariamente positivos): (a) x2 +16y 2,
(b) 4x2 + 4xy + 5y 2 . (ver 1.5.10)

3.1.11 Si (an ) es una sucesión tal que para n ≥ 1, (2 − an )an+1 = 1,


qué sucede con an cuando n tiende hacia infinito?

3.1.12 Sea S un conjunto, y sea ∗ una operación binaria sobre S que sat-
isface las leyes

x ∗ (x ∗ y) = y
(y ∗ x) ∗ x = y

Demuestre que x ∗ y = y ∗ x para todas x, y en S.

Ejemplos adicionales

La mayor parte de los problemas de inducción están basados en el descubrim-


iento de un patrón. Ası́, los problemas en las secciones 2.1, 2.2, 2.3, 2.4 ofre-
cen práctica adicional para esta heurı́stica. Véase también 1.7.2, 1.7.7, 1.7.8,
2.5.6.

3.2. Dibujo de una Figura

Es de mucha ayuda describir un problema gráficamente siempre que sea posi-


ble, por medio de una figura, un diagrama, o una gráfica. Una representación
gráfica usualmente hace más fácil asimiliar los datos relevantes y notar las
relaciones y las dependencias.
3.2. DIBUJO DE UNA FIGURA 125

3.2.1 Una cuerda de longitud constante se desliza a lo largo de un semicı́rcu-


lo. El punto medio de la cuerda y las proyecciones de sus extremos son los
vértices de un triángulo. Demuestre que el triángulo es isósceles y que nunca
cambia de forma.
X M
Y

A C N D B
Figura 1.1

Solución. Denótese por AB la base del semicı́rculo, y sean XY la cuerda,


M el punto medio de XY , C y D las proyecciones de X y Y sobre AB (fig.
1.2) Denótese la proyección de M sobre AB como N. Entonces N es el punto
medio de CD y se sigue que el triángulo ∆CMD es isósceles.

Para mostrar que la forma del triángulo es independiente de la posición de


la cuerda, basta mostrar que el ∠MCD permanece sin cambio, o equivalen-
temente, que el ∠XCM es constante, para todas las posiciones de XY . Para
ver que tal es el caso, extiéndase XC hasta cortar el cı́rculo completo en Z
(fig. 1.3). Entonces CM es paralela a ZY (C y M son los puntos medios de
XZ y XY respectivamente), y consecuentemente ∠XCM = ∠XZY es igual
a la mitad del arco XY , y este arco depende sólo de la longitud de la cuerda
XY . Esto completa la demostración.
X
M
b

Y b

A b

C
b

D
b b

Z
b

Figura 1.2

Uno podrı́a preguntarse: Cómo fue que se le ocurrió a alguien extender XC


126 CAPÍTULO 3. HEURÍSTICA

de esta manera? Este es precisamente el paso que hace el argumento tan


bonito, y es ciertamente un paso difı́cil de motivar. Casi todo lo que puede
decirse es que el uso de lı́neas y arcos auxiliares (frecuentemente encontrados
por reflexión, extensión, o rotación) es una práctica común en geometrı́a. El
sólo hecho de tomar esto en cuenta contribuirá a las posibles aproximaciones
a un problema dado.
X
M
b

Y b

A b

C
b b

O D
b b

Z
b

Figura 1.3

Otro acercamiento interesante a este problema es dar coordenadas a los pun-


tos y proceder analı́ticamente. Para mostrar que el triángulo es independiente
de la posición de la cuerda, basta con mostrar que la razón entre la altura y
la base, MN/CD, es constante.

Sea O el punto medio de AB, y sea θ = ∠Y OB. Es claro que toda la config-
uración está determinada por θ (fig. 1.4)

Sea α = ∠XOY. Usando esta notación,

CD = cosθ − cos(θ + α)
senθ + sen(θ + α)
MN = .
2

y la razón altura-base es
senθ + sen(θ + α)
F (θ) = , 0≤θ ≤π−α
2(cos θ − cos (θ + α))
No es inmediatamente claro que esta cantidad es independiente de θ; este es
el contenido de 1.8.1 y 6.6.7
3.2. DIBUJO DE UNA FIGURA 127

3.2.2 Una partı́cula que se mueve sobre una lı́nea recta comienza del re-
poso y alcanza una velocidad v0 después de recorrer una distancia s0 . Si el
movimiento es tal que la aceleración nunca aumenta, encuentre el tiempo
máximo para el recorrido.
3 V
V0 b b
P
2 b

1 S0

t
b

O
1 2 3 4 5 6
−1 Figura 1.4

Solución. Concentre la atención en la gráfica de la velocidad v = v(t) (fig.


1.5). Tenemos que v(0) R t = 0, y la gráfica de v nunca es cóncava hacia arriba
(distancia recorrida= 0 v(t)dt). A partir de esta representación, es claro que
maximizaremos el tiempo de recorrido cuando la curva v(t) de 0 a P sea una
lı́nea recta (fig. 1.6). En el tiempo máximo t0 , 21 t0 v0 = s0 , o equivalentemente,
t0 = 2s0 /v0 .
3 V
V0 b b
P
2

1
S0
t
b b

O
1 2 3 4 5 6
−1 Figura

3.2.3 Si a y b son enteros positivos sin factores comunes, muestre que

 a   2a  hh ii (a − 1)(b − 1)
(b−1)a
b
+ b
+ ... + b
=
2

Solución. Cuando b = 1, vemos que la suma de la izquierda es 0 y ası́ se


cumple el resultado.
128 CAPÍTULO 3. HEURÍSTICA

No es claro cómo una figura podrı́a ser de ayuda para establecer esta iden-
tidad puramente aritmética. Además, la afirmación involucra dos variables
independientes, a y b, y ab , 2a
b
, 3a
b
, .... son los valores de la función f (x) = ax
b
cuando x = 1, 2, 3, ...., respectivamente.
   
Es posible interpretar ab , 2a b
, .... geométricamente?

Para concretar, considérese el caso a = 5 y b = 7. Los  5kpuntos


 Pk = (k, 5k/7), k =
1, 2, ..., 6, están cada uno en la recta y = 5k/7, y 7 es igual al número de
puntos de la retı́cula en la lı́nea
P vertical
 5k  a través de Pk que está por encima
del eje x y abajo de Pk . Ası́, k=1 6 7
es igual al número de puntos de la
retı́cula que están dentro del ∆ABC (ver fig. 1.7). Por simetrı́a, este número
es la mitad del número de púntos de la retı́cula que están en el interior del
rectángulo ABCD. Hay 4 × 6 = 24 puntos de la retı́cula en ABCD, lo que
significa que el triángulo ABC contiene 12 puntos de la retı́cula en su interior.

5 b b b b b b b

4 b b b b b b b

3 b b b b b b b

2 b b b b b b b

1 b b b b b b b

1 2 3 4 5 6 7
−1 Figura

El mismo argumento puede aplicarse en el caso general. La condición de que


a y b no tienen factores comunes nos asegura que ninguno de los puntos de
la retı́cula interior de ABCD caerá en la recta y = ax/b. Ası́,

b−1  
X ka 1
= k
k=1
b 2
3.2. DIBUJO DE UNA FIGURA 129

(donde k es el número de puntos de la retı́cula en el interior de ABCD)


(a − 1)(b − 1)
=
2

3.2.4 (El problema de los saludos de mano) El Sr. y la Sra. Adams


fueron hace poco a una fiesta en la que habı́a otras tres parejas. Ocurrieron
varios saludos de mano. Nadie saludó a su propio cónyugue, ni a la misma
persona dos veces, y por supuesto, nadie se saludó a si mismo.

Después de que los saludos hubieron terminado, el Sr. Adams le preguntó a ca-
da persona, incluida su esposa, a cuántas personas habı́a saludado de mano?.
Para sorpresa suya, cada quien dió una respuesta diferente. A cuántas per-
sonas saludó de mano la Sra. Adams?

Solución. Aunque un diagrama no es propiamente la solución , es de gran


ayuda visualizar los datos gráficamente de la siguiente manera. Represéntese
a los ocho individuos con los ocho puntos que muestra la figura 1.7

Ahora bien, las respuestas a la encuesta del Sr. Adams deben haber sido
los números 0, 1, 2, 3, 4, 5, 6. Ası́, uno de los individuos, A digamos, saludó a
otros seis, digamos B, C, D, E, F, G. Indı́quese esto en la gráfica dibujando
segmentos desde A a estos puntos, como en la figura 1.8.

En este diagrama vemos que H debe haber sido la persona que no saludó a
nadie. Además, A y H deben ser esposos, pues A ha saludado a seis personas,
sin contar a su cónyugue.

Por suposición, alguno entre B, C, D, E, F, G ha saludado a cinco personas.


Re-etiquetando de ser necesario, podemos suponer que se trata de la persona
B saludó son A, C, D, E, F. Esto se muestra en la figura 1.9. A partir de este
dibujo vemos fácilmente que G es la única persona que pudo haber contestado
üno”, y B y G deben estar casados.

De nuevo, como antes, re-etiquetando los puntos C, D, E, F de ser necesario,


podemos suponer que C saludó a cuatro personas y que éstas eran A, B, D, E.
El diagrama correspondiente se muestra en la figura 1.10. Usando el mismo
razonamiento, F y C son esposos, y consecuentemente, D y E son esposos.
130 CAPÍTULO 3. HEURÍSTICA

D y E han ambos saludado a otros tres. Dado que el Sr. Adams no recibió dos
respuestas ”tres”, D y E deben corresponder al Sr. y la Sra. Adams ; es decir,
el Sr. Adams saludó a otros tres.
A B b
A b B b b

H C H
b b b b
C

b
D b

G
b
C b

D
b b b b

F E F E
Figura Figura 1.8
1.7

A b b B A
b b B

H H
b b
C b b
C

b b

G
b
D G b

D
b
F Eb b

F
b

E
Figura 1.9 Figura 1.10

Problemas
3.2. DIBUJO DE UNA FIGURA 131

3.2.5 Dos postes, de alturas a y b, están a una distancia d (con respecto


al nivel del suelo). Un alambre se extiende desde la parte superior de cada
uno de ellos a un punto P en el suelo entre ellos. En dónde deberı́a estar
localizado P para minimizar la longitud total del cable? (Sugerencia: Suponga
que los postes se encuentran en los puntos C y D, que sus partes superiores
son A y B, respectivamente. Deseamos minimizar AP + P B. Aumente este
diagrama reflejándolo en la lı́nea base CD. Suponga que B se refleja a B ′
(P B = P B ′ ). Ahora el problema es: En dónde deberı́a estar localizado P para
minimizar AP + P B ′ ?)

3.2.6 Sea ABC un triángulo de ángulos agudos, y sea D en el interior


del segmento AB. Localice los puntos E sobre AC y F sobre CB tales que
el triángulo inscrito DEF tenga perı́metro mı́nimo.(Sugerencia: Refleje D
sobre la lı́nea AC a un punto D ′ ; refleje D sobre CB a un punto D ′′ y
considere el segmento D ′ D ′′ ).

3.2.7 En un cuarto rectangular que mide 30 pies de largo y 12 pies de


altura, y 12 pies de ancho. Una mosca, con un ala rota, se encuentra en un
punto a la mitad de una barda cuadrada y un pie abajo del techo. Un pedazo
de comida se encuentra localizado un pie arriba del piso y a la mitad de la
barda opuesta. La mosca tiene únicamente la energı́a para caminar 40 pies.
Muestre que hay un camino por el cual la mosca pueda caminar y conseguir
la comida.

3.2.8 Los triángulos equilateros ABP y ACQ están construidos externa-


mente sobre los lados AB y AC del triángulo ABC. Pruebe que CP = BQ.
(Sugerencia: Para encontrar una solución bonita, rote el plano del triángu-
lo 600 por el punto A, en una dirección que lleve B en la dirección de C.
Qué sucede con el segmento CP ?)

3.2.9 Sean a y b números enteros positivos tales que a < b. Si dos puntos
son seleccionados al azar de un segmento de longitud b, cuál es la probabilidad
de que la distancia entre ellos sea al menos a? (Sugerencia: Sean x y y los
números escogidos al azar del intervalo [0, b], y considere a estas variables
aleatorias independientes sobre dos ejes separados. Qué área corresponde a
|x − y| ≥ a?).
132 CAPÍTULO 3. HEURÍSTICA

3.2.10 Dé una interpretación geométrica al siguiente problema. Sea f difer-


enciable con f continua en [a, b]. Muestre que si existe un número c en (a, b]

tal que f (c) = 0, entonces podemos encontrar un número d en (a, b) tal que
f (d) − f (a)
f ′ (d) = .
b−a

3.2.11 Sea a, b números reales, a < b. Indique geométricamente la ubi-


cación precisa de cada uno de los siguientes números: (a+b)
2
(= 21 a + 12 b); 32 a +
1
3
b; 13 a + 23 b; [m/(m + n)]a + [n/(m + n)]b, donde m > 0 y n > 0. Este último
número corresponde al centro de gravedad de un sistema de dos masas, uno
de masa m, localizado en a, y el otro, de masa n, localizado en b.

3.2.12 Use la gráfica de y = senx para mostrar lo siguiente. Dado un


triángulo ABC,
SenB+SenC
(a) 2
≤ Sen B+C2
m n m n
(b) m+n
SenB + m+n
SenC ≤ Sen( m+n
B + m+n
C), m > 0, n > 0

3.2.13
PnUse Pun diagrama (un arreglo rectangular
Pn Pn(ai aj )) para mostrar que
n
(a) ai aj = i=0 ai aj ,
Pi=0
n Pj=0
n Pj=0
n Pn
(b) aa = ai aj ,
Pn i=j 2i j
j=0 Pn Pn i=0P j=iP
n i Pn 2
(c) ( i=0 ai ) = i=0 j=0 ai aj = 2 i=0 j=0 ai aj − i=0 ai .

Ejemplos adicionales

1.3.11, 1.9.2, 1.9.4, 1.11.3, 2.1.3, 2.5.5, 2.6.11

3.3. Formulación de un Problema Equivalente

El mensaje de la sección precedente es que el primer paso para resolver un


problema es juntar información, explorar, entender, relacionar, conjeturar,
3.3. FORMULACIÓN DE UN PROBLEMA EQUIVALENTE 133

analizar. Pero, qué pasa cuando no es posible hacer esto de manera significa-
tiva, sea porque los cálculos se vuelven muy complicados o porque el proble-
ma no admite casos especiales que nos hagan vislumbrar la solución? En esta
sección consideraremos algunos problemas de este tipo. La recomendación
de esta sección es tratar de reformular el problema a una forma equivalente
pero más simple. El llamado es a la imaginación y creatividad propia. Al-
gunas técnicas estándar de reformulación involucran cálculos algebraı́cos o
manipulaciones trigonométricas, substitución o cambio de variable, uso de
correspondencias biunı́vocas, y reinterpretación del lenguaje de otra materia
(álgebra, geometrı́a, análisis, combinatoria, etc.)

3.3.1 Encuentre una fórmula general para la n-ésima derivada de f (x) =


1/(1 − x2 ).

Solución. Un paso simplificador común al trabajar con funciones racionales


es escribir la función como una suma de fracciones parciales. En este caso,

 
1 1 1
f (x) = 2
+
1−x 1+x

y en esta forma es fácil mostrar que

 
(n) n! 1 (−1)n
f (x) = 2
+
(1 − x)n+1 (1 + x)n+1

3.3.2 Encuentre todas las soluciones de x4 + x3 + x2 + x + 1 = 0

Solución. Esta ecuación puede resolverse dividiendo entre x2 , luego sub-


stituyendo y = x + 1/x, y aplicando la fórmula para resolver ecuaciones de
segundo grado. Tenemos

 x2 + x12 + x + x1 + 1 = 0,
x2 + 2 + x12 + x + x1 + (1 − 2) = 0,
2 
x + x1 + x + x1 − 1 = 0,
y 2 + y − 1 = 0,
134 CAPÍTULO 3. HEURÍSTICA

Las raı́ces de esta ecuación son


√ √
−1 + 5 −1 − 5
y1 = , y2 = .
2 2

Sólo queda determinar x resolviendo las ecuaciones

1 1
x+ = y1 y x+ = y2 ,
x x

Que son equivalentes a

x2 − y1 x + 1 = 0 y x2 − y2 x + 1 = 0.

Las cuatro raı́ces encontradas al resolver éstas son

√ p √
−1 + 5 10 + 2 5
x1 = +i ,
4 4
√ p √
−1 + 5 10 + 2 5
x2 = −i ,
4 4
√ p √
−1 − 5 10 − 2 5
x3 = +i ,
4 4
√ p √
−1 − 5 10 − 2 5
x4 = −i .
4 4

Otra aproximación a este problema es multiplicar cada lado de la ecuación


original por x − 1. Ya que

(x − 1)(x4 + x3 + x2 + x + 1) = x5 − 1, un problema equivalente es encontrar


todas las x que satisfagan x5 = 1. Estas son las cinco raı́ces de la unidad.

Como una consecuencia de haber trabajado este problema en dos formas


diferentes, vemos que
3.3. FORMULACIÓN DE UN PROBLEMA EQUIVALENTE 135
√ p √
2 2 −1 + 5 10 + 2 5
Cos π + iSen π = +i .
5 5 4 4

Al igualar las partes real e imaginarias se obtiene

√ p √
−1 + 5 10 + 2 5
Cos720 = , Sen72 = i 0
4 4

(Fórmulas similares pueden ser obtenidas para x2 , x3 , y x4 )

3.3.3 Sea P un punto dentro de un triángulo dado ABC, D, E, F son los


pies de las perpendiculares desde P a las lı́neas BC, CA, AB, respectiva-
mente. Encuentre todos los P para los cuales
BC CA AB
+ +
PD PE PF
es mı́nimo.
b
A
b
F
c b
E q
b

r
P b

C
p b

a
b

B b
D
Figura 1.11

Solución. Denótense las longitudes de BC, AC, AB, por a, b, c, respectiva-


mente (ver figura 1.11). Queremos minimizar ap + qb + rc .

Notar que

Area ∆ABC = Area ∆BP C + Area ∆CAP + Area ∆ABP


1 1 1
= ap + bq + c
2 2 2
ap + bq + cr
=
2
136 CAPÍTULO 3. HEURÍSTICA

Ası́, ap + bq + cr es una constante, independientemente de la colocación de


P . Por lo tanto, en vez de minimizar ap + qb + rc , minimizaremos (ap + bq +
cr)( ap + qb + rc ). Se obtiene

(ap + bq + cr)( ap + qb + rc ) = a2 + b2 + c2 + ab( pq + pq ) + bc( qr + qr ) + ac( pr + pr ) ≥

a2 + b2 + c2 + 2ab + 2bc + 2ac = (a + b + c)2

La desigualdad en el segundo paso se sigue del hecho de que para cualesquiera


dos números positivos x y y tenemos xy + xy ≥ 2, con igualdad si y sólo si
x = y. Como resultado de este hecho, (ap + bq + cr)( ap + qb + rc ) alcanzará su
valor mı́nimo (a+b+c)2 cuando, y sólo cuando, p = q = r. Equivalentemente,
a
p
+ qb + rc alcanza un valor mı́nimo cuando P se encuentra en el incentro del
triángulo.

3.3.4 Probar que si m y n son enteros positivos y 1 ≤ k ≤ n, entonces

    
Pk n m n+m
i=0 =
i k−i k

Solución. La afirmación del problema constituye una de las ideas funda-


mentales que involucran coeficientes binomiales. En el lado izquierdo está una
suma de productos de coeficientes binomiales. Obviamente, una substitución
directa de factoriales en los coeficientes binomiales no nos da ninguna intro-
spección.

Muy seguido, las series finitas (especialmente aquellas que involucran coefi-
cientes binomiales) pueden ser sumadas combinatoriamente. Para entender
lo que se quiere decir aquı́, transforme el problema de series en un problema
de conteo de la siguiente manera. Sea S = A∪B, donde A es un conjunto con
n elementos y B es un conjunto, ajeno a A, con m elementos. Contaremos,
de dos formas diferentes, el número de  subconjuntos
 (distintos) de S con k
m+n
elementos. Por un lado este número es . Por otro lado, el número
k
  de Scon exactamente i elementos de A (y k − i elementos
de subconjuntos
n m
de B) es . Se sigue que
i k−i
3.3. FORMULACIÓN DE UN PROBLEMA EQUIVALENTE 137

 
m+n
= No. de k − subconjuntos de S.
k
Xk
= (No. de k − subconjuntos de S con i elementos de A)
i=0
k   
X n m
=
i=0
i k−i

Los problemas de conteo frecuentemente pueden ser simplificados ”Identi-


ficando” (por medio de una correspondencia uno-a-uno) los elementos del
conjunto con los de otros cuyos elementos puedan ser contados más fácil-
mente. Los siguientes tres ejemplos ilustran esta idea.

3.3.5 Sobre un cı́rculo se seleccionan n puntos y se dibujan las cuerdas que


los unen por pares. Suponiendo que no hay tres de estas cuerdas que sean
concurrentes (excepto en los extremos), cuántos puntos de intersección hay?
b

b
b

b b
b

b
b b

b
b

Figura 1.12

Solución. Los casos para n = 4, 5, 6 se muestran en la figura 1.12. Nótese


que cada punto de intersección (interior) determina, y está determinado por,
cuatro de los n puntos dados alrededor del cı́rculo (estos cuatro puntos pro-
ducirán en forma única dos cuerdas que se intersectarán  en el interior del
n
cı́rculo). Ası́, el número de puntos de intersección es .
4

3.3.6 Dado un entero positivo n, encuentre el número de cuádruplas de


enteros (a, b, c, d) tales que 0 ≤ a ≤ b ≤ c ≤ d ≤ n.
138 CAPÍTULO 3. HEURÍSTICA

Solución. La idea clave que hace el problema transparente es notar que hay
una correspondencia uno-a-uno entre las cuádruplas de nuestro conjunto y el
de subconjuntos de cuatro elementos tomados de {0, 1, ..., n + 3}. Especı́fica-
mente, identifı́quese a (a, b, c, d), 0 ≤ a ≤ b ≤ c ≤ d ≤ n, con el subconjunto
{a, b+1, c+2, d+3}. Es fácil ver que esta correpondencia es uno-a-uno. Cada
elemento de nuestro conjunto corresponde exactamente a un subconjunto  de
n+4
cuatro de {0, 1, ..., n + 3}, y viceversa. Ası́, el número deseado es .
4

3.3.7 El número cinco puede ser expresado como una suma de 3 números
naturales, tomando en cuenta el orden, en 6 maneras, especı́ficamente, como

5= 1+1+3 =1+3+1 =3+1+1 =1+2+2= 2+1+2= 2+2+1

Sean m y n números naturales tales que m ≤ n. De cuántas maneras puede


ser escrito n como una suma de m números naturales, tomando en cuenta
el orden?

Solución. Escribir n como una suma de n unos:

n = |1 + 1 +
{z... + 1}
n

 de maneras de escoger m − 1 signos


El número que buscamos es elnúmero
n−1
de suma de los n − 1; esto es .
m−1

Problemas

3.3.8 Mostrar que x7 + 2x5 + 10x2 − 1 no tiene ninguna raı́z mayor que 1.
(Sugerencia: ya que generalmente es más fácil probar que una ecuación no
tiene raı́ces positivas, nos vemos empujados a considerar el problema equiv-
alentemente obtenido al hacer la sustitución algebráica x = y + 1.)
3.3. FORMULACIÓN DE UN PROBLEMA EQUIVALENTE 139

3.3.9 El número tres puede ser expresado como la suma de uno o más
enteros positivos, tomando en cuenta el orden, de cuatro maneras, especı́fi-
camente, como 3, 1+2, 2+1, y 1+1+1. Mostrar que cualquier entero positivo
n puede ser ası́ expresado en 2n−1 formas.

3.3.10 De cuántas maneras puede ser expresado el 10 como una suma de


5 enteros no negativos, cuando se toma en cuenta el orden? (Sugerencia:
Encontrar un problema en el que la frase “5 enteros no negativos” sea reem-
plazada por 5 enteros “positivos”).

3.3.11 Para cuáles valores de a el sistema de ecuaciones

x2 = y 2
(x − a)2 + y 2 = 1

tiene exactamente cero, una, dos, tres, cuatro soluciones, respectivamente?


(Sugerencia: Traduzca el problema a un problema geométrico equivalente).

3.3.12 Dado n objetos arreglados en un renglón, un subconjunto de estos


objetos es llamado no amigable si nunca dos de sus elementos están con-
tiguos. Mostrar
 que el número
 de subconjuntos no amigables con k elementos
n−k+1
cada uno es . (Sugerencia: Adoptar una idea similar a la uti-
k
lizada en 1.3.6)

3.3.13 Sea a(n) el número de representaciones del entero positivo n como


una suma de 1’s y 2’s tomando el orden en cuenta. Sea b(n) el número
representaciones de n como una suma de enteros mayores que 1 , nuevamente
tomando el orden en cuenta y contando el sumando n. La tabla de abajo
muestra que a(4) = 5 y b(6) = 5 :
140 CAPÍTULO 3. HEURÍSTICA

a-sumas b- sumas
1+1+2 4+2
1+2+1 3+3
2+1+1 2+4
2+2 2+2+2
1+1+1+1 6

(a) Mostrar que a(n) = b(n + 2) para toda n, describiendo una


correspondencia uno-a-uno entre las a-sumas y las b- sumas.

(b) Mostrar que a(1) = 1, a(2) = 2, y para n > 2, a(n) = a(n − 1) + a(n − 2).

3.3.14 Hallando el área de un triángulo de dos formas diferentes, mostrar


que si p1 , p2 , p3 son las alturas de un triángulo y r es el radio del cı́rculo
inscrito, entonces 1/p1 + 1/p2 + 1/p3 = 1/r.

3.3.15 Usar
 un 
argumento
 de conteo
 para  probar
 que
 para enteros r, n,
r r+1 r+2 n n+1
0 < r ≤ n, + + + .... + = .
r r r r r+1

Ejemplos adicionales

Hay tantos ejemplos de esta heurı́stica que es difı́cil señalar aquellos que son
los más tı́picos. Dignos de atención son las demostraciones indirectas en las
secciones 1.10,1.11.

3.4. Modificación del Problema

En el transcurso del trabajo sobre un problema A podemos vernos llevados


a considerar otro problema B. Comúnmente, este cambio de problemas es
anunciado con frases tales como ’basta con mostrar que.....’ o ’sin pérdida
de generalidad....’. En la última sección vimos ejemplos en los cuales A y B
3.4. MODIFICACIÓN DEL PROBLEMA 141

eran problemas equivalentes, esto es, la solución de cualquiera de ellos implica


la solución del problema modificado (o auxiliar), el problema B, implica la
solución de A, pero no necesariamente viceversa.

3.4.1 Dados los números positivos a, b, c, d, demuestre

a3 + b3 + c3 b3 + c3 + d3 c3 + d3 + a3 d3 + a3 + b3
+ + + ≥ a2 + b2 + c2 + d2 .
a+b+c b+c+d c+d+a d+a+b

Solución. Debido a la simetrı́a del problema, basta con demostrar que para
cualesquiera números positivos x, y y z

x3 + y 3 + z 3 x2 + y 2 + z 2
≥ .
x+y+z 3

Si este fuera el caso, el lado izquierdo de la desigualdad original es al menos

a2 + b2 + c2 b2 + c2 + d2 c2 + d2 + a2 d2 + a2 + b2
+ + + ≥ a2 + b2 + c2 + d2
3 3 3 3

Ahora, para probar esta última desigualdad, no hay pérdida de generalidad


al suponer que x + y + z = 1. Pero si no, simplemente dividamos cada lado de
la desigualdad por (x+y+z)2 , y hagamos X = x/(x+y+z), Y = y/(x+y+z)
y Z = z/(x + y + z).

De esta manera , el problema original se reduce al siguiente problema modificado:

Dados números positivos X, Y, Z tales que X + Y + Z = 1, probar que

X2 + Y 2 + Z2
X3 + Y 3 + Z3 ≥ .
3
142 CAPÍTULO 3. HEURÍSTICA

3.4.2 Sea C cualquier punto en el segmento AB entre A y B, y dibújense


semicı́rculos sobre el mismo lado de AB con AB, AC y CB como diámetro
(figura 1.13). Sea D un punto en el semicı́rculo de diámetro AB tal que
CD es perpendicular a AB, y sean E y F puntos sobre los semicı́rculos de
diámetros AC y CB, respectivamente, de forma que EF sea un segmento de
su tangente común. Probar que ECF D es un rectángulo.

Solución. Nótese que basta con mostrar qu A, E, y D son colineales (el


mismo argumento mostrarı́a que B, F y D son colineales). Si este fuera el
caso, AEC = 900 (E está en el cı́rculo AEC), ADB = 900 , y el resultado es
verdadero. Se hace evidente, sin embargo, que con poca introspección, hay
muchas formas de encaminarse mal con esta aproximación; es difı́cil evitar
dar por cierta la conclusión.
D b

Fb

b
E

A b

C
b b

B
Figura 1.13

Una forma de ganar introspección en las relaciones entre los parámetros de


un problema es notar el efecto cuando a uno de ellos se le hace variar (mod-
ificación del problema). En este problema, se hace variar a D a lo largo
de la circunferencia. Sean G y H (figura 1.14) las intersecciones de los seg-
mentos AD y BD con los cı́rculos de diámetro AC y CB (y centros O y
O ′) respectivamente. Entonces AGC = ADB = CHB = 900, de tal forma
que GDHC es un rectángulo. Además, OGC = OCG (∆OGC es isósceles),
y CGH = GCD porque GH y CD son diagonales de un rectángulo. Ası́,
OGH = OCD. Ahora bien, mientras D se mueve para hacer CD perpendic-
ular AB, OGH se moverá también 900 , de forma que GH sea tangente al
cı́rculo O, y G conicida con E. Un argumento similar muestra que GH
es tangente al cı́rculo O ′, entonces H = F . Esto completa la demostración.
(Nótese que la frease ün argumento similar”, otra técnica de simplificación,
tiene el mismo efecto cuando es colocada después de un argumento que el
3.4. MODIFICACIÓN DEL PROBLEMA 143

que tiene ”basta mostrar queçuando está colocada antes del argumento.)
Db

Gb

H
b

A b b

O C
b

O′
b b

B
Figura 1.14

Nótese que se ha resuelto el problema resolviendo un problema más general.


Esta es una técnica común de resolución de problemas; se verán más ejemplos
en la sección 1.12.

3.4.3 Demuestre que no existen enteros positivos x, y, z tales que

x2 + y 2 + z 2 = 2xyz

Solución. Suponga que x, y, z son enteros positivos tales que x2 + y 2 + z 2 =


2xyz. Ya que x2 + y 2 + z 2 es par (= 2xyz), dos entre x, y, z son pares y el otro
es par , o los tres son pares. Suponga que x, y, z son pares. Entonces existen
enteros positivos x1 , y1 , z1 tales que x = 2x1 , y = 2y1 , z = 2z1 . Del hecho que
(2x1 )2 + (2y1 )2 + (2z1 )2 = 2(2x1 )2 (2y1 )2 (2z1 )2 se sigue que x1 , y1 , z1 satisfacen
que x1 2 + y1 2 + z1 2 = 22 x1 y1 z1 . De nuevo, de esta ecuación, si x1 , y1, z1 son
pares un argumento similar muestra que existen enteros positivos x2 , y2 , z2
tales que x2 2 + y2 2 + z2 2 = 23 x2 y2 z2 .

Continúese de esta forma. Eventualmente se llegará a una ecuación de la


forma a2 + b2 + c2 = 2n abc donde no todos a, b, c son pares (y de aquı́ que
dos entre a, b, c sean pares y uno impar).

Ası́, hemos sido llevados a considerar el siguiente problema modificado: De-


muestre que no existen enteros positivos x, y, z y n, con x, y pares, tales que

x2 + y 2 + z 2 = 2n xyz
144 CAPÍTULO 3. HEURÍSTICA

(Este es el problema 1.9.3)

R∞ 2
3.4.4 Evalúe 0
e−x dx.

Solución. Las técnicas usuales de integración estudiadas en el primer año


de cálculo no funcionarán para esta integral. Para evaluar la integral trans-
formaremos la integral simple en una integral doble.
R∞ 2
Sea I = 0 e−x dx. Entoces

hR i hR i
∞ −x2 ∞ −y 2
I2 = 0
e dx 0
e dy
R ∞ hR ∞ −x2 i −y2
= 0 e dx e dy
R ∞ R0∞ −x2 −y2
= R0 R0 e e dxdy
∞ ∞ 2 2
. = 0 0 e−(x +y ) dxdy

Ahora cámbiese a una integral equivalente utilizando coordenadas polares.


Entonces tenemos

R π R∞ 2
I 2 = 02 0 e−r rdrdθ
Rπ 2
= 02 − 12 e−r |θ0 dθ

= 21 02 dθ
π
=
4

π
Se sigue que I = 2

Un problema modificado (auxiliar) puede surgir de muchas maneras. Puede


aparecer mediante un cambio de notación (como en 1.4.4; ver sección 1.5) o
por simetrı́a (como en 1.4.1; ver sección 1.6). frecuentemente es el resultado
de ’trabajar hacia atrás’ (ver sección 1.8) o de argumentar por contradicción
(como 1.4.3; ver sección 1.9). No es extraño considerar un problema más
general en el contexto (como 1.4.2; ver sección 1.12). De esta manera ve-
mos que la modificación de un problema es una heurı́stica muy general. Por
esta razón, haremos a un lado más ejemplos y problemas, dejándolos más
apropiadamente para las secciones más especializadas que siguen.
3.5. ELECCIÓN DE UNA NOTACIÓN APROPIADA 145

3.5. Elección de una Notación Apropiada

Uno de los primeros pasos al trabajar sobre un problema de matemáticas


es traducir el problema a términos simbólicos. Al empezar, todos los con-
ceptos claves deberian ser identificados y etiquetados; las redundancias en la
notación pueden ser eliminadas al ir descubriendo las relaciones.

3.5.1 Una mañana comenzó a nevar de manera fuerte y constante. Una


removedora de nieve comenzó a las 8:00am. A las 9:00am se habı́a alejado 2
millas. A las 10:00am se habı́a alejado 3 millas. Suponiendo que la máquina
remueve un volumen constante de nieve por hora, determine la hora en que
comenzó a nevar.

Solución. Es difı́cil creer que hay suficiente información en el problema para


resolver la pregunta. Sin embargo, si hay una solución, debemos proceder sis-
temáticamente identificando primero aquellas cantidaddes que son descono-
cidas. Introducimos la siguiente notación: sea t el tiempo que ha transcurrido
desde que comenzó a nevar, y sea T el tiempo en la que la removedora sale
(medido desde t = 0) . Sea x(t) la distancia que ha avanzado la removedora
al tiempo t (sólo estamos interesados en x(t) para t ≥ T ). Finalmente, sea
h(t) la profundidad de la nieve en el tiempo t.

Estamos ahora listos para traducir el problema a términos simbólicos. El


hecho de que la nieve caiga a razón constante significa que la profundidad
está incrementándose constantemente; esto es,
dh
= c, c constante.
dt

Integrando cada lado se obtiene


h(t) = ct + d, c, d constantes.

Ya que h(0) = 0, obtenemos d = 0. Ası́ h(t) = ct.

El hecho de que la removedora quite la nieve a una razón constante significa


que la velocidad de la removedora es inversamente proporcional a la profundi-
dad en cualquier tiempo t (por ejemplo, el doble de profundidad corresponde
a la mitad de la velocidad).
146 CAPÍTULO 3. HEURÍSTICA

Simbólicamente, para t ≥ T ,

dx k
dt
= h(t) , k constante
k K
= ct
= t, K = kc constante.

Integrando ambos lados se obtiene

x(t) = Klogt + C, C constante.

Tenemos tres condiciones: x = 0 cuando t = T , x = 2, cuando t = T + 1,


y x = 3, cuando t = T + 2. Con estas dos condiciones podemos evaluar las
constantes K y C, y con la tercera, podemos resolver para T . Resulta que
(los detalles no son de intéres aquı́)


5−1
T = ≈ 0,618 hrs. ≈ 37 minutos, 5 segundos.
2
Ası́, comenzó a llover a las 7:22:55 AM.

3.5.2 (a) Si n es un entero positivo tal que 2n+1 es un cuadrado perfecto,


muestre que n + 1 es la suma de dos cuadrados perfectos sucesivos.

(b) Si 3n + 1 es un cuadrado perfecto, muestre que n + 1 es la suma de tres


cuadrados perfectos

Solución. Introduciendo la notación adecuada esto se reduce a un problema


sencillo de álgebra. Para la parte (a), suponga que 2n + 1 = s2 , s es un
entero. Ya que s2 es un número impar, también lo es s. Sea t un entero tal
que s = 2t + 1. Entonces 2n + 1 = (2n + 1)2 , y resolviendo para n tenemos
que
(2t + 1)2 − 1 4t2 + 4t
n= = = 2t2 + 2t.
2 3
Consecuentemente,

n + 1 = 2t2 + 2t + 1 = t2 + (t + 1)2
3.5. ELECCIÓN DE UNA NOTACIÓN APROPIADA 147

(b) Suponga que 3n+1 = s2 , s es un entero. Evidentemente , s no es múltiplo


de 3, ası́ que s = 3t ± 1 para algún entero t. Entonces 3n + 1 = (3t ± 1)2 , y
ası́

(3t ± 1)2 − 1 9t2 ± 6t


n= = = 3t2 ± 2t.
2 3

De esta manera,

n + 1 = 3t2 ± 2t + 1 = 2t2 + (t ± 1)2 = t2 + t2 + (t ± 1)2

3.5.3 En el triángulo ABC, AB = AC, D es el punto medio de BC, E es


el pie de la perpendicular dibujada desde D a AC, y F es el punto medio de
DE (figura 1.15). Demuestre que AF es perpendicular a BE.

Solución. Podemos transformar el problema a términos algebráicos aso-


ciando coordenadas a los puntos relevantes y mostrando que las pendientes
mBE y mAF son recı́procos negativos.

Una forma de proceder es tomar el triángulo como aparece en la figura 1.15:


tomar D como el origen (0, 0), A = (0, a), B = (b,0). Esta es una forma nat-
ural de señalar la figura porque aprovecha la simetrı́a bilateral del triángu-
lo isóseles (ver los ejemplos en la sección 1.6). Sin embargo; en este caso
en particular, esta notación lleva a algunas complicaciones menores cuando
buscamos las coordenadas de E y F.
148 CAPÍTULO 3. HEURÍSTICA

b
A

b
E
F b

C
b

B
b

D
b

Figura 1.15
B
3
b

2
b

D
1
F
b

b b b

A 1 2 3 E 4C
Figura 1.16
−1
Una mejor asignación de coordenadas es tomar A = (0, 0), B = (4a, ab),
C = (4c, 0), como en la figura 1.16. Entonces a2 + b2 = c2 , D = (2a + 2c + 2b),
E = (2a + 2c, 0), y F = (2a + 2c + b). (casi no hay cálculos aquı́; todos los
puntos relevantes tienen coordenadas asignadas). Se sigue que

  
b 4b b2
mBE mAF = = 2 = −1,
2 (a + c) 4a − (2a + c) a − c2
y la prueba queda concluida.

3.5.4 Sea −1 < a0 < 1 y defı́nase recursivamente

(1 + an−1 )1/2
an = , n > 0.
2
Sea An = 4n (1 − an ) . Qué pasa con An cuando n tiende a infinito?
3.5. ELECCIÓN DE UNA NOTACIÓN APROPIADA 149

Solución. Intentar expresar directamente an en términos de a0 lleva a com-


plicadas expresiones sin esperanza que contiene sucesiones anidadas de radi-
cales, y no hay forma de condensarlas en una forma cerrada.

La introspección clave requerida es observar que hay un único ángulo θ,


0 < θ < π, tal que a0 = cos θ. Para este θ,

  21  
1 + cos θ θ
a1 = = cos .
2 2

Similarmente,
  12    
1 + cos (θ/2) θ θ
a2 = = cos , ..., an = cos .
2 4 2n

Ahora podemos calcular

 
n θ
An = 4 1 − cos n
2
θ
 
n
4 1 − cos 2n 1 + cos 2θn
= 
1 + cos 2θn

4n sin2 2θn
= 
1 + cos 2θn
!  !2
θ2 sin 2θn
=  .
1 + cos 2θn θ
2n

Conforme n crece, θ2 /(1 + cos(θ/2n )) tiende a θ2 /2, y (sen(θ/2n ))/(θ/2n ) se


aproxima a 1 (recuerde que senx/x → 1 conforme x → 0), y ası́ , An converge
a θ2 /2 conforme n tiende a infinito.

Problemas
150 CAPÍTULO 3. HEURÍSTICA

3.5.5 Escriba una ecuación para representar las siguientes afirmaciones:

(a) En el restaurante Mindy, por cada cuatro personas que ordenaron pastel
de queso, hubo cinco que pidieron pay de manzana.

(b) Hay seis veces la cantidad de profesores en estudiantes en este colegio.

3.5.6 Unos cables son extendidos desde la punta de cada uno de los polos a
la base del otro. Cuál es la altura medida desde el piso al punto en que estos
alambres se cruzan?

3.5.7 Una pieza de papel de 8 pulgadas de ancho es doblada como en la


figura 1.17 de forma que una esquina es puesta en el lado opuesto. Exprese
la longitud del doblez L en términos del ángulo θ solamente.

3.5.8 Sean P1 , P2 , ....., P12 los vértices sucesivos de un dodecaedro regular


(12 lados). Concurren las diagonales P1 P9 , P2 P11 , P4 P12 .?

8
Figura 1.17
3.5. ELECCIÓN DE UNA NOTACIÓN APROPIADA 151

3.5.9 Utilice álgebra para respaldar sus respuestas a cada uno de los sigu-
ientes.

Un carro viaja desde A hasta B a razón de 40 millas por hora y luego regresa
de B a A a razón de 60 millas por hora? 50 millas por hora es apróximada-
mente la velocidad promedio par el viaje redondo?

Le dan a usted una taza de café y una de crema que contienen la misma
cantidad de lı́quido. Una cucharada de la mezcla es extraı́da de la taza y se
devuelve a la de crema. Hay más o menos crema ahora en la taza de café que
café en la taza de crema? ( Este problema tiene una elegante solución no
algebráica basda en la observación de que el café en la taza de crema ha
desplazado una cantidad igual de crema que debe estar ahora en la taza de
café.)

Imagine que la tierra es una esfera suave y que se le enreda una cuerda
alrededor del Ecuador. Suponga ahora que la cuerda es alargada 6 pies y que
la nueva longitud es empujada al parejo para formar un nuevo cı́rculo justo
encima del Ecuador. Es la distancia entre la cuerda y la superficie de la tierra
más o menos de 1 pulgada?

3.5.10 Un teorema bien conocido afirma que un primo p > 2 puede ser es-
crito como la suma de dos cuadrados perfectos (p = m2 + n2 , con m y n enteros)
si y sólo si p es mayor por 1 que un múltiplo de 4. Dando este resultado por
cierto, demuestre que:

Todo primo mayor por 1 que un múltiplo de 8 puede ser escrito en la forma
x2 + 16y 2, x y y enteros.

Todo primo mayor por 5 que un múltiplo de 8 puede ser escrito en la forma
(2x + y)2 + 4y 2, x y y enteros.

Ejemplos adicionales

1.1.10, 2.5.10, 3,2,15, 3.3.11, 3.3.28, 3.4.2, .3.4.4, 4.1.5, 6.4.2, 7.2.4, 8.1.15,
8.2.3, 8.2.17. También, ver las Secciones 2.5 (Relaciones de Recurrencia), 3.2
152 CAPÍTULO 3. HEURÍSTICA

(Aritmética Modular), 3.4 (Notación Posicional), 8.3 (Geometrı́a Vectorial),


8.4 (Geometrı́a en los Números Complejos).

3.6. Explotación de la Simetrı́a

La presencia de simetrı́a de un problema provee usualmente de medios para


reducir la cantidad de trabajo para llegar a una solución. Por ejemplo, con-
siderese el producto (a + b + c) (a2 + b2 + c2 − ab − ac − bc) . Ya que cada
factor es simétrico en a, b, c (la expresión no cambia cuando cualquier par de
sus variables son intercambiadas), lo mismo sucederá con el producto. Como
resultado de ello, si a3 aparece en el producto, también aparecerán b3 y c3 .
Similarmente, si a2 b aparece en el producto, también lo harán a2 c, b2 a, b2 c, c2 b,
y cada uno aparecerá con el mismo coeficiente, etc. De esta manera, una re-
visada rapida muestra que el producto tendrá la forma

 
A a3 + b3 + c3 + B a2 b + a2 c + b2 c + c2 a + c2 b + C (abc) .

Es fácil checar que A = 1, B = 0, y C = −3.

3.6.1 Se construyen triángulos equilateros ABK, BCL, CDM, DAN den-


tro del cuadrado ABCD. Demuestre que los puntos medios de los cuatro
segmentos KL, LM, MN, NK y los puntos medios de los ocho segmentos
AK, BK, BL, CL, CM, DM, DN, AN son los 12 vértices de un dodecaedro
regular.
3.6. EXPLOTACIÓN DE LA SIMETRÍA 153

Db b
C

b
K

b b b

a b b

b b

L b b
O b
N
b b

b b

b b

A b b

B
Figura 1.18

Solución. Los doce vértices están indicados en la Figura 1.18 con puntos
gruesos; dos de estos vértices están señalados como a y b según se muestra.

Utilizando la simetrı́a de la figura, basta mostrar que ∠bOK = 15◦ , ∠aOb =


30◦ , y |aO| = |bO| .

Notese que AN es parte del bisector perpendicular de BK, y entonces |KN| =


|NB| . Utilizando simetrı́a que sigue que ∠CBN = 15◦ . Considérese ahora
el triángulo equilátero DBN; nótese que Ob une los puntos medios de DB
y DN, ası́ qu Ob es la paralela a BN y de la mitad de longitud. De esta
manera |Ob| = s/2 y ∠bOK = 15◦ . Desde aquı́ es fácil verificar que
b

∠aOb = ∠DOK − ∠bOK = 45◦ − 15◦ = 30◦ , y |Oa| = |KN| /2 = s/2.

La presencia de simetrı́a en un problema también provee de visión clara


que nos permite ver y descubir relaciones que pueden ser más difı́ciles de
encontrar por otros medios. Por ejemplo, considerando sólo la simetrı́a se
ocurre que el valor máximo de xy, bajo la restricción x + y = 1, x > 0,
y > 0, deberı́a ocurrir cuando x = y = 1/2 (x y y están simétricamente
154 CAPÍTULO 3. HEURÍSTICA

relacionadas). Este es un ejemplo del principio de razón insuf iciente, que


puede resumirse de la siguiente manera: ”Donde no hay suficiente razón para
distinguir, no puede haber distinción.” Ası́, no hay razón para esperar que el
valor máximo ocurra cuando x tiene el valor distinto de 1/2, esto es, cercano
a 0 o a 1. Para verificar esto, sea x = 1/2 + e. Entonces y = 1/2 − e, y,
xy = (1/2 + e) (1/2 − e) = 1/4 − e2 . De esta forma es claro que el máximo
ocurre cuando e = 0; esto es, x = y = 1/2.

El siguiente problema ofrece varios Ejemplos adicionales de este principio.

3.6.2 .

(a) De todos los rectángulos que pueden inscribirse en un cı́rculo dado, cuál
tiene mayor área?

b b

x
b b

Figura 1.19

(b) Maximizar senA + senB + senC, donde A, B, C son las medidas de los
tres ángulos de un triángulo.

(c) De todos los triángulos con un perı́metro dado, cuál tiene mayor área?

(d) De todos los paralelepı́pedos de volumen 1, cuál tiene la menor área de


superficie?

(e) De todos los n − ágonos que pueden ser inscritos en un cı́rculo dado, cuál
tiene mayor área?
3.6. EXPLOTACIÓN DE LA SIMETRÍA 155

Solución. .(a) El principio de razón insuficiente nos lleva a sospechar que el


rectángulo con mayor área que puede inscribirse en un cı́rculo es un cuadrado
(Figura 1.19). Para verificar esto, sean x y y el largo y el ancho del rectángu-
lo, y supóngase sin pérdida de generalidad que las unidades han sido escogidas
de manera que el diámetro del cı́rculo es la unidad. Queremos maximizar xy
sujetos a que x2 + y 2 = 1. Esto es equivalente a maximizar x2 y 2 bajo la
restricción x2 + y 2 = 1. Pero este es el mismo problema que se consideró pre-
viamente a este ejemplo; el valor máximo ocurre cuando x2 + y 2 = 1/2, esto
es, cuando el rectángulo es un cuadrado.

(b) Nótese que la suma, senA + senB + senC, es siempre positiva (ya que
cada uno de los términos es positivo), y que puede ser hecha arbitrariamente
pequeña (en magnitud) haciendo A arbitrariamente cercano a 180◦ . No hay
razón para esperar que el máximo ocurra en otro punto que no sea A = B =
C = 60◦ (un triángulo equilátero). Una demostración de esto se sigue de la
discusión en 2.4.1.

De manera similar, sospechamos que las respuestas a (c),(d), y (e) son un


triángulo equilátero, un cubo, y un n − ágono regular.

3.6.3 Evaluar

Z π
2 dx

2
0 1 + (tan x)
156 CAPÍTULO 3. HEURÍSTICA

1
1/2

pi/41 pi/2 2

Figura 1.20

Solución. . He aquı́ un problema que no puede resolverse utilizando las


técnicas usuales de integración; es decir, el integrando no tiene una an-
tiderivada. Sin embargo, el problema puede ser manejado si notamos que
el integrando (Figura 1.20) es simétrico alrededor del punto (π/4, 1/2) .
 √ 
Para mostrar que esto es ası́ (no es obvio), sea f (x) = 1/ 1 + (tan x) 2 .
√ + f (π/2 − x) = 1 para toda x, 0 ≤ x ≤ π/2. Ası́,
Basta mostrar que f (x)
calculamos, para r = 2,

1 1
f (π/2 − x) + f (x) = +
1 + tan (π/2 − x) 1 + tanr (x)
1 1
= +
1 + cot (x) 1 + tanr (x)
r

tanr (x) 1
= +
1 + tan (x) 1 + tanr (x)
r

= 1.

Se sigue de la simetrı́a recientemente demostrada que el área bajo la curva


en [0, π/2] es la mitad de área en el rectángulo (ver Figura 1.20); esto es, la
3.6. EXPLOTACIÓN DE LA SIMETRÍA 157

integral es (π/2) /2 = π/4.

Otra manera de sacar ventaja de la simetrı́a es la elección de notación. He


aquı́ un par de ilustraciones.

3.6.4 Sea P un punto en la gráfica de y = f (x), donde f es un polinomio


de grado tres; supóngase que la tangente en P intersecta de nuevo a la curva
en Q; y sea A el área de la región limitada por la curva y el segmento P Q.
Sea B el área de la región definida si comenzamos por Q en vez de P. Cuál
es la relación entre A y B?.

P
b

−2 −1 x0 1Q
b

−1

−2

−3

R
b

−4
Figura 1.21

Solución. . Sabemos que un polinomio cúbico es simetrı́co respecto a su


punto de inflexión. Ya que las áreas de interés no son afectadas por la elección
del sistema de coordenadas, fijaremos el origen en el punto de inflexión. Ası́,
podemos suponer que la función cúbica es

f (x) = ax3 + bx, a 6= 0

(ver Figura 1.21).

Supóngase que x0 es la abcisa de P. Resulta que la abcisa de Q es −2x0 .


(No nos detendremos en los detalles de estos cálculos tan directos. Existe,
158 CAPÍTULO 3. HEURÍSTICA

ciertamente, una manera elegante de llegar a este hecho, pero utiliza ideas
que no se verán en esta sección).

Integrando directamente se muestra que el área A es igual a Kx40 , donde K


es independiente de x0 . (De nuevo no nos detendremos en los detalles del
cálculo.)

Podemos aplicar ahora nuestras conclusiones previas al punto Q. La tan-


gente en Q intersectará a la curva en R, la abcisa del cual es evidentemente
−2 (−2x0 ) = 4x0 , y el área B es igual a K (−2x0 )4 = 16Kx40 = 16A.

3.6.5 Determine todos los valores de x que satisfacen

tan x = tan (x + 10◦ ) tan (x + 20◦ ) tan (x + 30◦) .

Solución. . Introduciremos simetrı́a haciendo un simple cambio de variable.


Ası́, sea y = x + 15◦ . La ecuación es entonces

tan (y − 15◦ ) = tan (y − 5◦ ) tan (y + 5◦ ) tan (y + 15◦ ) ,


que equivale a

sen (y − 15◦ ) cos (y + 15◦ ) sen (y − 5◦ ) sen (y + 5◦ )


=
cos (y − 15◦ ) sen (y + 15◦ ) cos (y − 5◦ ) cos (y + 5◦ )
usando las identidades

1
sen (A) cos (B) = [sen (A − B) + sen (A + B)] ,
2
1
sen (A) sen (B) = [cos (A − B) − cos (A + B)] ,
2
1
cos (A) cos (B) = [cos (A − B) + cos (A + B)] ,
2
3.6. EXPLOTACIÓN DE LA SIMETRÍA 159

obtenemos
sen (−30◦ ) + sen2y cos (−10◦ ) − cos 2y
= ,
sen (30◦ ) + sen2y cos (−10◦ ) + cos 2y
o equivalentemente,
2sen2y − 1 cos 10◦ − cos 2y
= .
2sen2y + 1 cos 10◦ + cos 2y
Esto se simplifica en
sen (4y) = cos 10◦ ,
4y = 80◦ + 360◦ k, 100◦ + 360◦ k, k = ±0, ±1, ±2, ... ,
lo cual implica que
x = 5◦ + 90◦ k, 10◦ + 90◦ k, k = ±0, ±1, ±2, ... .

Problemas

3.6.6 .

(a) Explote la simetrı́a para expandir el producto


 
x2 y + y 2 z + z 2 x xy 2 + yz 2 + zx2 .

(b) Si x + y + z = 0, demuestre que


 2  5 
x + y2 + z2 x + y5 + z5 x7 + y 7 + z 7
=
2 5 7

(substituya z = −x − y y aplique el teorema del binomio.)


160 CAPÍTULO 3. HEURÍSTICA

3.6.7 Las caras de cada una de las quince monedas, acomodadas como se
muestra en la Figura 1.22, son pintadas de blanco o de negro. Demuestre
que exiten tres monedas del mismo color cuyos centros son los vértices de un
triángulo equilátero. (Hay muchas maneras de explotar la simetrı́a y de crear
argumentos del tipo ”sin pérdida de generalidad”.)

Figura 1.22

3.6.8 Haga uso del principio de razón insuficiente para minimizar x21 +
x22 + ... + x2n , bajo la condición de que 0 < xi < 1, y x1 + x2 + ... + xn = 1.
Demuestre su conjetura (Para la demostración, tome xi = n1 + ei .)

3.6.9 Un punto P está localizado en el interior de un triángulo equilátero


ABC. Desde P se trazan perpendiculares que intersectan a cada lado en los
puntos D, E, y F , respectivamente. Dónde deberı́a estar P para que P D +
P E + P F sea un máximo? Dónde deberı́a estar P para que P D + P E + P F
sea un mı́nimo? Justifique sus respuestas. (Sugerencia: Es de gran ayuda
reflejar la figura respecto a uno de los lados. Qué pasa con P D + P E + P F
conforme P se mueve paralelamente a la lı́nea de reflexión?)

3.6.10 En la Figura 1.23, ABCD es un cuadrado, ∠EDC = ∠ECD =


15◦ . Demuestre que el triángulo AEB es equilátero. (La clave para este bello
problema es crear simetrı́a central. Especı́ficamente, agregar ángulos idénticos
de 15◦ sobre los lados AB, BC, y AD (como en el lado CD) y crear un
diagrama muy parecido al construido en 1.6.1.)

3.6.11 El producto de cuatro términos consecutivos de una serie aritmética


de enteros más la cuarta potencia de la diferencia común es siempre un
3.7. DIVISIÓN EN CASOS 161

cuadrado perfecto. Verifique esta identidad incorporando simetrı́a en la no-


tación.
B b
C b

b E

b b

A D
Figura 1.23

Ejemplos adicionales

1.4.1, 8.1.4, 8.1.5, 8.1.8, 8.2.3.

3.7. División en Casos

Sucede con frecuencia que un problema pueda ser dividido en un número


pequeño de subproblemas, cada uno de los cuales puede ser manejado por
separado a la manera de caso por caso. Esto es cierto especialmente cuando el
problema contiene un cuantificador universal (’para todo x ...’). Por ejemplo,
la demostración de una proposición de la forma ’para todos los enteros ...’
puede hacerse argumentando para los pares y los impares por separado. De
forma similar, un teorema sobre triángulos puede probarse dividiéndolo en
tres casos dependiendo de si el triángulo es agudo, recto u obtuso. Ocasional-
mente, los subproblemas pueden ser arreglados jerárquicamente en submetas,
de forma que los primeros casos, una vez establecidos, pueden ser utilizados
para probar las siguientes etapas. Tal procedimiento es llamado escalada.
162 CAPÍTULO 3. HEURÍSTICA

En las primeras etapas del análisis, es útil pensar en cómo subdividir un


problema en un número pequeño de subproblemas (con suerte) más sencillos.
La heurı́stica de esta sección se da muy con frecuencia de la siguiente manera:
’Si no puede resolver el problema, encuentre un problema relacionado más
sencillo y resuélvalo.’

3.7.1 Demuestre que un ángulo inscrito en un cı́rculo es igual a la mitad


del ángulo central que subtiende el mismo arco.
A A A B
b b

B
b

O
O
b b b

O
b
B P
b

P
b

P
b

Figura 1.24

Solución. Tenemos un cı́rculo, con centro O digamos, y un ángulo inscrito


AP B; se muestran algunos ejemplos en la figura 1.24. Tenemos que demostrar
que para todos los casos ∠AP B = 12 ∠AOB. Las tres figuras precedentes
representan situaciones esenciales diferentes. Especı́ficamente, el centro del
cı́rculo, O, está en el interior de ∠AP B (diagrama 2), o fuera de ∠AP B
(diagrama 3), o en uno de los rayos de ∠AP B (diagrama 1). Probaremos el
teorema considerando cada uno de estos casos por separado.

Caso1. Supongamos que el centro O está en P A. Entonces ∠AOB = ∠OP B+


∠OBP (un ángulo exterior es igual a la suma de los ángulos interiores no
adyasentes) = 2∠DP B (∆OP B es isósceles)= 2∠AP B. El resultado es in-
mediato.

Caso2. Si O está en el interior del ∠AP B (diagrama 2), extendemos la lı́nea


P O hasta cortar el cı́rculo en D. Acabamos de demostrar que 2∠AP B =
∠AOD y 2∠DP B = ∠DOB. La suma de estas ecuaciones dará el resultado
deseado.

Caso 3. Si O es exterior ∠AP B (diagrama 3), extendemos P O hasta cortar el


cı́rculo en Ḋ. Entonces, utilizando el caso 1, 2∠DP B = ∠DOB y 2∠DP A =
3.7. DIVISIÓN EN CASOS 163

∠DOA. Restando la segunda ecuación de la primera se obtiene el resultado


deseado. Esto completa la demostración.

3.7.2 Una función real de variable racional f satisface

f (x + y) = f (x) + f (y)

para todos x y y racionales. Demuestre que f (x) = f (1) · x para todo x


racional.

Solución. Procedemos por pasos. Primero probaremos el resultado para los


enteros positivos, luego los negativos, luego los recı́procos de los enteros, y
finalmente para todos los números racionales.

Caso 1 (Enteros positivos). El resultado se cumple cuando x = 1. Para x = 2,


tenemos f (2) = f (1 + 1) = f (1) + f (1) = 2f (1) . Para

x = 3, f (3) = f (2 + 1) = f (2)+f (1) = 2f (1)+f (1) = 3f (1) . Es claro que


este proceso puede ser continuado, y que para cualquier entero positivo n,
f (n) = nf (1) . (Puede darse una demostración formal basada en el principio
de inducción matemática, ver Capı́tulo2).

Caso 2 (Enteros no positivos). Primero, f (0) = f (0 + 0) = f (0) + f (0) .


Restando f (0) de ambos lados se obtiene 0 = f (0) ; esto es f (0) = 0 · f (1) .
Ahora, 0 = f (0) = f (1 + (−1)) = f (1) + f (−1) . De aquı́ que f (−1) =
f (1) . De manera similar, para cualquier entero positivo n, f (n) + f (−n) =
f (n + (−n)) = f (0) = 0, de forma que f (−n) = −nf (1) .

Caso 3(Recı́procos).
 Para x = 21 , procedemos como sigue: f (1)= f 21 + 21
= f 21 + f 21 = 2f 12 . dividiendo entre 2 se obtiene f 21 = f (1) /2.
Para x = 31 ,
    
f (1) = f 31 + 13 + 31 = f 31 + f 31 + f 31 = 3f 31 , o equivalentemente, 
1
f (1) /3. De forma similar, para  cualquier
 entero positivo
 n, f n
= f (1) /n.
Para x = −1n
, tenemos f n
1
+ f −1
n
= f 1
n
+ −1
n
= f (0) = 0, de forma
−1
que f n = −f (1) /n.
2
 1 1

Caso 4 (todos los racionales).
 2 Sea n un entero. Entonces f n
= f n
+ n
=
1 1 1 m
f n +f n = 2f n = n f (1) . De forma similar, si n es cualquier racional,
164 CAPÍTULO 3. HEURÍSTICA

com m un entero positivo y n un entero, entonces


m        
1 1 1 1 1 m
f =f + ... + =f + ... + f = mf = f (1) .
n n n n n n n
| {z } | {z }
m−veces m−veces

Esto establece el resultado, un buen ejemplo de escalada.

3.7.3 Demuestre que el área de un triángulo retı́cula es igual a I + 12 B − 1,


donde I y B representan respectivamente el número de puntos interiores y de
frontera de la retı́cula en el triángulo. (Un triángulo retı́cula es un triángulo
en el plano con puntos de una retı́cula como vértices.)
b
C C b
C b

B
b

b b b b b

A 1 B A 2 B A 3

b
C b
C

B b

B
b b

A 4 A 5
Figura 1.25

Solución. Este es un caso especial del teorema de Pick (ver 2.3.1.). Hay
varias pruebas ingeniosas, cada una de las cuales divide al conjunto de los
triángulos retı́cula en unos pocos de tipos especiales. Una forma de hacer esto
es ’circunscribir’ sobre el triángulo un rectángulo con bordes paralelos a los
ejes coordenados. Al menos un vértice del rectángulo debe coincidir con un
vértice del triángulo. Ahora es fácil ver que cada triángulo retı́cula puede ser
clasificado en una de las clases no equivalentes esbozadas en la Figura 1.25.

En la primera clase están aquellos triángulos rectos cuyos catetos son par-
alelos a los ejes coordenados. La segunda clase incluye triángulos de ángulos
3.7. DIVISIÓN EN CASOS 165

agudos uno de cuyos lados es paralelo a un eje coordenado. Tales triángulos


son la ‘suma’ de dos triángulos de la primera clase. En la tercera clase están
los triángulos obtusos con un lado paralelo a un eje coordenado. Ellos son
la ’resta’ de dos triángulos de la primera clase. Las clases cuarta y quin-
ta abarcan a aquellos triángulos que no tienen lados paralelos a los ejes de
coordenadas.

La demostración del resultado sigue un patrón de escalada. Para empezar,


consideremos el rectángulo ABCD del caso 1. Supongamos que los segmentos
AB y AD contienen a y b puntos de la retı́cula, respectivamente, sin contar
sus extremos. Entonces, con I y B los puntos interiores y de frontera de
ABCD,
1 1
I + B − 1 = ab + (2a + 2b + 4) − 1
2 2
= ab + a + b + 1
= (a + 1) (b + 1)
= Area ABCD

Ahora supongamos que AB, BC, y AC contienen a, b, y c puntos de la retı́cu-


la, respectivamente, sin contar sus extremos, y supongamos que ABC con-
tiene i puntos interiores. Entonces el rectángulo ABCD tiene 2i + c puntos
interiores, y tenemos, con I y B los puntos interiores y frontera de ABC,

1 1
I + B − 1 = i + (a + b + c + 3) − 1
2 2
1
= (2i + a + b + c + 1)
2 
1 1
= (2i + c) + (2a + 2b + 4) − 1
2 2
1
= Area ABCD
2
= Area ABC.
Los otros casos pueden ser manejados de forma similar; dejamos los detalles
al lector.

Problemas
166 CAPÍTULO 3. HEURÍSTICA

3.7.4 ( Desiguladad del triángulo ).

(a) Demuestre que para cualesquiera números reales x y y, |x + y| ≤ |x| +


|y| .

(b) Demuestre que para cualesquiera números reales x, y, y z, |x − y| ≤


|x − z| + |y − z| .

3.7.5 Encuentre todos los valores de x que satisfacen

3 2
< .
x−1 x+1

3.7.6 Sean S = {i (3, 8) + j (4, −1) + k (5, 4) / i, j, k son enteros}, y T =


{m (1, 5) + n (0, 7) / m, n son enteros} . Demuestre que S = T. (Nota: las
parejas ordenadas de enteros se suman componente a componente: (s, t) +
(s′ , t′ ) = (s + s′ , t + t′ ) , y n (s, t) = (ns, nt) .)

3.7.7 Una función real f, definida en los números racionales positivos, sat-
isface f (x + y) = f (x) f (y) para cualesquiera x y y racionales positivos.
Demuestre que f (x) = [f (1)]x para todo racional positivo x.

3.7.8 Determine F (x) si, para cualesquiera reales x y y, F (x) F (y) +


F (xy) = x + y.

Ejemplos adicionales

1.1.7,2.5.11c,2.5.12,2.5.13,2.6.3, 3.2.14, 3.2.15, 3.2.16, 3.2.17, 3.2.18, 3.4.1,


4.1.3, 4.1.4, 4.4.14, 4.4.29, 5.2.1, 5.3.14c, 6.5.4, 7.4.3, 7.6.2, 7.6.4, 7.6.10, 8.2.4.
Algunos ejemplos particulares que se reducen a estudiar casos muy especiales
son 3.3.8, 3.3.9, 3.3.21, 3.3.22, 3.3.26.
3.8. TRABAJO HACIA ATRÁS 167

3.8. Trabajo Hacia Atrás

Trabajar hacia atras significa dar cierta conclusión y entonces obtener de-
duciones de la conclusión hasta que llegamos al algo conocido o algo que
puede demostrarse fácilmente. Después de que llegamos a lo dado o a lo
conocido, entonces invertimos los pasos del argumento y procedemos hacia
la conclusión.

Este procedimiento es común en álgebra y trigonometrı́a de secundaria. Por


ejemplo, para encontrar todos los números reales que satisfacen 2x + 3 = 7,
argumentos como sigue. Supongamos que x satisface 2x + 3 = 7. Entonces,
restamos 3 de cada lado de la ecuación y dividimos cada lado entre 2, para
obtener x = 2. Ya que cada paso en esta deducción puede ser invertido,
concluimos que 2 ciertamente satisface 2x + 3 = 7 y es el único número que
lo hace.

Con frecuencia, en manipulaciones de rutina, tales como las del ejemplo pre-
vio, no se hace una reescritura explı́cita de los pasos. Sin embargo, es im-
portante estar atento a lo que
√ puede, √ y no puede, ser invertido. Por ejemp-
lo, considerese la ecuación x + 1 − x − 1 = 2. (Aquı́, como es usual, la
raı́z √
cuadrada es
√ la raı́z cuadrada positiva.) Escriba la ecuación en la for-
ma x + 1 = x√− 1 + 2, y eleve ambos lados al√cuadrado para obtener
x + 1 = x − 1 + 4 x − 1 + 4, o equivalentemente, x − 1 = −1/2. Eleve al
cuadrado una vez más para obtener x−1 = 1/4, o x = 5/4. Sin embargo, 5/4
no satisface la ecuación original. La razón de esto es que √
no todos los pasos
son reversibles. Ası́, en este ejemplo, procedemos desde x − 1 = −1/2 a
x − 1 = 1/4. Cuando
√ invertimos esto, sin embargo, el argumento va desde
x − 1 = 1/4 a x − 1 = 1/2.

3.8.1 Sea α un número real fijo, 0 < α < π, y sea


senθ + sen (θ + α)
F (θ) = , 0 ≤ θ ≤ −α.
cos θ − cos (θ + α)
Demuestre que F es constante. (Este problema surgió en 1.2.1.)

Solución. Supongamos que F es constante. Entonces F (θ) = F (0) para


todo θ, 0 ≤ θ ≤ −α. Esto es,
168 CAPÍTULO 3. HEURÍSTICA

senθ + sen (θ + α) senα


= , (3.1)
cos θ − cos (θ + α) 1 − cos α
[senθ + sen (θ + α)] [1 − cos α] = senα [cos θ − cos (θ + α)] , (3.2)
senθ + sen (θ + α) − senθ cos α − sen(θ + α) cos α
= senα cos θ − senα cos (θ + α) , (3.3)
senθ + sen (θ + α) − [senθ cos α + senα cos θ]
− [sen(θ + α) cos α − senα cos (θ + α)] = 0, (3.4)
senθ + sen (θ + α) − sen (θ + α) − sen (θ + α − α) = 0. (3.5)
La última ecuación es una identidad. Para la demostración, debemos invertir
los pasos. El único paso cuestionable es de ?? a ??: la prueba sólo es válida si
no dividimos por cero al ir de ?? a ??. Pero (1 − cos α) 6= 0 ya que 0 < α < π,
y cos θ − cos(θ + α) > 0 ya que 0 ≤ θ < θ + α ≤ π. La demostración puede
entonces llevarse a cabo; esto es, comenzando por la identidad conocida en
??, podemos argumentar (vı́a los pasos ??,??,??,??) que para todo θ, 0 ≤
θ ≤ π − α, F (θ) = senα/ (1 − cos α) = constante.

3.8.2 Si a, b, c denotan las longitudes de los lados de un triángulo, demuestre


que
3 (ab + bc + ca) ≤ (a + b + c)2 ≤ 4 (ab + bc + ca) .
Solución. Considerese la desigualdad del extremo izquierdo:

3 (ab + bc + ca) ≤ (a + b + c)2 ,


3 (ab + bc + ca) ≤ a2 + b2 + c2 + 2 (ab + bc + ca) ,
(ab + bc + ca) ≤ a2 + b2 + c2 ,
a2 + b2 + c2 − (ab + bc + ca) ≥ 0
2a2 + 2b2 + 2c2 − 2 (ab + bc + ca) ≥ 0
  
a2 − 2ab + b2 + b2 − 2bc + c2 + c2 − 2ca + a2 ≥ 0
(a − b)2 + (b − c)2 + (c − a)2 ≥ 0

Esta desigualdad es verdadera para todos los valores de a, b, c. Considerese


ahora la desigualdad de la derecha:
3.8. TRABAJO HACIA ATRÁS 169

(a + b + c)2 ≤ 4 (ab + bc + ca)


a2 + b2 + c2 + 2 (ab + bc + ca) ≤ 4 (ab + bc + ca)
a2 + b2 + c2 ≤ 2 (ab + bc + ca)
a2 + b2 + c2 ≤ a (b + c) + b (a + c) + c (b + a)

Esta desigualdad final es verdadera, ya que la suma de cualesquiera dos de los


lados de un triángulo es mayor que el lado restante. Ası́, a2 ≤ a (b + c) , b2 ≤
b (a + c) , y c2 ≤ c (b + a) .

Los pasos en cada uno de estos argumentos pueden ser invertidos, ası́ que la
prueba está completa.

3.8.3 Se tiene: AOB es el diámetro del cı́rculo O; BM es tangente al cı́rculo


en B; CF es tangente al cı́rculo en E e intersecta BM en C; la cuerda AE,
al ser prolongada, intersecta BM en D. Demuestre que BC = CD. (Ver la
Figura 1.26)
A b

a b

b
O
b
E
c d

b e
M
b b b b

B C D
Figura 1.26

Solución. Supongamos que BC = CD. Entonces CE = CD, ya que BC =


CE (tangentes al cı́rculo desde C en B y E son iguales). Ası́, ∠CED =
∠CDE (los ángulos base de un triángulo isósceles son iguales). Esto nos
lleva a considerar los ángulos de la manera en que están etiquetados en la
Figura 1.26.
170 CAPÍTULO 3. HEURÍSTICA

Ahora bien, ∠D es el complemento de ∠a ya que el ∆ABD es un triángulo


rectángulo, y el ∠e es complemento del ∠c ya que ∠BEA es un ángulo recto
(AOB es un diámetro). Ası́, ∠a = ∠c. Pero sabemos que ∠a = ∠c, ya que
ambos subtienden el mismo arco BE en el cı́rculo O.

La prueba puede ser completada invirtiendo los pasos. De esta manera (omiti-
mos las razones),partimos de que ∠a = ∠c, y por lo tanto, ∠e = ∠d.
Ası́ CD = CE, CE = BC, y por lo tanto BC = CD.

3.8.4 En un torneo con n jugadores P1 , P2 , ..., Pn , con n > 1, cada jugador


juega un juego con cada uno de los otros jugadores y las reglas son de tal
manera que no puede haber empates. Sean Wr y Lr el número de juegos
ganados y perdidos, respectivamente, por el jugador Pr . Demuestre que
n
X n
X
Wr2 = L2r .
r=1 r=1

Pn Pn
Solución. Supongamos que r=1 Wr2 = r=1 Wr2 . Entonces,

n
X 
Wr2 − L2r = 0,
r=1
n
X
(Wr − Lr ) (Wr + Lr ) = 0.
r=1

Pero Wr + Lr = n − 1 para cada r, ası́ que

n
X
(n − 1) (Wr − Lr ) = 0,
r=1
Xn
(Wr − Lr ) = 0,
r=1
n
X n
X
Wr = Lr .
r=1 r=1
3.8. TRABAJO HACIA ATRÁS 171

Esta última ecuación es verdadera, ya que el número total de juegos ganados


por n jugadores es igual al número total de juegos perdidos. La demostración
se sigue al invertir el argumento precedente.

Problemas

3.8.5 .

(a) Dados números positivos reales x y y, demuestre que


2 √ x+y
≤ xy ≤ .
1/x + 1/y 2

(b) Dados a y b, números reales positivos tales que a + b = 1, demuestre


que
2
a
≤ ax + by, x > 0, y > 0.
x
+ yb

3.8.6 .

(a) Si a, b, c son números reales positivos, y a < b + c, muestre que

a b c
< + .
1+a 1+b 1+c

(b) Si a, b, c son las longitudes de tres segmentos que pueden formar un


triángulo, muestre que lo mismo se vale para 1/ (a + c) , 1/ (b + c) , 1/ (a + b) .

3.8.7 Dos cı́rculos son tangentes externamente en A, y una tangente común


externa los toca en B y C. El segmento BA se prolonga hasta intersectar al
segundo cı́rculo en D. Demuestre que CD es un diámetro.
172 CAPÍTULO 3. HEURÍSTICA

3.8.8 Considerese el siguiente argumento. Suponga que θ satisface


cot θ + tan 3θ = 0.
Entonces, como
tan α + tan β
tan (α + β) = ,
1 + tan α tan β
se sigue que
tan θ + tan 2θ
cot θ + = 0
1 − tan θ tan 2θ
cot θ (1 − tan θ tan 2θ) + tan θ + tan 2θ = 0
cot θ − tan 2θ + tan θ + tan 2θ = 0
cot θ + tan θ = 0
1 + tan2 θ = 0
tan2 θ = −1.

Ya que la última ecuación no puede ser cierta, la ecuación original no tiene


solución (no necesitamos invertir ningún paso porque el paso final no lleva a
nada). Sin embargo, θ = 14 π si satisface cot θ + tan 3θ = 0. Qué está mal en
la argumentación?

3.8.9 Con las herramientas Euclidianas (regla y compás), inscriba un cuadra-


do en un triángulo dado de manera que un lado del cuadrado quede en un
lado dado del triángulo. (Sugerencia: Comience con el cuadrado y construya
un triángulo alrededor de él semejante al triángulo dado. Luego haga uso del
hecho de que figuras semejantes tienen partes proporcionales.)

Ejemplos adicionales

2.1.5, 7.1.1, 7.4.6. También ver la Sección 2.2 (Inducción) y la Sección 2.5
(Recurrencia).

3.9. Argumentación por Contradicción

Arguentar por contradicción significa suponer que la conclusión es falsa y


entonces obtener deducciones hasta que lleguemos a algo que sea contradic-
3.9. ARGUMENTACIÓN POR CONTRADICCIÓN 173

torio, ya sea con lo que está dado (el método indirecto) o a lo que se sabe que

es verdadero (reductio ad absurdum) . Ası́, por ejemplo, para probar que 2
es irracional, podemos suponer que es racional y proceder hasta obtener una
contradicción. El método es con frecuencia apropiado cuando la conclusión
es fácil de negar, cuando las hipótesis ofrecen poco material susceptible de
ser manipulado, o cuando se tienen pocas ideas sobre cómo proceder.

Como un ejemplo sencillo de este método de demostración, considere la sigu-


iente argumentación que muestra que la serie armónica diverge. Supóngase,
por el contrario, que converge, a r digamos. Entonces

1 1 1 1 1 1 1
r = 1+ + + + + + + + ...
2 3 4 5 6 7 8
1 1 1 1 1 1 1 1
> + + + + + + + + ...
2 2 4 4 6 6 8 8
1 1 1
= 1 + + + + ...
2 3 4
= r,

una contradicción. Nos vemos forzados a concluir que la serie diverge.

3.9.1 Dado que a, b, c son enteros impares, demuestre que la ecuación ax2 +
bx + c = 0 no puede tener raı́ces racionales.

Solución. Supongamos que p/q es una raı́z racional, donde (sin perdida de
generalidad) p y q son enteros pares. Primero estableceremos que ninguno
entre p y q puede ser par. Supongamos que p es par. De ax2 + bx + c = 0
tenemos que ap2 + bpq + cq 2 = 0. Ya que ap2 + bpq es par, cq 2 debe ser par,
pero esto es imposible, ya que c y q son ambos impares. Obtenemos una
contradicción similar si suponemos que q es par. Por lo tanto, ambos p y q
son impares y ap2 + bpq + cq 2 = 0. Pero esta última ecuación establece que
la suma de tres números impares es igual a cero, lo cual es imposible. Por lo
tanto, la ecuación no tiene raı́ces racionales.

Es ilustrativo considerar otra demostración de este resultado. Las raı́ces de


ax2 + bx + c = 0 son racionales si y sólo si b2 − 4ac es un cuadro perfecto.
Ası́, supongamos que b2 −4ac = (2n + 1)2 para algún entero n (por hipótesis,
174 CAPÍTULO 3. HEURÍSTICA

b2 − 4ac es impar, y por lo tanto, si es un cuadrado, debe ser el cuadrado de


un entero impar). Agrupando múltiplos de 4 tenemos que

b2 − 1 = 4[n (n + 1) + ac].

Ya que ni n ni n + 1 son pares, n (n + 1) + ac es impar. Ası́, el lado derecho


de la última ecuación es divisible por 4 pero no por 8. Sin embargo, el lado
izquierdo es divisible por 8, ya que b2 − 1 = (b − 1) (b + 1) y entre b − 1 y
b + 1 es divisible por 4, mientras que el otro es divisible por 2. Por lo tanto
la ecuación antes escrita no puede ser cierta, y tenemos una contradicción.
(En esta demostración, hemos llegado a una contradicción al examinar como
se relacionan dos números respecto a múltiplos de 8, más que respecto a
múltiplos de 2 como en la primera demostración.)

En las dos secciones siguientes se dan Ejemplos adicionales de demostración


por contradicción.

Problemas

3.9.2 En una fiesta con 2000 personas, en cada grupo de 4 hay al menos una
persona que conoce a las otras tres. Hay tres personas que no se conocen entre
sı́. Demuestre que las otras 1997 conocen a todo mundo en la fiesta. (Suponga
que ’conocer’ es una relación simétrica; esto es, si A conoce B entonces B
también conoce a A. Cuál es la respuesta si ’conocer’ no necesariamente es
una relación simétrica?)

3.9.3 Demuestre que no existen enteros positivos a, b, c y n tales que a2 +


b + c2 = 2n abc. (De 1.4.3, podemos suponer que a y b son impares y c es
2

par. Cómo son los lados de la ecuación respecto a 4?)


3.9. ARGUMENTACIÓN POR CONTRADICCIÓN 175

3.9.4 Cada par de comunidades de un condado están enlazadas por exac-


tamente un medio de transporte: camión, tren o avión. Los tres medios son
utilizados en el condado; ninguna de las comunidades cuenta con los tres
medios simultáneamente, y ninguna tercia de comunidades está enlazada por
parejas por el mismo medio. Cuatro comunidades pueden estar conectadas
de acuerdo a lo estipulado de la siguiente forma: camión, AB, BC, CD, DA;
tren, AC; avión, BD.

(a) Dé un argumento para mostrar que ninguna comunidad puede tener
un medio de transporte en especial que la conecte a tres comunidades
diferentes.

(b) Dé una demostración que muestre que cinco comunidades no pueden
ser conectadas de acuerdo a lo estipulado.

3.9.5 Sea S un conjunto de números racionales que es cerrado bajo la suma


y producto (esto es, si a y b son elementos de S, también lo son a + b y ab),
y que tiene la propiedad de que para cada número racional r se cumple una
y sólo una de las siguientes afirmaciones: r ∈ S, −r ∈ S, r = 0.

(a) Demuestre que 0 no es elemento de S.

(b) Demuestre que todos los enteros positivos son elementos de S.

(c) Demuestre que S es el conjunto de todos los números racionales posi-


tivos.

Ejemplos adicionales

1.5.10,1.6.7, 3.2.1, 3.2.6, 3.2.11, 3.2.13, 3.2.15, 3.2.17, 3.2.18, 3.3.4, 3.3.11,
3.3.15, 3.3.28, 3.4.2, 4.1.3, 4.4.6, 5.4.1. También ser Sección 1.10 (Paridad) y
la Sección 1.11 (Casos Extremos).
176 CAPÍTULO 3. HEURÍSTICA

3.10. Búsqueda de Paridad

La sencilla idea de paridad, ser par o ser impar, es un concepto poderoso en la


resolución de problemas con una gran cantidad de aplicaciones. Revisaremos
algunos ejemplos en está sección.

3.10.1 Sean nueve puntos de una retı́cula en el espacio euclidiano de tres


dimensiones. Muestre que hay un punto de la retı́cula en uno de los segmentos
que unen dos de estos puntos.

Solución. Sólo hay ocho patrones de paridad para estos puntos de la retı́cu-
la: (par, par, par), (par, par, impar),...,(impar, impar, impar). Ya que hay
nueve puntos dados, dos de ellos deben tener el mismo patrón de paridad.
Su punto medio es un punto de la retı́cula, y ası́ queda terminada la de-
mostración.

3.10.2 Coloque un peón en cada silla de un tablero de ajedrez de 7 por 7.


Es posible que cada peón haga simultáneamente un movimiento permitido?

Solución. Supongamos que un tablero está iluminado de la forma usual. El


tablero tiene 49 casillas; supongamos que 24 de ellas son balncas y 25 son
negras.

Consideremos los 25 peones que están en las casillas negras. Si cada uno de
ellos hiciera un movimiento permitido, tendrı́an que moverse a 25 casillas
blancas. Sin embargo, sólo hay 24 casillas blancas disponibles, por lo que
dicha jugada no puede llevarse a cabo.

3.10.3 Coloque un peón sobre el tablero de ajedrez de 4 por n. Es posible,


en 4n movimientos consecutivos del peón, visitar cada casilla del tablero y
regresar a la casilla de partida?
3.10. BÚSQUEDA DE PARIDAD 177

Figura 1.27

Solución. Antes de considerar este problema, es interesante analizar la mis-


ma pregunta para un tablero de 7 por 7. Suponga que se intenta dicho ’paseo
cerrado’. En el primer movimiento el peón se mueve a una casilla de color
opuesto; en el segundo regresa a una casilla del mismo color; y ası́ se sigue. Ve-
mos que después de un número impar de movimientos el peón ocupará una
casilla del color opuesto al de su casilla original. Un paseo cerrado por el
tablero de 7 por 7 requiere de 49 movimientos, un número impar. Ası́, el
peón no puede regresar a su casilla original, y el paseo cerrado es imposible.

Consideremos ahora el tablero de 4 por n. La argumentación utilizada para


el de 7 por 7 no puede ser aplicada en este caso, porque 4n es un número
par. Para manejar este caso, iluminemos el tablero de la forma que se indica
en la Figura 1.27.

observemos que los movimientos del peón hechos desde una casilla blanca
en las filas superior e inferior llevan a casillas blancas en las filas segunda y
tercera. Inversamente, en un paseo como el requerido, los movimientos del
peón desde las dos filas interiores llevan a casillas blancas en las dos filas
exteriores. Esto se debe a que hay exactamente n casillas blancas en las filas
exteriores, y éstas sólo pueden ser alcanzadas desde las n casillas blancas
en las filas inteiores. Ası́ pues, el camino del peón nunca puede pasar de las
casillas blancas a las negras, y por lo tanto el paseo cerrado es imposible.

3.10.4 Sea n un entero impar mayor que 1, y sea A una matriz simétrica
de n por n tal que cada renglón y cada columna son una permutación de los
enteros 1, ..., n. Muestre que cada uno de los enteros 1, ..., n debe aparecer en
la diagonal de A.
178 CAPÍTULO 3. HEURÍSTICA

Solución. Los elementos fuera de la diagonal están por pares ya que A es


simétrica. Cada número aparece exactamente n veces, y esto, junto con el
hecho de que n es impar, no lleva al resultado.

3.10.5 Sea a1 , a2 , ..., a2n+1 un conjunto de enteros con la siguiente propiedad


(P ) : si alguno de ellos es eliminado, los restantes pueden ser separados en
dos conjuntos de n enteros con sumas iguales. Demuestre que a1 = a2 = ... =
a2n+1 .

Solución. Primero, observamos que todos los enteros a1 , a2 , ..., a2n+1 tienen
la misma paridad. Para ver esto, sea A = a1 + a2 + ... + a2n+1 . La afirmación
se sigue después de observar que para cada i, A + ai es par (de otra forma
los números restantes no podrı́an ser separados de la manera requerida).

Sea a el número más pequeño de a1 , a2 , ..., a2n+1 , y para cada i, sea bi = ai +a.
El problema es equivalente a mostrar que bi = 0 para toda i.

Ahora bien b1 , b2 , ..., b2n+1 cumplen la propiedad (P ) . Ya que ninguno de


ellos es cero, positivo más grande tal que 2k divide a cada bi . Para cada i,
sea ci = bi /2k . Entonces c1 , c2 , ..., c2n+1 cumplen (P ) ; sin embargo, no todos
tienen la misma paridad ( ya que uno de ellos es cero, y otro es impar dada
la elección de k). Por lo tanto, todos los bi son cero y la demostración queda
terminada.

Problemas

3.10.6 .

(a) Quite la casilla inferior izquierda y la superior derecha de un tablero


normal de ajedrez de 8 por 8. Se puede cubrir el tablero resultante con
31 fichas de dominó ? Suponga que cada ficha cubre exactamente dos
casillas adyacentes del tablero.

(b) Sean P1 , ..., P13 puntos del plano y suponga que estan unidos por los
segmentos P1 P2 , P2 P3 , ..., P12 P13, P13 P1 . Es posible trazar una lı́nea recta
que pase por el interior de cada uno de estos segmentos?
3.10. BÚSQUEDA DE PARIDAD 179

3.10.7 .

(a) Es posible trazar una trayectoria que pase por los arcos de la Figu-
ra 1.28(a) que atraviese cada arco una vez y sólo una ? (Sugerencia:
Cuente el número de arcos que salen de cada vértice.)
(b) Es posible trazar una trayectoria por las lı́neas de la Figura 1.28(b)
que pase por cada unión una y sólo una vez? (Sugerencia: Ilumine los
vértices de forma alternada.)

b b

b b

(a)
b

b
b b

b b

b b

b b b

b b
b

(b)
Figura 1.28

3.10.8 Sea a1 , a2 , ..., an un arreglo arbitrario de los números 1, ...., n. De-


muestre que, si n es impar, el producto

(a1 − 1) (a2 − 2) ... (an − n)


180 CAPÍTULO 3. HEURÍSTICA

es un número par.


3.10.9 Muestre que (2a − 1) 2b − 1 = 2c + 1 es imposible para enteros no
negativos a, b, y c. (Sugerencia: Escriba la ecuación de la forma equivalente
2a+b − 2a − 2b = 2c e investigue las posibilidades de a, b y c.)

3.10.10 Muestre que x2 − y 2 = a3 siempre tiene soluciones enteras para x


y y siempre que a sea un número positivo.

Ejemplos adicionales

1.5.10,1.9.1, 2.2.7, 3.2.13, 3.3.4, 3.3.20, 4.2.16(a), 4.3.4, 7.4.6. Ver sección 3.2
para una generalización de este método.

3.11. Consideración de Casos Extremos

En las primeras etapas de exploración de un problema, con frecuencia es de


gran ayuda considerar las consecuencias de variar los parámetros del proble-
ma de un valor extremo a otro. En esta sección veremos que la existencia de
posiciones extremas son frecuentemente la clave para entender resultados de
existencia (problemas del estilo ’pruebe que existe x tal que P (x)’).

3.11.1 Dado un número finito de puntos en un plano, no todos colineales,


pruebe que existe una lı́nea recta que pasa exactemente por dos de ellos.
P b

b b b

Q
b

P3 P2 P1
Figura 1.29

Solución. Si P es un punto y L una lı́nea, sea d (P, L) la distancia desde


P a L. Sea S el conjunto de las distancias positivas d (P, L) conforme P
3.11. CONSIDERACIÓN DE CASOS EXTREMOS 181

varı́a por los puntos dados, y L varı́a por las lı́neas que no pasan por P pero
que si pasan por al menos dos de los puntos dados. El conjunto S es no
vacı́o (porque los puntos dados no son todos colineales ) y finito (hay sólo
un número finito de puntos y un número finito de rectas que pasan por al
menos dos puntos ). Por lo tanto S tiene un elemento al minimal, digamos
d (P, M) . Afirmamos que M pasa exactamente por dos de los puntos dados.

Supongamos que M pasa por tres de los puntos dados, digamos P1 , P2 y P3 .


Sea Q el punto de M más cercano a P. Al menos dos de los puntos P1 , P2
y P3 están del mismo lado de Q (uno de ellos podrı́a ser Q), digamos P2 y
P3 (ver Figura 1.29). Supongamos que los puntos están etiquetados de forma
que P2 está más cerca de P que P3 . Sea N la lı́nea através de P y P3 , y
observamos que d (P2 , N) < d (P, M), una contradicción respecto a nuestra
elección de P y M. Se sigue que M sólo puede pasar por dos de los puntos
dados.

Supongamos que M pasa por tres de los puntos dados, digamos P1 , P2 y P3 .


Sea Q el punto de M más cercano a P. Al menos dos de los puntos P1 , P2 , P3
están del mismo lado que Q (uno de llos podrı́a ser Q ), digamos P1 y P3 (ver
Figura 1.27). Supongamos que los puntos están etiquetados de forma que P2
está más cerca de P que P3 . Sea N la lı́nea através de P y P3 , y observemos
que d (P2 , N) < d (P, N) , una contradicción respecto a nuestra elección de P
y M. Se sigue que M sólo púede pasar por dos de los puntos dados.

3.11.2 Sea A un conjunto de 2n puntos en el plano, de los cuales no hay


tres que sean colineales. Suponga que n de ellos son rojos y los otros n son
azules. Pruebe o refute: Existen n segmentos de lı́nea recta cerrados, de los
cuales no hay dos que tengan un punto en común, tales que los extremos de
cada segmento son puntos de A de distinto color.

Solución. Si ignoramos las intersecciones de lı́neas, existe una variedad de


formas en las que los puntos rojos pueden ser apareados con lo azules con n
segmentos de lı́nea cerrados. Asignemos a cada uno de estos apareamientos
la longitud total de los segmentos de lı́nea en la configuración. Como sólo
hay un número finito de tales apareamientos, una de estas configuraciones
tendrá una longitud total minimal. Este apareamiento no tendrá cruces entre
lı́neas. (Si los segmentos R1 B1 y R2 B2 se intersectan, con R1 , R2 puntos rojos
y B1 , B2 puntos azules, entonces podrı́amos reducir la longitud total de la
182 CAPÍTULO 3. HEURÍSTICA

configuración reemplazando estos segmentos por R1 B2 y R2 B1 .)

3.11.3 En una fiesta, ningún muchacho baila con todas las muchachas, pero
cada muchacha baila al menos con un muchacho. Pruebe que existen dos
parejas bg y b′ g ′ que bailan, considerando que si b no baila con g ′ , tampoco g
baila con b′ .

Solución. Aunque no es necesario, el problema podrı́a ser más compren-


sible si lo ponemos en términos matriciales. Hagamos que los renglones de
una matriz correspondan a los muchachos y las columnas a las muchachas.
Escribamos un 1 o un 0 en el renglón b y la columna g dependiendo de si b
y g bailan juntos o no. La condición de que no hay un muchacho que baile
con todas las muchachas implica que (i) cada renglón tiene al menos una
entrada 0. Similarmente, (ii) cada columna tiene al menos una entrada 1.
Queremos probar que existen dos renglones, b y b′ , y dos columnas, g y g ′,
cuyas entradas de intersección tienen el patrón

   
.. .. .. ..
. . . .
   
 ··· 1 ··· 0 ···   ··· 0 ··· 1 ··· 

 .. .. 
 o

 .. .. 

 . .   . . 
 ··· 0 ··· 1 ···   ··· 1 ··· 0 ··· 
   
.. .. .. ..
. . . .

Sea h un renglón arbitrario. Por (i) hay una entrada 0 en este renglón, dig-
amos en la columna k, y por (ii) hay una entrada 1 en la columna k, digamos
en el renglón m :

k
 
.. ..
. .
 
 ··· 1 ··· 0 ··· 
h 
 .. .. 

 . . 
 
m  · · · 0? · · · 1 · · · 
.. ..
. .
3.11. CONSIDERACIÓN DE CASOS EXTREMOS 183

Ahora bien, ya que los hicimos si hay una columna que tenga 1 en el renglón
h y 0 en la columna m, En general, tal columna podrı́a existir. Sin embargo,
si h hubiera sido escogido con anticipación como renglón con un número
maximal de 1’s, entonces tal columna no hubiera existido y el problema no
hubiera sido resuelto.

Con estos antecedentes, podemos reescribir la solución en lenguaje indepen-


diente de la notación matricial. Sea b un muchacho que baila con un número
maximal de muchachas. Sea g ′ una muchacha con quien b no baila, y b′ un
muchacho con el que g ′ baila. Entre los compañeros de b, debe haber al menos
una muchacha g que no baile con b′ (de otra forma b′ tendrı́a más compañeras
que b ). Entonces las parejas bg y b′ g ′ resuelven el problema.

3.11.4 Demuestre que el producto de n enteros sucesivos siempre es divisible


por n!.

Solución. Primero, observamos que es suficiente demostrar el resultado para


n enteros positivos sucesivos. El resultado es obviamente cierto si uno de los
enteros es 0, mientras que si todos los enteros son negativos, basta mostrar
que n! divide a su valor absoluto.

Ası́ pues, supongamos que existen n enteros sucesivos cuyo producto no


es divisible por n!. De todos estos n números, escojamos al más pequeño;
llamémosle N. Observemos que N > 2, ya que el producto de dos enteros
sucesivos siempre es par. Estamos suponiendo, por lo tanto, que existe un
número no negativo m tal que (m + 1) (m + 2) ... (m + n) no es divisi-
ble por N!. De todos estos números m, sea M el más chico. Observemos
que M > 0, ya que N! es divisible por N!. Ası́, estamos suponiendo que
(M + 1) (M + 2) ... (M + N) no es divisible por N!. Ahora,

(M + 1) (M + 2) ... (M + N − 1) (M + N)
= M[(M + 1) (M + 2) ... (M + N − 1)] +
N[(M + 1) (M + 2) ... (M + N − 1)]

Por la forma en que escogimos M, N! divide a M [(M + 1) (M + 2) ...


(M + N − 1)]. Por la forma en que escogimos N, (N − 1)! divide a (M + 1) (M + 2) ...
184 CAPÍTULO 3. HEURÍSTICA

(M + N − 1), y en consecuencia N! divide a N [(M + 1) (M + 2) ... (M + N − 1)].


Combinando, vemos que N! divide al lado derecho de la última ecuación, en
contradicción con nuestra suposición. Esta contradicción establece el resul-
tado.

(Una prueba ingeniosa de este resultado esidentificar


 al cociente (m + 1) (m + 2) ...
m+n
(m + n) /n! como el coeficiente binomial , y por lo tanto es un en-
n
tero si m es un entero.)

Problemas

3.11.5 Sea f (x) un polinomio de grado n con coeficientes reales y tal que
f (x) ≥ 0 para todo número real x. Muestre que f (x) + f ′ (x) + ... + f (n) (x)
para todo real x. ( f (k) (x) denota la k-ésima derivada de f (x) . )

3.11.6 Dé un ejemplo para mostrar que el resultado de 1.11.1 no necesari-


amente se cumple para un número infinito de puntos en el plano. En dónde
falla la demostración de 1.11.1 para el caso infinito?

3.11.7 Muestre que existe un número racional, c/d, con d < 100, tal que
h c i  73 
k = k para k = 1, 2, ..., 99.
d 100

3.11.8 Suponga que Pn es una afirmación, para n = 1, 2, 3, ... . Suponga


además que

(i) P1 es cierto, y

(ii) para cada entero positivo m, Pm+1 es cierto si Pm es cierto.


3.12. GENERALIZACIÓN 185

Demuestre que Pn es cierto para todo n. (Sugerencia: Sea S el conjunto de


todos los enteros positivos para los cuales Pn no es cierto. Sea m el elemento
más pequeño en S, suponiendo que S es no vacı́o.)

Ejemplos adicionales

3.1.9, 3.3.11, 3.3.28, 4.4.7, 4.4.10, y los referentes dados en 6.3.7. También, ver
sección 7.6 (El Principio del Emparedado) y 6.2 (El teorema del valor inter-
medio) para ejemplos que requieren la consideración de casos ”extremales”.

3.12. Generalización

Puede parecer paradójico, pero con frecuencia se presenta el caso en que un


problema puede ser simplificado, y hecho accesible y comprensible, cuando
se le generaliza. Este hecho de la vida es muy apreciado por los matemáticos;
de hecho, la abstracción y la generalización son caracterı́sticas básicas de la
matemática moderna. Un contexto más general provee de una perspectiva
más amplia, hace a un lado los rasgos no esenciales, y proporciona todo un
arsenal de técnicas nuevas.

Pn k2
3.12.1 Evalúe la suma k=1 2k .
P
Solución. Evaluaremos
 en cambio la suma S (x) = nk=1 k 2 xk y entonces
calcularemos S 21 . La razón para introducir la variable x es que ahora
podemos utilizar técnicas del análisis. Sabemos que

n
X 1 − xn+1
xk = , x 6= 1.
k=1
1−x

Derivando ambos lados obtenemos


186 CAPÍTULO 3. HEURÍSTICA

n
X (1 − x) (− (n + 1) xn ) + (1 + xn+1 )
kxk−1 =
k=1
(1 − x)2
1 − (n + 1) xn + nxn+1
= .
(1 − x)2

Multiplicando ambos lados de está ecuación por x, derivando una segunda


vez, y multiplicando el resultado por x obtenemos

n
X x (1 + x) − xn+1 (nx − n − 1)2 − xn+2
S (x) = k 2 xk = .
k=1
(1 − x)3

Se sigue que

  n  2
1 X k2 1 1 1
S = k
= 6 − n−2 n − n − 1 − n−1
2 k=1
2 2 2 2
 2 
n + 4n + 6
= 6− .
2n

3.12.2 Evalúe el siguiente determinante (determinante de Vandermonde):

 
1 a1 a21 · · · a1n−1

 1 a2 a22 · · · a2n−1 

det  .. .. .. .. .
 . . . . 
1 an a2n · · · ann−1

Solución. Supondremos que ai 6= aj , i 6= j, porque de otra forma el de-


terminate serı́a cero. Con el objeto de enfocarnos más claramente en la idea
principal, consideraremos el caso n = 3 :
3.12. GENERALIZACIÓN 187

 
1 a a2
det  1 b b2  .
1 c c2

En este determinante, reemplacemos c por una variable x. Entonces el de-


terminate es un polinomio P (x) de grado 2. Además, P (a) = 0 y P (b) = 0,
ya que la matriz correspondiente, con c sustituı́da por a o b respectivamente,
tiene entonces dos renglones idénticos. Por lo tanto,

P (x) = A (x − a) (x − b)
para alguna constante A. Ahora, A es el coeficiente de x2 , y, regresando al
determinante, encontramos que este coeficiente es

 
1 a
det .
1 b

Ası́ A = b − a, y el determinante original de 3 por 3 es

P (c) = (b − c) [(c − a) (c − b)].

El caso general es análogo. Sea Dn el determinante buscado (de orden n


). Sustituimos an en el último renglón de la matriz por la variable x. El
determinante resultante es un polinomio Pn (x) de grado n − 1, que se hace
cero en a1 , a2 , ..., an−1 . Luego, por el Teorema del Factor,

Pn (x) = A (x − a1 ) (x − a2 ) ... (x − an−1 ) ,


donde A es constante. Como en el caso anterior, A es el coeficiente de xn , y
expandiendo sobre el último renglón queda claro que A = Dn−1 . Esto es,

Dn = Pn (an ) = Dn−1 [(an − a1 ) (an − a2 ) ... (an − an−1 )].


Podemos repetir el argumento para Dn−1 ; etc. El resultado final será
188 CAPÍTULO 3. HEURÍSTICA

n
"k−1 #
Y Y
Dn = (ak − ai ) .
k=2 i=1

R∞ R∞
3.12.3 Dado que 0
(senx) /x dx = 21 π, evalúe 0
(sen2 x) /x2 dx.

Solución. Evaluaremos la integral más general

Z ∞ 
I (a) = sen2 ax /x2 dx, a ≥ 0.
0
utilizando una técnica llamada diferenciación de parámetros.

Derivando cada lado de la ecuación con respecto a a; obtenemos

Z ∞
′ 2senax cos ax · x
I (a) = dx
x2
Z0 ∞
sen2ax
= dx.
0 x
Ahora, con y = 2ax, obtenemos dy = 2adx, y

Z ∞
′ seny 1
I (a) = = π.
0 y 2
Integrando ambos lados queda

1
I (a) = πa + C. C constante.
2
1
Ya que I (0) = 0, tenemosR ∞que C2 = 0,2 Ası́ I (a)1 = 2 πa, a ≥ 0. Haciendo
a =R 1 se tiene I (1) = 0 (sen x) /x dx = 2 π. (Casualmente, el valor

de 0 (senx) /x dx puede ser encontrado al evaluar una curva en el plano
complejo.)

Problemas
3.12. GENERALIZACIÓN 189

3.12.4 Tomando x los valores apropiados en la expansión binomial


n  
X n n k
(1 + x) = x
k=1
k

(o en una de sus derivadas) evalúe cada uno de los siguientes

   
Pn n
2
Pn k n
(a) k=1 k , (b) k=1 3 ,
k  k 
Pn 1 n Pn n
(c) k=1 k+1 , (d) k=1 (2k + 1) .
k k

3.12.5 Evalúe  
1 a a2 a4
 1 b b2 b4 
det  .
 1 c c2 c4 
1 d d2 d4
(Sustituya d por una variable x; haga uso del hecho de que la suma de las
raı́ces de un polinomio de cuarto grado es igual al coeficiente de x3 .)

3.12.6 .

R∞ R∞
(a) Evalúe 0 (e−x senx) /x dx. (Considere G (k) = 0 (e−x senkx) /x dx
y utilice diferenciación paramétrica.)
R1 R1
(b) Evalúe 0 (x − 1) / ln x dx. (Considere H (m) = 0 (xm − 1) / ln x dx
y utilice diferenciación paramétrica.)

(c) Evalúe Z ∞
arctan (πx) − arctan x
dx.
0 x
R∞ arctan(ax)−arctan x
(Considere F (a) = 0 x
dx y utilice diferenciación paramétri-
ca.)
190 CAPÍTULO 3. HEURÍSTICA
√ √
3.12.7 Cuál número es más grande. 3 60 o 2 + 3 7? (Elevar al cubo cada
número lleva a complicaciones que no son de fácil resolución.
p Considere
√ en
cambio el problema más general: Cuál cantidad es mayor. 4 (x + y) o x+
3 3
√3 y, donde x, y ≥ 0? Haga x = a3 , y = b3 .)

Ejemplos adicionales

1.4.2, 2.2.6, 2.2.7, 4.1.4, 5.1.3, 5.1.4, 5.1.9, 5.1.11, 5.4.4, 5.4.5, 5.4.6, 5.4.7,
6.9.2, 7.4.4. También, ver la sección 2.4 (Inducción y Generalización)
Capı́tulo 4

Dos Importantes Principios: El


de Induccción y el del Palomar.

Las proposiciones matemáticas vienen en dos formas: proposiciones univer-


sales las cuales establecen que algo es verdadero para todos los valores de
X en algún conjunto especificado, y proposiciones existenciales las cuales
establecen que algo es verdadero para algún valor de X en algún valor es-
pecificado. Las primeras son expresables en la forma ”para todo X (en un
conjunto S), P (X)”., las del segundo tipo son expresables en la forma ”allı́ ex-
iste un X (en el conjunto S)tal que P (X)”, donde P (X) es una afirmación
a cerca de X. En este capı́tulo vanos a considerar dos importantes técnicas
para el trato con esos dos tipos de afirmaciones: (i) el principio de inducción
matemática, para proposiciones universales, y (ii) el principio del palomar
para proposiciones existenciales.

4.1. Inducción: Construcción Sobre P (k).

Sea a un entero y P (n) una proposición (afirmación) sobre n para cada entero
n ≥ a. El principio de inducción matemática establece que:

Si

191
192CAPÍTULO 4. DOS IMPORTANTES PRINCIPIOS: EL DE INDUCCCIÓN Y EL DEL

(i) P (a) es verdadero, y

(ii) para cada entero k ≥ a, P (k) verdadero implica P (k + 1) verdadero


entonces P (n) es verdadero para todo entero n ≥ a.

Nótese que el prinicipio lo habilitamos, en dos simples pasos, probar un


número infinito de proposiciones (a saber, P (n) es verdadero para todo entero
n ≥ a).

El método es especialmente conveniente cuando un modelo ha sido estable-


cido (ver sección 1.1 ”búsqueda de un modelo”) para los primeros casos
especiales (P (a), P (a + 1), P (a + 2),.....).

En esta sección vamos a considerar argumentos de inducción como, en el


paso (ii), procediendo directamente de la verdad de P (k) hacia la verdad de
P (k + 1), esto es la verdad de P (k + 1) es ”construida sobre” la consideración
inicial de la verdad de P (k).

Esto es en ligero contraste con los argumentos (considerados en la siguiente


sección) en los que se comienza con una consideración de P (k + 1).

4.1.1 Use inducción matemática para probar el teorema del binomio:


P 
(a + b)n = ni=0 ni ai bn−i , n un entero positivo.

Solución. Es fácil checar que la fórmula vale para n = 1.

Suponiendo que la fórmula vale para el entero k (estamos trabajando sobre


la verdad de P (k)), multiplicando ambos lados por (a + b) obtenemos

   
k Pn k i k−i
(a + b) (a + b) = i=0 ab (a + b)
  i  
P k i+1 k−i Pk k i k+1−i
= ki=0 a b + i=0 ab
i i

en la primera suma, hacemos el cambio de variables j = i + 1, y obtenemos.


4.1. INDUCCIÓN: CONSTRUCCIÓN SOBRE P (K). 193

   
Pk+1 k j k+1−j
Pk k i k+1−i
= j=1 ab + i=0 ab
 j − 1   i    
Pk k j k+1−j k+1
Pk k i k+1−i k+1
= j=1 ab +a + i=1 ab +b
 j − 1      i
Pk k k
= ak+1 + i=1 + ai bk+1−i + bk+1
 i −1 i
k+1
Pk k + 1 i k+1−i
=a + i=1 ab + bk+1
  i
Pk+1 k + 1 i k+1−i
= i=0 ab
i
     
k k k+1
donde hemos hecho uso de la identidad básica + =
i−1 i i
(ver sección 2.5.2). Esta es la forma para P (k + 1), entonces por inducción,
la prueba se ha completado.

4.1.2 Suponga que 0 < a1< a2 <  .... < an , y suponga ei = ±1. Probar que
Pn n+1
i=1 ei ai toma al menos valores distintos según varie ei sobre 2n
2
posibles combinaciones de signos.

Solución.
  Cuando n = 1, hay exactamente 2 valores distintos (a1 y −a1 ),
2
y = 1, el resultado es válido.
2

Supóngase que este resultado


 es verdadero cuando n = k; esto es, que

Pn k+1
i=1 ei ai tiene al menos valores distintos, y suponga que ak+1 > ak .
2   
k+2 k+1
Nosotros necesitamos generar − = k + 1 sumas adicionales.
2 2
P
Esto lo podemos conseguir en la siguiente forma: Suponga que S = ki=1 ai y
note que S + ak+1 , S + (ak+1 − 2a1 ) , ..., S + (ak+1 − 2ak−1 ) , S + (ak+1 − 2ak ) ,
son distintos y mayores que cada una de las sumas obtenidas anteriormente.
(Para ver esto note que S + (ak+1 − 2ak ) > S + (ak+1 − 2ak+1 ) = S − ak+1 ).
Alli hay k + 1 números en esa lista, entonces el resultado se sigue por induc-
ción.
194CAPÍTULO 4. DOS IMPORTANTES PRINCIPIOS: EL DE INDUCCCIÓN Y EL DEL

La inducción matemática es un método que se puede utilizar en un proble-


ma de la forma ”Probar que P (n) es verdadero para todo n ≥ a. Pero se
debe notar que la inducción también se aplica a muchos problemas donde la
cuantificación es sobre conjuntos mas generales. Por ejemplo, una proposi-
ción acerca de todos los polinomios puede ser probada por inducción sobre
el grado del polinomio. Un teorema acerca de todas las matrices puede ser
manejado por inducción sobre el tamaño de la matriz. Varios resultados ref-
erentes a proposiciones en lógica simbólica son realizados por inducción sobre
el número de conectivos lógicos en la proposición. La lista de conjuntos in-
ductivos poco comunes puede continuar indefinidamente. Nosotros estamos
satisfechos de ver exactamente dos ejemplos aqui., otros ejemplos son dis-
tribuidos a traves del libro. (ver las cuatro secciones siguientes y las listas en
los ejmplos adicionales).

4.1.3 Si V , E y F son, respectivamente, los números de vertices, aristas y


caras de un mapa plano conexo, entonces

V − E + F = 2.

Solución. El entendimiento intuitivo de los términos es este resultado es


probablemente exacto, pero para asegurarse, aqui están las definiciones.

Una red es una figura (en un plano o en el espacio) consistente de un número


finito y mayor que cero de arcos, donde dos arcos no se intersectan excep-
to posiblemente en sus puntos finales. Los puntos finales de esos arcos son
llamados vértices de la red.

Un camino en una red es una sucesión de diferentes arcos en la red que


puede ser recorrido continuamente sin repasar un arco. Una red es conexa si
cualesquiera dos diferentes vértices de la red son vertices de algún camino en
la red. Un mapa es una red, junto con una superficie la cual contiene a la
red. Si esta superficie es plana el mapa es llamado un mapa plano, los arcos
de un mapa plano son llamados aristas. Las caras de un mapa plano son las
regiones que estan definidas por las fronteras (aristas) de el mapa (el oceano
es tomado en cuenta como una cara).
4.1. INDUCCIÓN: CONSTRUCCIÓN SOBRE P (K). 195

b b b b

b b

b b b

b b
b b b

1 2 3
Figura

La figura 2.1. muestra tres ejemplos de redes conexas. Las primeras dos son
mapeos planares. En el primero, V = 4, E = 4, F = 2., en el segundo,
V = 5, E = 6, F = 3. La tercera red es un mapa no plano. Sin embargo,
si nosotros lo aplastamos sobre un plano y llamamos vertices a los nuevos
puntos de intersección, entonces tenemos V = 10, E = 20, F = 12.

Ahora volviendo a la consideración del teorema. La idea clave en la prueba


de este resultado es que es realizable porque los mapas planos conexos se
pueden construir de un vertice singular por una sucesión de las siguientes
construcciones (cada uno de los cuales es un mapeo conexo):

i)] sumando un vertice en una arista existente (. . quedando ası́ . . .).

ii)] sumando una arista a un vertice la cual parte de el y regresa a el (.


quedando ası́ ).

iii)] sumando una arista entre dos vertices existentes (: : quedando ası́ ).

iv)] sumando una arista y un vertice a otro vertice existente (. quedando


ası́ . .).

Vamos aplicar inducción sobre el número de pasos requeridos para la con-


strucción del mapeo planar conexo. Si la red consiste de un solo punto, en-
tonces V = 1, F = 1, E = 0, teniendose V − E + F = 2.

Supóngase que el resultado es valido cuando se requieren K pasos en la con-


strucción. El cambio neto para cada uno de los pasos es dado en la siguiente
tabla.
196CAPÍTULO 4. DOS IMPORTANTES PRINCIPIOS: EL DE INDUCCCIÓN Y EL DEL

Operación △V △E △F △ (V − E + F )
(i) +1 +1 0 0
(ii) 0 +1 +1 0
(iii) 0 +1 +1 0
(iv) +1 +1 0 0

Observese que la cantidad V −F +E permanece invariable cuando es tomado


el (k + 1) ésimo paso. Ası́ la prueba se completa por inducción.

4.1.4 Dado un entero positivo n y un número real x, probar que:

     
1 2 n−1
[[x]] + x+ + x+ +···+ x + = [[nx]] .
n n n

Solución. Aunque hay un parametro n entero en este problema, no podemos


usar inducción sobre n para un x fijo. También por regla no podemos usar
inducción sobre x, ya que x toma rangos sobre los números reales (pues dado
un número x real no podemos determinar el número real y mayor inmediato a
x). Por tanto no es claro como se puede aplicar la inducción a este problema.

La
h idea ies probar la fórmula para un n fijo y para todo x en el subintervalo
k (k+1)
n n
para k = 0, ±1, ±2, . . . .
 
Primero supóngase que x pertenece al subintervalo [0 1/n) entonces x + nl =
Pn−1  1

0 para todo l = 0, 1, . . . , n − 1, también que i=0 x + n
= 0. Además
[nx] = 0. Luego la fórmula es válida en el “primer” subintervalo.
 
Ahora supóngase que la fórmula es cierta en el subintervalo k−1n n
k
, donde k
es un entero positivo y x se supone un número real en este intervalo. Entonces

     
1 2 n−1
[[x]] + x+ + x+ +···+ x + = [[nx]] .
n n n
4.1. INDUCCIÓN: CONSTRUCCIÓN SOBRE P (K). 197
 
Ahora sumando 1/n a x (lo que nos da un nuevo número en nk , k+1 n
) en cada
uno de los términos, excepto en el término final, del lado izquierdo se repite

cada uno de los términos de la ecuación previa, y el término final x + n−1 n
,
nos queda [x + 1] que excede a [x] por 1. Luego reemplazando en la ecuación
previa a x por x + n1 el lado izquierdo crece en 1.

Al mismo tiempo, cuando x en [nx] es reemplazado por x + n1 , el valor se


incrementa en 1. Ya que ambos lados de la ecuación se incrementan en 1
cuando x es reemplazado por x + n1 , elresultado continua siendo verdadero
para todos los números en el intervalo nk , k+1
n
.

Por inducción la fórmula es válida para todos los valores positivos de x. Un


argumento similar muestra que es válida para todos los valores negativos de
x (reemplazando x por x − n1 ).

El siguiente ejemplo es una buena ilustración de la construcción de P (k + 1)


a partir de P (k) .

4.1.5 Si a > 0 y b > 0, entonces (n − 1) an + bn ≥ nan−1 b, n es un entero


positivo, con la igualdad solo si a = b.

Solución. El resultado es verdadero para n = 1, suponiendo que la fórmula


es verdadera para el entero k. Para construir P (k + 1) , debemos, obtener el
lado izquierdo apropiado

(i) multiplicar por a:

(k − 1) ak+1 + bk a ≥ kak b

(ii) sumando ak+1 :

kak+1 + bk a ≥ kak b + ak+1

(iii) restando bk a :
198CAPÍTULO 4. DOS IMPORTANTES PRINCIPIOS: EL DE INDUCCCIÓN Y EL DEL

kak+1 ≥ kak b + ak+1 − bk a

(iv) sumando bk+1 :

kak+1 + bk+1 ≥ kak b + ak+1 − bk a + bk+1

Estamos suponiendo que esta desigualdad es una igualdad solo si a = b. Solo


queda mostrar que kak b + ak+1 − bk a + bk+1 ≥ (k + 1) ak b con la igualdad
solo si a = b. Para ello trabajaremos hacia atrás:

kak b + ak+1 − bk a + bk+1 ≥ (k + 1) ak b


−ak b + ak+1 − bk a + bk+1 ≥ 0,
ak (a − b) + bk (b − a) ≥ 0,

ak − bk (a − b) ≥ 0


el último paso es verdadero ( ak − bk y (a − b) tienen el mismo signo) con
la igualdad se da solo si a = b, entonces la prueba se sigue por inducción.
(Observe que: este resultado es un caso especial de el significado geométrico
de una desigualdad aritmética, ver sección 7.2.).

Problemas

4.1.6 .
√ √ √ √
Use inducción para probar que 1 + 1/ 2 + 1/ 3 + · · · + 1/ n < 2 n.

Use inducción para probar que 2!4! . . . (2n!) ≥ ((n + 1)!)n .


4.1. INDUCCIÓN: CONSTRUCCIÓN SOBRE P (K). 199

4.1.7 El plano euclidiano es dividido en regiones por el trazo de un número


finito de lineas rectas. Mostrar que es posible colorear cada una de esas re-
giones de azul o rojo en una manera tal que no haya dos regiones adyacentes
que tengan el mismo color.

4.1.8 Probar que la ecuación x2 + y 2 = z n tiene una solución en los enteros


positivos (x, y, z) para todo n = 1, 2, 3, . . . . (Para una buena prueba, divida
en dos casos: n par y n impar. Para una prueba no inductiva, ver 3.5.1.).

4.1.9 Un grupo de n personas juega un torneo robin-round. Cada juego


finaliza en una victoria o una dorrota. Mostrar que es posible etiquetar a los
jugadores con p1 , p2 , p3 , . . . , pn en una manera tal que p1 derrote a p2 , p2
derrote a p3 ,. . .,pn−1 derrote a pn .

4.1.10 Si cada persona, en un grupo de n personas, es amigo de al menos


la mitad de personas en el grupo, entonces es posible sentar las n personas
en un cı́rculo de manera que cada uno este sentado solo junto a amigos.

4.1.11 Los siguientes pasos conducen a otra prueba del teorema del bi-
nomio. Conocemos que (a + x)n se puede escribir como un polinomio de gra-
do n, de manera que ahi hay constantes A0 , A1 , . . . An tales que
(a + x)n = A0 + A1 x + A2 x2 + · · · + An xn .

Use inducción para describir la ecuación que resulta tomando progresiva-


mente la k − ésima derivada de cada lado de esta ecuación (k = 1, 2, . . . , n) .

Evaluar Ak para k = 0, 1, . . . , n poniendo x = 0 en la k − ésima ecuación


obtenida en la parte (a).

4.1.12 Supóngase que f : R → R es una función para la cual f (2x − f (x)) =


x para todo x, suponga que r es un número real fijo.

Probar que si f (x) = x + r, entonces f (x − nr) = (x − nr) + r para todo


entero positivo n.
200CAPÍTULO 4. DOS IMPORTANTES PRINCIPIOS: EL DE INDUCCCIÓN Y EL DEL

Probar que si f es una función uno a uno (o sea f (x) = f (y) implica x = y
) entonces la propiedad en (a) es válida para todo entero n.

Ejemplos adicionales

1.1.2, 1.1.8, 3.2.8, 6.5.13, 7.1.4.

4.2. Inducción: Pensando sobre P (k + 1)

En esta sección vamos a considerar argumentos de inducción en los cuales


empezamos con un avance directo sobre p (k + 1) y en los cuales maniobramos
hacia atras para explotar la hipotetica verdad de P (k) . Teoricamente, los
argumentos en esta sección se puedieron todos refundar como en la sección
previa y viceversa. Sin embargo, desde un punto de vista práctico, muchas
veces es mas convincente pensar un camino, que es mejor que el otro.

n5 n4 n3 n
4.2.1 Probar que 5
+ 2
+ 3
− 30
es un entero para n = 0, 1, 2, . . . .

Solución. El resultado es obvio cuando n = 0, suponiendo que la fórmula


da un entero cuando n = k. Necesitamos probar que

(k + 1)5 (k + 1)4 (k + 1)3 (k + 1)


+ + −
5 2 3 30
es un entero. Expandiendo la fórmula

k 5 + 5k 4 + 10k 3 + 10k 2 + 5k + 1 k 4 + 4k 3 + 6k 2 + 4k + 1 k 3 + 3k 2 + 3k + 1 k + 1
+ + −
5 2 3 30
y recombinando (para hacer uso de P (k)):

 
k5 k4 k3 k    
+ + − + k 4 + 2k 3 + 2k 2 + k + 2k 3 + 3k 2 + 2k + k 2 + k .
5 2 3 30
4.2. INDUCCIÓN: PENSANDO SOBRE P (K + 1) 201

La primera parte es un entero por la hipótesis inductiva y la segunda parte es


también un entero porque es la suma de enteros. Luego la prueba se sigue por
inducción. (Observe que dificil hubiera sido llegar a la verdad de P (k + 1)
partiendo de la de P (k) .)

4.2.2 Sean a, b, p1 , p2 , . . . , pn números reales con a 6= b, Definimos f (x) =


(p1 − x) (p2 − x) (p3 − x) · · · (pn − x) . Mostrar que

 
p1 a a a ··· a a

 b p2 a a ··· a a 


 b b p3 a ··· a a 
 bf (a) − af (b)

det  b b b p4 ··· a a 
= .
 .. .. .. .. .. .. ..  b−a
 . . . . . . . 
 
 b b b b · · · pn−1 a 
b b b b ··· b pn

Solución. Este es similar a muchos problemas de determinantes que se


pueden resolver con inducción matemática. Cuando n = 1, tenemos det (p1 ) =
p1 y f (x) = (p1 − x) entonces

bf (a) − af (b) b (p1 − a) − a (p1 − b)


= = p1
b−a b−a

y el resultado se cumple.

Supóngase que la igualdad es cierta para k − 1, k > 1, considere el caso


para k números reales p1 , . . . pk . (Comenzamos por la situación para P (k) y
el método recae hacia atras sobre la verdad de P (k − 1) para completar el
paso inductivo.) Deseamos evaluar:
202CAPÍTULO 4. DOS IMPORTANTES PRINCIPIOS: EL DE INDUCCCIÓN Y EL DEL

 
p1 a a a ··· a a

 b p2 a a ··· a a 


 b b p3 a ··· a a 


det  b b b p4 ··· a a 

 .. .. .. .. .. .. .. 
 . . . . . . . 
 
 b b b b · · · pk−1 a 
b b b b ··· b pk

Restando la segunda columna a la primera (esto no hace cambiar el deter-


minante):

 
p1 − a a a a ··· a a

 b − p2 p2 a a ··· a a 


 0 b p3 a ··· a a 


det  0 b b p4 ··· a a 

 .. .. .. .. .. .. .. 
 . . . . . . . 
 
 0 b b b · · · pk−1 a 
0 b b b ··· b pk

y expandiendo la primera columna obtenemos

   
p2 a ··· a a a a ··· a a

 b p3 · · · a a 


 b p3 · · · a a 


(p1 − a) det  .. .. .. .. ..   .... . . .. .. 
. . . . . −(b − p2 ) det  . . . . . 
   
 b b ··· pk−1 a   b b ··· pk−1 a 
b b ··· b pk b b ··· b pk

Estos dos nuevos determinantes (de matrices (k − 1) por (k − 1)) son de la


forma para la cual podemos aplicar la suposición inductiva P (k − 1) . Para
hacer esto, necesitamos introducir alguna notación. Para el primer determi-
nate, establecemos F (x) = (p2 − x) (p3 − x) . . . (pk − x) y para el segundo,
establecemos G (x) = (a − x) (p3 − x) . . . (pk − x) . Entonces por la suposi-
ción inductiva,
4.2. INDUCCIÓN: PENSANDO SOBRE P (K + 1) 203

   
bF (a) − aF (b) bG (a) − aG (b)
(p1 − a) − (b − p2 ) .
b−a b−a
Pero G (a) = 0 y (p1 − a) F (a) = f (a) , y por lo tanto tenemos

bf (a) − a (p1 − a) (p2 − b) . . . (pk − b) − a (a − b) (p2 − b) . . . (pk − b) ,

b−a

bf (a) − a (p2 − b) . . . (pk − b) [(p1 − a) + (a − b)]

b−a

bf (a) − af (b)
b−a
El resultado se sigue por inducción.

Problemas

4.2.3 De una prueba para el paso inductivo en 1.1.3.

4.2.4 Para todo x en el intervalo 0 ≤ x ≤ π, probar que |sen nx| ≤ n sen


x, n un entero no negativo.

4.2.5 Suponga que Q denota el conjunto de los números racionales. Hallara


todas las funciones f : Q → Q las cuales satisfacen las dos condiciones
siguientes: (i) f (1) = 2 y (ii) f (xy) = f (x) f (y) − f (x + y) + 1 para todo
x, y en Q.
204CAPÍTULO 4. DOS IMPORTANTES PRINCIPIOS: EL DE INDUCCCIÓN Y EL DEL

4.2.6 Si a, b, c ≥ 1, probar que 4 (abc + 1) ≥ (1 + a) (1 + b) (1 + c) . (Sug-


erencia: Probar, en forma mas general, que 2n−1 (a1 a2 . . . an + 1) ≥ (1 + a1 ) (1 + a2 ) . . . (1 +

4.2.7 Dado un conjunto de 51 enteros entre 1 y 100 (inclusive), mostrar


que al menos un elemento de el conjunto debe dividir a otro elemento de el
conjunto. (Sugerencia: probar en forma más general, que la misma propiedad
se cumple cuando n + 1 enteros son elegimos de los enteros entre 1 y 2n
(inclusive).) Para una prueba no inductiva ver 2.6.1.

4.2.8 Critique la prueba dada abajo para el siguiente teorema: Una matriz
n × n de enteros no negativos tiene la propiedad de que para cualquier ele-
mento que sea cero, la suma de la fila mas la columna que contienen a dicho
elemento es al menos n. Mostrar que la suma de todos los elementos de la
matriz es al menos n2 /2.

Prueba ( ?): El resultado se cumple para n = 1. Suponiendo que el resultado


se cumple para n = k −1, considere una matriz k por k. Si no tiene elementos
iguales a cero, el resultado se cumple obviamente. Si aij = 0, la suma de la fila
i y la columna j es al menos k, por hipótesis, y la suma de los elementos en la
submatriz (k − 1) × (k − 1) obtenida al suprimir la fila i y la columna j es al
menos (k − 1)2 /2 (por hipótesis inductiva). Se sigue la suma de los elementos
en la matriz k × k es al menos (k − 1)2 /2 + k = (k 2 − 2k + 1) /2 + k =
(k 2 + 1) /2 ≥ k 2 /2. El resultado se sigue por inducción.

Ejemplos adicionales.

1.1.11, 1.12.2, 3.1.11, 4.2.21, 4.3.5, 4.3.24, 6.5.12, 6.6.1, 7.1.6, 7.1.13, 7.2.5,
7.3.5.

4.3. Inducción Forma Fuerte

Suponga que a es un entero y P (n) una proposición acerca de n para todo


entero n ≥ a. La forma fuerte de inducción matemática estable que:

si
4.3. INDUCCIÓN FORMA FUERTE 205

(i) P (a) es verdadero,

(ii) para cada entero k ≥ a, P (a) , P (a + 1) , . . . , P (k) verdaderos implica


P (k + 1) sea verdadero,

Entonces P (n) es verdadero para todo n ≥ a.

La diferencia de la forma de inducción anterior esta en que ahora esta-


mos dando una suposición fuerte en el paso (ii), a saber, podemos suponer
P (a) , P (a + 1) , . . . , P (k) , en lugar solamente de P (k), para probar P (k + 1) .
Teóricamente, las dos formas de inducción son equivalentes, pero en la prácti-
ca hay problemas los cuales son mas fáciles de resolver con esta inducción
fuerte.

4.3.1 (Teorema de Pick) Probar que el área de un polı́gono retı́cula sim-


ple (un polı́gono con puntos reticulares como vértices cuyos lados no se
cruzan) es dado por I + 12 B − 1, donde I y B denotan respectivamente
el número de puntos interiores y puntos reticulares frontera del polı́gono.

Solución. Nosotros usaremos inducción sobre el número de lados del polı́gono.


El caso de un triángulo es dado en 1.7.3. Considere, entonces, un polı́gono
reticula simple P con k lados, k > 3. polı́gono reticula simple P con k lados,
k > 3. Primero establecemos que un tal polı́gono tiene una diagonal interior.
Esto es claro si el polı́gono es convexo (equivalente, si todos

Ası́ que supóngase el ángulo interior de algún vértice, llamado V, con mas de
1800 . Entonces un rayo que nace en V y barre el interior del polı́gono debe
tocar otro vértice (de no ser ası́ el polı́gono encierra un área infinita), y esto
determina una diagonal interior D con V como un punto final.

Supóngase que nuestro polı́gono P tiene I puntos interiores y B puntos fron-


tera. La diagonal interior D divide a P en dos polı́gonos retı́culas simples
P1 y P2 con I1 y I2 puntos interiores respectivamente, también con B1 y
B2 puntos frontera respectivamente. Supóngase que hay x puntos reticulares
sobre D, excluyendo sus puntos extremos. Entonces B = B1 + B2 − 2 − 2x,
I = I1 + I2 + x.

Ahora, si A, A1 , A2 denotan las áreas de P, P1 y P2 respectivamente. Entonces,


206CAPÍTULO 4. DOS IMPORTANTES PRINCIPIOS: EL DE INDUCCCIÓN Y EL DEL

A = A1 + A2
   
1 1
= I1 + B1 − 1 + I2 + B2 − 1
2 2
1
= (I1 + I2 ) + (B1 + B2 ) − 2
2
1
= (I1 + I2 + x) + (B1 + B2 − 2x) − 2
2
1
= I + (B + 2) − 2
2
1
= I + B − 1.
2
El resultado se sigue por inducción.

Observe en este ejemplo que es el primer paso del argumento de inducción el


mas difı́cil (realizado en 1.7.3); el paso inductivo (paso (ii) ) es concepcional-
mente mas simple.

Problemas

4.3.2 .

(a) Probar que cualquier entero positivo y mayor que uno puede ser escrito
como un producto de números primos.

(b) El postulado de Bertrand’s, alguna vez postulado pero ahora un cono-


cido teorema, establece que para cualquier número x > 1. alli existe un
número primo entre x y 2x. Use este hecho para mostrar que cualquier
entero positivo puede ser escrito como una suma de distintos primos.
(Para esta prueba suponga que uno es un primo).

4.3.3 .

(a) Mostrar que cualquier entero positivo puede ser escrito como una suma
de distintos números de Fibonacci.
4.4. INDUCCIÓN Y GENERALIZACIÓN 207

(b) Suponga que k ≫ m significando esto k ≥ m+ 2. Mostrar que cualquier


entero positivo n tiene una representación de la forma n = Fk1 + Fk2 +
. . . + Fkr , donde Fki son números de Fibonacci y k1 ≫ k2 ≫ . . . ≫
kr ≫ 0.

(c) Mostrar que la representación en la parte (b) es única.

Ejemplos adicionales

3.1.1, 3.1.2, 3.1.18, 3.5.5, 6.2.3.

4.4. Inducción y Generalización

Hemos visto (en la sección 1.12) que un problema es algunas veces más fácil de
manejar cuando se reconsidera en forma mas general. Esto es verdadero tam-
bién en problemas de inducción. Por ejemplo, puede suceder que las proposi-
ciones originales P (1) , P (2) , P (3) , . . . , no contengan suficiente información
que permitan a uno llevar a cabo el paso inductivo (paso (ii) ). En este caso
es natural reformular las proposiciones en una forma más consistente, más
generalmente Q (1) , Q (2) , . . . (donde Q (n) implica P (n) para cada n), y
mirando nuevamente hacia una prueba inductiva.

4.4.1 Si A1 + · · · + An = π, 0 < Ai ≤ π, i = 1, . . . , n., entonces

π
senA1 + · · · + senAn ≤ nsen .
n
208CAPÍTULO 4. DOS IMPORTANTES PRINCIPIOS: EL DE INDUCCCIÓN Y EL DEL

b
Pi
b

Ai b
Pi+1
b

Figura 2.2

Solución. Suponga que P (k) es la proposición de el teorema para un k


dado y suponiendo que P (k) es verdadera. Para el paso inductivo, suponga
que A1 + A2 + · · · Ak + Ak+1 = π, 0 < Ai ≤ π, i = 1, . . . , k + 1. En esta forma,
no es claro como hacer uso de P (k) ., sin embargo podemos, por ejemplo,
agrupar Ak y Ak+1 juntos, ası́ que A1 + .....+ Ak−1 + (Ak + Ak+1 ) = π y
aplicando la suposición inductiva obtenemos

π
sen A1 + · · · + sen Ak−1 + sen (Ak + Ak+1 ) ≤ k sen .
k
Pero todavı́a no vemos claro que esto implique P (k + 1) :
π
sen A1 + · · · + sen Ak + sen Ak+1 ≤ (k + 1) sen .
k+1

El requisito de que las A′i s sumen π parece muy restrictivo. Considere in-
mediatamente la siguiente proposición Q (n) :

Si 0 < Ai ≤ π, i = 1, . . . , entonces

 
A1 + · · · + An
sen A1 + · · · + sen An ≤ n sen .
n

(Observe que Q (n) implica P (n) . ) Obviamente Q (1) es verdadera. Supóngase


que Q (k) es verdadera, también que 0 < Ai ≤ π, , i = 1, . . . , k + 1. Entonces
4.4. INDUCCIÓN Y GENERALIZACIÓN 209

sen A1 + · · · + sen Ak + sen Ak+1


 
A1 + · · · + Ak
≤ k sen + sen Ak+1
k
   
k A1 + · · · + Ak 1
= (k + 1) sen + sen Ak+1
k+1 k k+1
  
k 1
≤ (k + 1) sen (A1 + · · · + Ak ) + sen Ak+1
k+1 k+1
 
A1 + · · · + Ak+1
= (k + 1) sen .
k+1

(la desigualdad se mantiene en sentido favorable, en estos pasos gracias al


resultado de 1.2.12b).)

El resultado se sigue ahora por inducción.

Ahora ya podemos probar la conjetura creada en 1.6.2 e): El polı́gono de


mayor área que puede ser inscrito en un cı́rculo es el polı́gono regular, para
verificar esto, suponga que P1 , P2 , . . . , Pn , n ≥ 3, son los vertices sucesivos de
un polı́gono inscrito (inscrito en un cı́rculo de radio r). Suponga que O denota
el centro del cı́rculo; suponga que Ti denota el área del triángulo P1 OPi+1 ,
i = 1, . . . , n (establecemos que Pn+1 = P1 ); Sea Ai = ∠Pi OPi+1 (figura 2.2).
Entonces

   
1 1 1
Ti = 2 r cos Ai r sen Ai
2 2 2
1 1
= r 2 cos Ai sen Ai
2 2
1 2
= r sen Ai .
2

El polı́gono de área máxima debe satisfacer 0 < Ai < π para cada i. nuestro
resultado precedente muestra que
210CAPÍTULO 4. DOS IMPORTANTES PRINCIPIOS: EL DE INDUCCCIÓN Y EL DEL

n
X
Area del polı́gono = Ti
i=1
n n
X 1 1 X
= r sen Ai = r 2
2
sen Ai
i=1
2 2 i=1
" n #
n 2 1X
≤ r sen Ai
2 n i=1
  
1 2 2π
= n r sen .
2 n

El lado derecho representa el área de un n − ágono regular, esto completa la


prueba.

1
4.4.2 Sea f (x) = (x2 − 1) 2 , x > 1; probar que f (n) (x) > 0 para n impar y
f (n) (x) < 0 para n par.

Solución. Podemos suponer que se puede expresar f (k+1) (x) en términos


de f (k) (x) . Pero una mirada a las primeras derivadas hace que este plan
paresca imposible:

x 1
f ′ (x) = (x2 −1)1/2
, f ′′ (x) = − (x2 −1) 3/2 ,

3x 2
f ′′′ (x) = (x2 −1) 5/2 , f (iv) (x) = − (x12x +1
2 −1)7/2
,
60x3 +31x 4 2
(v)
f (x) = (x2 −1)9/2 , f (vi) (x) = − 522x(x2+266x +31
−1)11/2
.

1/2
Considerese inmediatamente la siguiente reformulación: si f (x) = (x2 − 1) ,
x > 1, entonces

gn (x)
f n (x) = ,
(x2 − 1)(2n−1)/2
donde gn (x) es un polinomio de grado n − 2, además
4.4. INDUCCIÓN Y GENERALIZACIÓN 211



 una función impar cuyos coeficientes todos

son no negativos si n es impar,
gn (x) es

 una función par cuyos coeficientes todos

son no positivos si n es par.

Esta proposición puede ser establecida por inducción (omitimos los confusos
detalles), y esto implica el resultado original.

4.4.3 Suponga que Fi denota el i-ésimo término en la suseción de fibonacci.


2
Probar que Fn+1 + Fn2 = F2n+1 .

Solución. La fórmula vale para n = 1, también suponiendo que la fórmula


vale para el entero k > 1, entonces

2 2
Fk+2 + Fk+1 = (Fk+1 + Fk )2 + Fk+1
2
2
= Fk+1 + 2Fk+1 Fk + Fk2 + Fk+1
2
2 2 2
= (Fk+1 + Fk ) + (2Fk+1Fk + Fk+1 )
2
= F2k+1 + (2Fk+1Fk + Fk+1 ),

El último paso uso la hipótesis inductiva.


2
Habremos terminado si podemos mostrar que 2Fk+1 Fk + Fk+1 = F2k+2 , podi-
2
endo entonces continuar los argumentos previos, F2k+1 + (2Fk+1Fk + Fk+1 )=
F2k+1 + F2k+2 = F2k+3 , y esto completa el paso inductivo. Por lo tanto, que-
2
da por probar que 2Fk+1 Fk + Fk+1 = F2k+2 . Procederemos por inducción. La
fórmula es cierta si n = 1, suponiendo que es cierta para n = k, entonces
tenemos:

2
2Fk+2 Fk+1 + Fk+2
2
= 2(Fk+1 + Fk )Fk+1 + Fk+2
2 2
= 2Fk+1 + 2Fk+1 Fk + Fk+2
2 2
= (2Fk+1 Fk + Fk+1 ) + (Fk+1 + Fk+2 )
2 2
= F2k+2 + (Fk+1 + Fk+2 ).

2
Pero ahora estamos de vuelta con el problema anterior : Cómo hacer Fk+2 +
2 2 2
Fk+1 = F2k+3 ?. Si es ası́, entonces F2k+2 + (Fk+1 + Fk+2 ) = F2k+2 + F2k+3 =
212CAPÍTULO 4. DOS IMPORTANTES PRINCIPIOS: EL DE INDUCCCIÓN Y EL DEL

F2k+4 y la inducción es completa. De esta manera, los problemas estan in-


terrelacionados: la verdad de lo primero depende finalmente de la verdad de
lo segundo, y a la inversa, la verdad de lo segundo depende finalmente de la
verdad de lo primero.

Podemos resolver la dificultad con una prueba de las dos en la siguiente


forma. Considere las dos proposiciones

2
P (n) : Fn+1 + Fn2 = F2n+1 ,
2
Q(n) : 2Fn+1 Fn + Fn+1 = F2n+2 .

P (1) y Q(1) son cada una verdadera, los argumentos previos muestran que
P (k) y Q(k) implican P (k + 1), ahora P (k + 1) y Q(k) implica Q(k + 1).
De ello se sigue que P (k) y Q(k) implican P (k + 1) y Q(k + 1), la prueba se
completa ası́.

4.4.4 Sea f (x) = a1 senx + a2 sen2x + .... + an sennx, donde a1 , ....an son
números reales y donde n es un entero positivo. Suponga que |f (x)| < |senx|
para todo x real, probar que |a1 + 2a2 + .... + nan | ≤ 1.

Solución. Supóngase que intentamos usar inducción sobre el número de


términos en f (x). Cuando n = 1, f (x)
 = a1 senx
 y de la hipótesis
 |f (x)| ≤
|senx|, se sigue que |a1 | = a1 sen π2 = f π2 ≤ sen π2 = 1

Supóngase que el resultado se cumple para k, y considere la función


f (x) = a1 senx + a2 sen2x + .... + ak senkx + ak+1 sen(k + 1)x,

Para alguna elección de números reales a1 , a2 , ..., ak+1 , supóngase que |f (x)| ≤
|senx| para todo x real. Como sen(k + 1)x = senkxcosx + senxcoskx, pode-
mos escribir

f (x) = (a1 + ak+1 coskx)senx + a2 sen2x + ....


+ak−1 sen(k − 1)x + (ak + ak+1 cosx)senkx.

Tenemos ahora reescrito f (x) como una suma de k térmionos, más o menos
del mismo tipo con lo cual ya podemos aplicar la hipótesis de inducción. La
4.4. INDUCCIÓN Y GENERALIZACIÓN 213

dificultad es que los coeficientes de los términos seno en esta expresión son
no constantes; la razón es que algunos de ellos contiene funciones de x. Esto
sugiere considerar el siguiente problema más general.

Sean a1 (x), .....an (x) funciones diferenciables de x, y sea f (x) = a1 (x)senx +


+a2 (x)sen2x + .... + an (x)sennx. Suponga que |f (x)| ≤ |senx| para todo x
real, probar que |a1 (0) + 2a2 (0) + ..... + nan (0)| ≤ 1.

Si podemos probar esta proposición, tendremos resuelto también el problema


original, porque, tomando ai (x) ≡ ai , ai una constante, i = 1, 2, ....., n, para
todo x, cubrimos el problema original.

Nuevamente procederemos por inducción. Estamos suponiendo |a1 (x)senx| ≤


|senx|. Mientras x se aproxima a 0, senx 6= 0, para tales x |a1 (x)| ≤ 1. Ahora
a1 (x) es continua en x = 0, entonces |a1 (0)| ≤ 1. Esto implica que el resultado
se cumple para el caso n = 1.

Ahora supóngase que el resultado se cumple para n = k, y considere la


función

f (x) = a1 (x)senx + a2 (x)sen2x + .... + ak+1 (x)sen(k + 1)x,

donde |f (x)| < |senx| con las ai (x) diferenciables. Como antes, esta puede
ser reescrita en la forma equivalente
f (x) = [a1 (x) − ak+1 (x)coskx] senx + a2 (x)sen2x + .....
+ak−1 (x)(k − 1)x + [ak (x) + ak+1 (x)cosx] senkx

Podemos ahora aplicar la hipótesis inductiva, y cocluir que

|[a1 (0) + ak+1 (0)] + 2a2 (0) + ..... + (k − 1)ak−1 (0) + k [ak (0) + ak+1 (0)]| ≤ 1

pero esto es lo mismo que

|a1 (0) + 2a2 (0) + ..... + kak (0) + (k + 1)ak+1(0)| ≤ 1


214CAPÍTULO 4. DOS IMPORTANTES PRINCIPIOS: EL DE INDUCCCIÓN Y EL DEL

que es la forma deseada (una prueba no inductiva es dada en 6.3.2)

Problemas

4.4.5 Suponga que S denota un retı́culo cuadrado n × n, n ≥ 3. Mostrar


que es posible dibujar un camino poligonal consistente de 2n − 2 segmentos
los cuales pasan a traves de todos los n puntos reticulares de S.

1
4.4.6 Sea f0 (x) = , y defina fn+1 (x) = xfn′ (x). Probar que fn+1 (x) >
(1 − x)
0 para 0 < x < 1.
4.5. RECURRENCIA 215

4.5. Recurrencia

En la segunda solución de 1.1.1, suponga que An denota el número de subcon-


juntos de un conjunto con n elementos. Hemos mostrado que An+1 = 2An ,
A0 = 1. Este es un ejemplo de relación recurrente. Aunque no tengamos
una fórmula explı́cita para An (como el método de inducción requiere), la
relación de recurrencia define una ”espiral ” o algoritmo el cual nos muestra
como calcular An+1 . En esta sección veremos acerca de problemas que pueden
ser reducidos a problemas equivalentes con parámetros pequeños, la idea es
aplicar la reducción con argumentos recursivos hasta que los parámetros al-
canzan valores para los cuales el problema puede ser resuelto.

4.5.1 (Problema de la Torre de Hanoi). Supóngase n anillos, con diferentes


diámetros (en su circunferencia exterior), son pasados sobre una estaca verti-
cal, el más grande en el fondo, en forma de piramide (figura 2.3). Otras dos
estacas verticales son colocadas suficientemente lejos. Deseamos transferir
todos los anillos, uno en una vez, a la segunda estaca para formar una pi-
ramide idéntica. Durante las transferencias, no estamos permitidos a colocar
un anillo sobre otro menor (esto hace necesario usar el tercer anillo). Cual es
el menor número de movimientos necesarios para completar la transferencia?

Solución. Suponga que Mn denota el mı́nimo número de movimientos para


una pila de n anillos. Claramente M1 = 1, ahora suponga n > 1. En orden
hay que conseguir pasar el anillo más grande al fondo de la segunda estaca,
para ello hay que mover los restantes n−1 anillos a la tercera estaca. Esto nos
toma un mı́nimo de Mn−1 movimientos (según nuestra notación elegida). Un
movimiento es necesario para transferir el mayor anillo a la segunda estaca,
finalmente Mn−1 movimientos son necesarios para transferir los restantes n−1
anillos a la segunda estaca. De esta manera

Mn = 2Mn−1 + 1 M1 = 1.

Un proceso de inducción fácil, basado en esta recurrencia, muestra que Mn =


2n − 1,

(Mn+1 = 2Mn + 1 = 2 [2n − 1] + 1 = 2n+1 − 1).


216CAPÍTULO 4. DOS IMPORTANTES PRINCIPIOS: EL DE INDUCCCIÓN Y EL DEL

Figura 2.3.

Suponga que a1 , a2 , ....., an es una permutación de 1, 2, ...., n. Podemos inter-


pretar esta permutación geométrica en la siguiente forma. Tome un tablero
de ajedrez n × n, ahora para cada i, coloque una torre en la i-ésima colum-
na (partiendo de la izquierda) y la ai -ésima fila (partiendo de abajo). Por
ejemplo, la permutación 3, 2, 5, 4, 1 esta representada en la figura 2.4. En
esta manera vemos que una permutación de 1, 2, ...., n corresponde a una
colección de n torres ”inofensivas” en un tablero de ajedrez n × n. Esta cor-
respondencia permite a uno pensar las permutaciones geométricas y utilizar
el lenguaje e imagenes de torres inofensivas en un tablero de ajedrez.
5 R

4 R

3 R

2 R

1 R
1 2 3 4 5
Figura 2.4

4.5.2 Suponga que Qn denota el número de maneras de colocar n torres


inofensivas en un tablero de n × n tal que que el arreglo es simétrico respecto
a la diagonal que va de la esquina inferior izquierda a la esquina supeior
4.5. RECURRENCIA 217

derecha. Mostrar que

Qn = Qn−1 + (n − 1)Qn−2 .

Solución. Una torre en la primera columna puede o no ocupar el cuadro en


la esquina inferior izquierda del tablero. Si ello ocurre, hay Qn−1 maneras de
colocar las restantes n − 1 torres. Si ello no ocurre, una torre puede ocupar
cualquiera de los n − 1 cuadros en la primera columna. Una vez que es
colocada, se determina de manera única la colocación de una torre puesta
simétricamente (simétrica con respecto a la diagonal dada) en la primera fila.
Las restantes n − 2 torres pueden ser colocadas en Qn−2 maneras. Poniendo
esas ideas juntas da el resultado.

4.5.3 Una moneda es lanzada n veces. Cuál es la probabilidad de que dos


caras aparezcan una tras otra en algún momento en la secuencia de tiros?

Solución. Suponga que Pn denota la probabilidad de que no aparezcan dos


3
caras consecutivas en n lanzamientos. Claramente P1 = 1, P2 = . Si n > 2,
4
allı́ hay dos casos.

Si el primer lanzamiento es cruz, entonces dos caras consecutivas no apare-


ceran en los restantes n − 1 lanzamientos con probabilidad Pn−1 (de acuerdo
a nuestra notación). Si el primer lanzamiento es cara, el segundo lanzamiento
debe ser cruz para evitar dos caras consecutivas, entonces dos caras consecu-
tivas no aparecerán en los restantes n − 2 con probabilidad Pn−2 . Ası́ de esa
manera,

1 1
Pn = Pn−1 + Pn−2 , n > 2.
2 4

Esta recurrencia puede ser transformada a una forma más familiar multipli-
cando cada lado por 2n :

2n Pn = 2n−1 Pn−1 + 2n−2Pn−2


218CAPÍTULO 4. DOS IMPORTANTES PRINCIPIOS: EL DE INDUCCCIÓN Y EL DEL

y poniendo Sn = 2n Pn para cada n:

Sn = Sn−1 + Sn−2 .

Esta es la recurrencia para la suseción de Fibonacci (observe que Sn = Fn+2 ).


Ası́ pues, la probabilidad que buscamos es Qn = 1 − Pn = 1 − Fn+2 /2n .

Problemas

4.5.4 Suponga que Pn denota el número de regiones formadas cuando n


lı́neas son dibujadas en el plano Euclidiano de tal manera que no hay tres que
son concurrentes ni dos que son paralelas. Mostrar que Pn+1 = Pn + (n + 1).

4.5.5 .

Suponga que En denota el determinante de una matriz n × n teniendo −1’s


abajo de la diagonal principal (que va de la izquierda arriba a la derecha
abajo) y 1’s en y sobre la diagonal principal. Mostrar que E1 = 1 y que
E2 = 2En−1 para n > 1.

Suponga que Dn denota el determinante de otra matriz n × n cuyo (i, j)-


ésimo elemento (el elemento en la i-ésima fila y la j-ésima columna) es el
valor absoluto de la diferencia de i y j. Mostrar que Dn = (−1)n−1 (n−1)2n−2 .

Suponga que Fn denota el determinante de otra matriz n × n con a en la


diagonal principal, b en la superdiagonal (la diagonal inmediata abajo de la
diagonal principal teniendo n − 1 elementos). Mostrar que Fn = aFn−1 −
bcFn−2 , n > 2. Que pasa cuando a = b = 1 y c = −1?

Evalúe el determinante An n×n, cuyo (i, j)-ésimo elemento es a|i−j| hallando


una relación recursiva entre An y An−1 .

4.5.6 .
4.5. RECURRENCIA 219

Suponga que a1 , a2 , ...., an son números reales positivos y An = (a1 + .... +


1
(n−1)/n
an )/n. Mostrar que An ≥ An−1 ann con la igualdad solo si An−1 = an .(Sugerencia:
aplique la desigualdad de 2.1.5)

La desigualdad entre la media geométrica y la media aritmética. Usando la


parte (a), mostrar que

a1 + ...... + an
≥ (a1 ....an )1/n
n
con la igualdad solo si a1 = a2 = .....an .

4.5.7 Dos jugadores de ping-pong, A y B, acuerdan varios juegos. Los ju-


gadores son igualmente capaces, supóngase, que quien quiera que juegos. Los
jugadores son igualmente capaces, supóngase, que quien quiera puede ser el
jugador A en un juego, o el jugador B en otro). Supongase que A sirve
primero en el primer juego, pero después el perdedor sirve primero. Suponga
que Pn denota la probabilidad de que A gane el n-ésimo juego. Mostrar que

Pn+1 = Pn (1 − P ) + (1 − Pn )Ṗ .

4.5.8 Un estudiante juega a lanzar una moneda equitativa y alcanza un


punto para cada cara que aparece y dos puntos para cada cruz. Probar que
la probabilidad de que el estudiante alcance exactamente n puntos en algún
1 n 
momento en una suseción de n lanzamientos es 2 + − 21 . (Sugerencia:
3
suponga que Pn denota la probabilidad de alcanzar exactamente n puntos en
algún momento. Exprese Pn en términos de Pn−1 y Pn−2 . Use esta relación
recurrente para dar una prueba inductiva).

4.5.9 (Problema de Josephus). Arregle los números 1, 2, ..., n consecutiva-


mente (en el sentido de las agujas del reloj) sobre la circunferencia de un
cı́rculo. Ahora, remueva el número 2 y proceda en el sentido de las agujas
del reloj removiendo cualquier otro número, entre aquellos que quedan, hasta
que solo un número queda. (ası́, para n = 5, los números son removidos en el
220CAPÍTULO 4. DOS IMPORTANTES PRINCIPIOS: EL DE INDUCCCIÓN Y EL DEL

orden 2, 4, 1, 5 quedando solo el 3). Suponga que f (n) denota el número final
que queda. Mostrar que

f (2n) = 2f (n) − 1
f (2n + 1) = 2f (n) + 1

este problema continua en (3.4.5).

4.5.10 .

(a) Suponga que Rn denota el número de maneras de colocar n torres in-


ofensivas en el tablero de ajedrez n × n, también que el arreglo es
simétrico con respecto a una rotación (en el sentido de la agujas del
reloj) de 900 de el tablero alrededor de el centro. Mostrar que

R4n = (4n − 2)R4n−4 ,


R4n+1 = R4n ,
R4n+2 = 0 = R4n+3 .

(b) Suponga que Sn denota el número de maneras de colocar n torres in-


ofensivas en un tablero de ajedrez n × n, también que el arreglo es
simétrico con respecto a el centro del tablero. Mostrar que

S2n = 2nS2n−2
S2n+1 = S2n .

(c) Suponga que Tn denota el número de caminos de colocar n torres in-


ofensivas en un tablero de ajedrez de n × n, también que el arreglo es
simétrico con respecto a las dos diagonales. Mostrar que

Tn = 2,
T2n+1 = T2n ,
T2n = 2T2n−2 + (2n − 2)T2n−4 .

4.5.11 Un (n + 2)-ágono regulares es inscrito en un cı́rculo. Suponga que


Tn denota el número de formas posibles para juntar sus vértices en pares tal
4.5. RECURRENCIA 221

que los segmentos resultantes no se intersectan uno con otro. Si establecemos


T0 = 1, mostrar que

Tn = T0 Tn−1 + T1 Tn−2 + T2 Tn−3 + ...... + Tn−1 T0

(para una continuación de este problema, ver 5.4.10)

4.5.12 Sean a1 , a2 , ....., an una permutación de el conjunto Sn = {1, 2, ...., n}.


Un elemento i en Sn es llamado un punto fijo de esta permutación si ai = i.

a. Un desarreglo de Sn es una permutación de Sn sin puntos fijos. Sea gn


el número de desarreglos de Sn . Mostar que

g1 = 0, g2 = 1,
gn = (n − 1)(gn−1 + gn−2 ), Para n > 2.

(Sugerencia: Un desarreglo, o intercambia el primer elemento con otro


o no lo es)

b. Sea fn el número de permutaciones de Sn con exactamente un punto


fijo. Mostar que |fn − gn | = 1.

4.5.13 Supóngase que n hombres entregan sus sombreros al llegar a un


banquete, como los dejan, los sombreros les son de vueltos tomandolos al
azar. Cuál es la probabilidad de que un hombre no consiga en esta devolución
su propio sombrero? (Sugerencia: Suponga que Pn denota esta probabilidad.
Entonces Pn = gn /n!, donde gn es como en 2.5.13. Sea Cn = Pn − Pn−1 . Use
la relación de recurrencia hallando en 2.5.13 (a) para mostrar que C2 = 1/2,
Cn = −Cn−1 /n. Use esto para mostrar que Pn = 1/2! − 1/3! + .... + (−1)n /n!.
Entonces para n grande Pn ≈ 1/e)

4.5.14 .

R π/2
(a) Sea In = 0
senn xdx. Hallar una relación de recurrencia para In .
222CAPÍTULO 4. DOS IMPORTANTES PRINCIPIOS: EL DE INDUCCCIÓN Y EL DEL

(b) Mostrar que


1 × 3 × 5 × ..... × (2n − 1) π
I2n = .
2 × 4 × 6 × ....... × 2n 2
(c) Mostrar que
2 × 4 × 6 × ..... × (2n − 2)
I2n+1 = .
1 × 3 × 5 × ....... × (2n − 1)

Ejemplos adicionales

1.1.1(Solución 2), 4.3.9, 5.3.5, 5.3.14, 5.3.15, 5.4.8, 5.4.9, 5.4.24, 5.4.25, 5.4.26.
Estrechamente relacionadas la inducción y la recursión son argumentos basa-
dos sobre ”argumentos repetidos”. Ejemplos de que eso ha significado aquı́ son
4.4.4, 4.4.17, la prueba de el teorema del valor medio en 6.1, 6.1.5, 6.1.6,
6.3.6, 6.8.10, y la heurı́stica para el significado geométrico de una desigual-
dad arimética dada en la sección 7.2

4.6. Principio del Palomar

Cuando una colección suficientemente grande de objetos dividida en un número


suficientemente pequeño de clases, una de las clases contendrá un ciertamente
mı́nimo número de objetos. Este hecho se preciso más en la siguiente proposi-
ción evidente:

Principio del palomar. Si kn + 1 objetos (k ≥ 1) son distribuidos entre n


cajas una de las cajas va a contener al menos k + 1 objetos

Este principio, incluso cuando k = 1, es una muy poderosa herramienta para


probar teoremas de existencia.

4.6.1 Dado un conjunto de n + 1 enteros positivos, ninguno de los cuales


excede a 2n, mostrar que al menos un miembro de el conjunto debe dividir a
otro miembro de el conjunto.

Solución. Este es el mismo que el de 2.2.7, donde estaba planteado por


inducción sobre n. Sin embargo, el problema es realmente un problema de
4.6. PRINCIPIO DEL PALOMAR 223

existencia para un n dado, y puede ser llevado muy atinadamente hacia el


principio del palomar, como veremos en seguida.

Suponga que los números elegidos se denota por x1 , x2 , ....., xn+1 , ahora para
cada i escribimos xi = 2ni yi , donde ni es entero no negativo y yi es impar.
Sea T = {yi : 1, 2, ....., n + 1}. Entonces T es una colección de n + 1 enteros
impares, cada uno de ellos menor que 2n. De aquı́ que allı́ hay solo n números
impares menores que 2n, el principio del palomar implica que dos núemros
en T son iguales, digamos yi = yj , i < j. Entonces

xi = 2ni yi y xj = 2nj yi .

Si ni ≤ nj , entonces xi divide a xj ; si ni > nj , entonces xj divide a xi . Esto


completa la prueba.

4.6.2 Considere cinco puntos cualesquiera P1 , P2 , P3, P4 , P5 en el interior de


un cuadrado S de lado con longitud 1. Denote por dij la distancia entre los
puntos
√ Pi y Pj . Probar que al menos una de las distancias dij es menor que
2/2.

Figura 2.5

Solución. Divida S en cuatro cuadrados congruentes como se muestra en


la figura 2.5. Por el principio de palomar, dos puntos pertenecen a uno de
esos cuadrados (un punto en la frontera de dos cuadrados pequeños puede ser
considerado
√ de ambos cuadrados). La distancia entre esos puntos es menor
que 2/2.
224CAPÍTULO 4. DOS IMPORTANTES PRINCIPIOS: EL DE INDUCCCIÓN Y EL DEL

Figura 2.6

4.6.3 Supóngase que cada cuadrado de un tablero de ajedrez 4 × 7, como


se muestra en la figura, es coloreado en blanco o negro. Probar que en una
tal coloración, el tablero debe contener un rectángulo (formado por las lı́neas
horizontales y verticales de el tablero), tal como el resaltado en la figura 2.6,
cuyos cuadrados de las distintas esquinas son todos del mismo color.

Figura 2.7

Solución. Un tal rectángulo existe incluso en un tablero 3×7. El color de las


configuraciones de cada columna debe de ser de uno de los tipos mostrados
en la figura 2.7. Suponga que una de las columnas es del tipo 1. Habremos
terminado si una de las 6 restantes es del tipo1, 2, 3 o 4. Suponga pues que
cada una de las seis columnas restantes es del tipo 5, 6, 7 u 8. Entonces por
el principio del palomar , dos de esas seis columnas deben ser del mismo tipo
y habremos terminado.

El mismo razonamiento se hace si una de las columnas es del tipo 8.

Ahora suponga que ninguna de las columnas es del tipo 1 o del tipo 8.
Entonces tenemos siete columnas pero solo seis tipos. Por el principio del
palomar, dos columnas deben tener el mismo tipo y la prueba es terminada.
4.6. PRINCIPIO DEL PALOMAR 225

4.6.4 Probar que existen tres enteros a, b, c no todos cero y cada uno con
valor absoluto menor que un millon, tales que

√ √
a + b 2 + c 3 < 10−11 .

√ √
Solución. Sea S el conjunto de 1018 números reales √ r + s √2 + t 3 con r, s, t
cada uno en {0, 1, 2, ...., 106 − 1}, ahora sea d = (1 + 2 + 3)106 . Entonces
cada x en S está en el intervalo 0 ≤ x < d. Dividiendo este intervalo en
1018 − 1 subintervalos iguales, cada uno de longitud e = d/(1018 − 1). Por el
18
principio del palomar, dos de los 10 √ números
√ de S deben estar en el mismo
subintervalo. Su diferencia, a + b 2 + c 3, dá los deseados a, b, c. ya que
e < 107 /1018 = 10−11 .

4.6.5 Dado un conjunto de diez números naturales entre 1 y 99 inclusive


(notación decimal), probar que allı́ existen dos subconjuntos no vacios y dis-
juntos, del conjunto con sumas iguales de sus elementos.

Solución. Con la elección del conjunto de diez números, podemos formar


210 − 1 = 1023 (diferentes) subconjuntos no vacios. Cada uno de esos subco-
juntos tiene una suma menor que 1000, ya que incluso 90+91+. . .+99 < 1000.
Por tanto por el principio del palomar, dos subconjuntos A y B deben tener
la misma suma. Ası́ por extracción de los elementos que pertenescan a ambos
conjuntos obtenemos dos conjuntos disjuntos x = A−A∩B y y = B −A∩B,
con la misma suma. (Ni x ni y son vacios, porque esto significarı́a que: A ⊂ B
o B ⊂ A, lo cual es imposible, ya que la suma de sus elementos da el mismo
número.)

Problemas

4.6.6 Sea A conjunto de 20 enteros distintos elegidos de la progresión ar-


itmética 1, 4, 7, . . . , 100. Probar que existen dos enteros distintos en A cuya
suma es 104.

4.6.7 .
226CAPÍTULO 4. DOS IMPORTANTES PRINCIPIOS: EL DE INDUCCCIÓN Y EL DEL

a. Sea S una región cuadrada (en el plano) de lado con longitud 2 pulgadas.
Mostrar que entre cualesquiera nueve puntos en S, hay tres los cuales son los
vértices de un triángulo de área ≤ 1/2 de una pulgada cuadrada.

b. Diecinueve dardos son lanzados hacia un blanco el cual tiene la forma de


un hexágono regular con lado de longitud
√ un pie. Mostrar que dos dardos
están a una distancia de a lo más 3/3 pies uno de otro.

4.6.8 Mostrar que si hay n personas en una fiesta, entonces dos de ellas
conocen el mismo número de personas (entre aquellas presentes).

4.6.9 Quince sillas son equitativamente colocadas alrededor de una mesa


circular en la cual estan asignadas fichas para quince invitados. Los invitados
ignoran la información de estas fichas hasta después que se han sentado,
y resulta que ninguno está sentado frente a su propia ficha. Probar que la
mesa puede ser rotada de manera que al menos dos de los invitados estén
simultaneamente sentados correctamente.

4.6.10 Sea X un número real. Probar que entre los números

x, 2x, . . . , (n − 1) x
hay uno que difiere de un entero por a lo mas 1/n.

4.6.11 .

(a.) Probar que en un grupo de seis personas hay tres que, o bien son mutu-
amente conocidas o bien son mutuamente desconocidas. (Sugerencia: Repre-
sente las personas con los vértices de un hexágono regular. Una dos vértices
con un segmento de lı́nea rojo si el par representado por esos vértices es de
conocidos de lo contrario una esos vértices con un segmento de lı́nea azul.
Considere uno de los vértices, llamemosle A. Al menos tres segmentos de
lı́nea parten de A teniendo el mismo color. Aqui hay dos casos a considerar.)
4.6. PRINCIPIO DEL PALOMAR 227

(b.) Diecisiete personas mantienen correspondencia entre si (cada uno con


todos los demás) por correo. En sus cartas solo tres temas son discutidos.
Cada par de correspondencias trata solo uno de los temas. Probar que hay al
menos tres personas que se escriben unos a otros sobre el mismo tema.

4.6.12 Probar que no existe un conjunto de siete enteros positivos, no may-


ores de 24, en el cual sus subconjuntos tienen todos sumas diferentes.

Ejemplos adicionales

1.10.1, 3.2.1, 3.2.5, 3.2.19, 3.2.20, 3.3.24, 4.4.10.

Potrebbero piacerti anche